You are on page 1of 184

Major Topic

Abnormalities of Teeth
Disesses

Abbreviation Abn ofTth


Bld Dis Con Tis Les

Major Topic
Odontogenic Tnmors Pigmetrted Lsions of the Orat Covitv
Pseudocyst

Abbreviation
Odont Tum Pig Les of Oral Cav
Pseudocysts

ofthe Blood

CoDnectiye Tissue Lesions Itrfl rmmatory Jaw Lesions

Infl Jaw Les M&G Jaw Dis


Misc.
Neo

Red-Blue Lesiors

R-B Lcsions
SG Tum

Metabolic & Genetic


Jaw Diseases Miscellaneous
Neoplasms

Salivary Ghnd Tumors


Terms UlcratiYe.Conditions
Diseases

Terms Ulc Cond

Nerve & Muscl

Nrv&Mus Disord Vesiculo-Bullous


N-O Cyst

V-B Dis
V-P Les

Disorders
Non-Odontogenic Cysts Non-Odontogenic Tumors Odontogenic Cysts

Verrucal-Prpillary
Lesions

N-O Tum
Odont Cyst

White Lesions

W Les

ORAL PATHOLOGY

Abn of Tth

The congenital absence of many, but not all, teeth is known as:

. H,?odontia

. Oligodontia
. Diphyodontia
. Anodontia

CopyriSht O 20ll'1012

' Denul

Decks

Tlr o forms of anodontia:

L Complete true: is a rare condition in which alt of the teeth are missing. It may involve both the primary and permanent dentitions. It is usually associated with hereditarv ectodermal dysplasia. See picture #l in booklet :. Partial anodontia (commonly referred to as congenitally missing teeth): is rather cor non. Teeth usually affected include the third molars (maxillary more often than nandibular), maxillary lateral incisors ald mandibular second premolars. Note: As a general rule, if only one or a few teeth are missing, the absent tooth will be the most distal tooth ofany glen type (if molat then it would be the third molar). See picture #2 in booklet

Other terms to b familiar with include:

. Ofigodontia: refers to the congenital absence of many (usually

six or more) but

not

all, teeth . Hypodontia: refers to the absence ofonly a few teeth . Diphyodontia: is having 2 successive sets of teeth (as in humans) as opposed to pol)?hyodontia fraving more than two sets of teeth in a ldetime) . Hypsodontia: having teeth with high crowns /cusps). Hypsodont dentition is associated with a diet ofabrasive foo<is

PATHOLOGY

Which type of dentinogensis imperfecta features multiple pulp exposures, perirpical rtdiolucencies, and a variable radiographic appearance?

. Type I

' '

Type Type

II III

CoDrighr C 20ll-201: - Denral Decks

The permanent maxillary centrals in the x-ray below are vital. What is the most probabl interpretation ofthe condition illustrated?

. Mesiodens . Concrescence

. Fusion . Dens in Dente

Cop).righr C 201 l'201 2 - Denlal Decks

Dentinogenesis imprfecta is an autosomal dominant condition in which there is an intrinsic alteration in the dentin. lt affects dentin ofboth the primary and the permanent dentition. It is also known as hereditary opalescent dentin.
Dentinogenesis imperfecta has been divided into three types: . Type I: dentin abnormality occurs in patients that have ostogenesis imperfecta fcra,'acterized blt blue sclera or a history of bone frqchu?s). In this form, primary teeth are more severely affected than permanent teeth. . Tlpe lI: most common, only the dentin abnormality exists with no bone involvement.

'Tlpe III

(Brandl'whe

there are clinical and radiographic vadations in this type. Features of type III that are not seen in type I and ll include multiple pulp exposures, periapical radiolucencies, and a

\pe)

bke Type

lI, only the dentin abnormality exists, however,

variable radiographic appearance

Important: Clinically, dentinogenesis imperfecta is usually easily detected and identified.


The teeth exhibit an unusual translucent or opalescent appearance with color variation fiom yellorv-brorvn to gray. The entire crown appears discolored owing to the abnormal underlying dentin. See picture #3 in booklt Other characteristics ofthe teeth include: . Although the enamel is structurally and chemically normal, it fractures easily, resulting in rapid tvear (due to poor dentin support) . There is excessive constdction at the CEJ, giving the crowns a tulip or bell shape . Short roots . Obliterated pulps (\'pe I and I1 only) *** Note' In Type IlI, the dentin appean thin, and the pulp chambers and root canal are extremely large, giving the appearance ofthin dentin shells. See picture #4 in booklet Treatment is full crown coverage for cosmetic purposes. Note: These teeth should not be used as abutments, because the roots are prone to fractue under stress.

Mesiodens is the most common suprnumerary tooth, appearing singly or in pairs as a small tooth with a cone-shaped crown and a short root between the maxillary central incisors; it may be erupted, impacted or even inverted.

\Iesiodens appear situated in the maxilla near the midline and almost always posterior to the normal central incisors. Many of them, therefore, are bypassed by the permanent incisors $'hich are permitted to erupt into their normal position in the arch. Remember:

. Fusion:

is a developmental union of two or more teeth in which the dentin and one other dental tissue are united /rn ay be the root). See picture #6 in booklt . Concrescence: is a condition in which only the cementum of two or more teeth becomes united. See picture #7 in booklet . Dens in dente /a/so called dens invaginatus)r means "tooth within a tooth"' it is caused by a deep invagination ofthe enamel organ during formation, most likely found associated with a maxillary lateral incisor. See picture #5 in booklet

. Molars

.lncisors

. Premolars . Canines

4
Cop)riglt
@

201l'2012 - Dnral Decks

\-,
. Enamel pearls

An enamel defect resulting from the incomplete formstion of the enamel matrix is ealld:

. Enamel hypocalcifi cation . Enamel hypoplasia


. Regional odontodysplasia

Coplright O 201 I -20

12

- Denral Decfts

Hypercementosis is often confined to the apical half of the root but, in some instances, may involve the entire root. In large majority of instances, it affecls vital teeth, is not associated with any one particular systemic disease and may be regarded as a dental anomaly. It rnay be seen when a tooth has lost its antagonist or when there is chronic inflammation of the tooth. The premolars are most frequently involved. Next in frequency are the first and second molars. Hypercementosis produces no significant clinical signs or symptoms indicative of its presence. It is seen radiographically as a bulbous enlargement that has surrounding it a continuous and unbroken periodontal membrane space and a normal lamina dura. See

picture below.
Note: There is a form of hlpercementosis which is a common feature in Paget's disease that involves thejaws. In addition to Paget's disease, hypercementosis is associated with supraeruption, apical periodontal infection, occlusal trauma, toxic thyroid goiter,
acromegaly, and pituitary gigantism.

Example of gneral hypercementosis: Maxillary premolars

Enamel hypoplasia is a developmcntal def'ect in which the cnamel ofthe tccth is hard in contcxt but thin and dcficient in amount. It results from incomplete formation ofthe enamel matrix with a deficiency in the cementing substance. Enamel hypoplasia a1l'ects both the deciduous and pcnnanent teeth. It is usually due to illness or injury during tooth formation or due to a genetic disorder Note: Thc gcnetic lbrms ofcnamel hypoplasia arc generally considcred to be types ofamelogenesis imperfecta. The clinical appearance ofenamel hypoplasia includes: l) the lack ofcontact between teeth, 2) the rapid breakdown ofocclusal surfaces, 3) a yellowish-brorvn stain that appears whcre the dentin is exposed. Note: lfonly one permancnt tooth is affected, it is usually caused by physical damagc or

periapical infections to the primary tooth that this pernanent tooth replaced. This is known

as

"Turner's h-vpoplasia."
Remember: Enarncl hypocalcification is a hereditary dental dcfcct in which thc enamel is soft and undercalcificd in context yct normal in quantity. It is caused by the dcfective maturation of amcloblasts ftrere r.r a defect in the mineraliz tion ofthe fonned matrir). The teeth are chalky in consistency, the surfaces wear down rapidly and a yellow to brown stain appears as the underlying dentin is exposcd. This condition affects both the deciduous and permanent teeth as well. See picture #9 in booklt involves the hard tissues that are derived from both epithelial (enarnel) an<l mesenchymal (dentin and cemenhnt) components of thc tooth-fonning apparatus. Thc tccth in a region or quadrant of the maxilla or mandible are affccted to thc extent that they exhibit short roots, open apical foramina' and enlarged pulp chambers' The thinness and poor mineralization quality ofthe enamel and dentin layers havc given rise to thl; term "ghost teeth." The permanent teeth are aflected more than the primary teeth , and the maxillary anteriol teeth are affccted more than other teeth. The cause is unknown, although nume.ous etiologic factors have been suggested /1.e., trauma, nutritional defcie cies, inlbcliotts). Because ofthe poor quality ofthe affected teeth, their rcmoval is usually indicated.

\ote: Regional odontodysplasia

ORAL PATHOLOGY

Abn of Tth

Match the following alterations in tooth rnorphology on the lft with the correct diagram depicting the alteration on the right.

. Fusion . Twinning

Concresence

. Gemination

Copyng}r i)

l0lt :0ll

Dental De.ts

ORALPATHOLOGY

Abn of Tth

All of the following statements concerning amelogenesis inperfecta rre true EXCEPT one. Which one is the EXCEPTIOM

.It

is an inherited condition which is transmitted as a dominant trait

. Because ofthe enamel malformation, the teeth ofindividuals with amelogenesis imperfecta are often discolored, sensitive to temperature changes, and painful to brush

. It only affects . It

the permanent teeth

causes the enamel of the teeth to be soft and thin

. The teeth appear yellow. because the dentin is visible through the thin enamel . The teeth are easily damaged and susceptible to decay
7 CopyriShr C

l0ll-:0l2

- Derral Decks

Remember: . Gemination is a division of

a single tooth germ by invagination. This results in the incomplete tbrmation oftwo tceth. Frcquently occurs in the incisor region . Fusion is an abnormally shaped tooth that may appcar as an extra wide crown, a normal crown rvrth an extra root. or other combinations fesulting from the union of two adjacent tooth gcrms by dentin during development. . Concresence is the union ofthe roots of two or more normal teeth caused by con'flucnce of their

cemental surfaces.

. Trvinning

is the completc division of a single tooth bud. The dividert teeth are secn as com-

pletelySepalatewithnoconnectiontoeachotherexcepteachtendstobemirrorimageofthe
other . Ank:-losis is the fusion of the cementum or dentin to the surrounding alveolar bone after loss oflhe intervcning periodontal membrane lt is associated \Pith hypodontia The most commonly ankylosed tooth is the pfimary second molar; the pemanent second premolar is the tooth that fails to de\ elop and crupt. Clinically, percussion ofthe anlllosed tooth produces a drrll sound \ote: There is a change in the continuity ofthe occlusal plane which is caused by the continued eruprion ofthe non-ankylosed teeth and the growth ofthe alveolar process clinicallJ,, gemination and fusion can look similar. when counting these tceth, the "larger" tooth should It cc)unred as a single tooth. ln gemination, the patient has a larger tooth but a normal numbel ofteeth
o\ erall. In fusion, the patienl has a larger tooth, but one lcss tooth than normal

"

-{melogenesis imp erlecta (AI) is a relatively rare group ofinherited disorders characterized bl abnormal enamel formation. The term amelogenesis imperfecta is reserved for hereditary defects ofenamel that are not associated with defects in other parts ofthe body or other health problems. lt is a hereditary ectodrmal defect, unlike dentinogenesis imperfecta which is a hereditary rnesodemral defect.

AI enamel defects are highly variable and include abnomralities that are classified as hypoplastic defect in amount ofenamel),hypomaturatron (defect in fnal grov'tlt and mqhration ol'euottel crystallites), andhypocalcified (defect in initial crystallite Jbnnation followed by dete. ti\ e gt'ovlh). The enamel in both the hypomaturation and hypocalcified Al rypes is not mineralized to the level of normal enamel and can be described as hypomineralized Al can be intrerited as an x-linked, autosomal recessive (,4RJ, or autosomal dominant (AD) cor'ditiorr.
The The color ofthe enamel ranges from white opaque to yellow to brown, it is reduced in volume and pitled. Contact points between teeth are often open and occlusal surfaces and incisal edges

frequently are severely abraded. Although the enamel are soft and irregular, there is no increase jn caries mte. Note: Open bite is a common clinical finding.
The radiographic findings are fiequently distinctive and pathognomonic. when the enamel is totally absent, the radiographic appearance makes the diagnosis obvious. When some enamel is present, thin radiopaque coverings on the proximal surfaces ofthe teeth are noted. When the anatonic crown forms are normal or nearly normal, the softness ofthe defective enamel may not be easily distinguished from the dentin. ln all cases, howevel the dentin' pulp and cementum are unaffected by the disease process itself(arlr*e dentinogenesis inperkdq).
See

picture #8 and #10 in booklet

Exception: Amelogenesis imperfecta will only show pulp obliteration abrasion with secondary dentin formation.
Other than cosmetic restoration, no treatment is necessary.

if

there is advanced

. Erosion . Abfraction
. Attrition

. Abrasion

8
Cop).dght O 20ll-2012 - Denhl Decks

. Dentinogenesis imperfecta

. Erythroblastosis fetalis

. Porphyria
. Fluorosis . Diabetes mellitus

. Pulpal injury . Intemal resorption


. Tetracyclines
Coplrighr O
201

9 l'2012 - Dental Dcks

See

picture #13 in booklet

Types ofabrasion:

l. Toothbrush abrasion: most olten results in V-shaped wedges at the cervical margin in the canine and premolar areas. lt is caused by the use ofa hard toothbrush and/or a horizontal brushing stroke and/or a gritty dentifrice. 2. Occlusal abrasion: results in flattened cusps on all posterior teeth and wom incisal edges. lt results from the chewing or biting ofhard foods or objects and chewing tobacco.

. Attrition is the wearing away of enamel and dentin due to the normal function or most commonly, due to the excessive grinding or gdtting together oftecth by the patient lreJbrced to a.r bruxism). The most noticeable effects ofattrition are polished facets, flat incisal edges. discolored surfaces ofthe teeth and exposed dentin. Facets usually develop on the linguoincisal of the maxillary central incisors, the facioincisal of the mandibular canines and the linguoincisal ofthe maxillary canines. See picture #14 in booklet

. Erosion

is the loss of tooth structure from non-mechanical means. It can result from drinking acidic liquids or eating acidic foods. It is common in bulimic individuals as a result of regurgitated stomach acids. It affects smooth and occlusal surlaces. See picture #12 in booklet

. Abfraction lesions are cervical erosive lesions that can not be attributed to any particular cause: causing the enamel to "pop'' off starting at the base ofthe tooth and exposing the gum Iine ofthe tooth to excessive wear See picture #11 in booklet

Cause Dentinogenesis imperfecta


Erv1hroblastosis fetalis

Intrinsic Stain
Translucent or opalesccnt hue; usually gray to bluish-brown Bluish-black, greenish-bluc, tan or brown Red or brownish

Poryhyria
Fluorosis Pulpal injury

white opacities or light brown to brownish-black


Starts pink and usually becomes orangc-brown to bluish-black

Intcrnal resorption
Tetracvcline pomineralized enamel

Pinkish Varies from light yel)ow-orangc to dark gray-brown; depending on the tlDe oftetracycline used and the duration oltherapy

H1

White to brown

E\trinsic stains can be caused by bacteria, iron, tobacco, foods, beverages, gingival hemorhage. restorative materials and medications.
Treatment:

. lIost
stain

extrinsic stains can be removed with abrasives; reduce or eliminate cause

of

. Intrinsic stains may

need bleaching, esthetic restorations, or prosthetic rehabilitation

ORAL PATHOLOGY

Abn of Tth

What is th most probable diagnosis of the x-ray below?

10

Coptright

(l

20l

l-:0 l 2'

Denral Decks

ORAL PATHOLOGY

Abn of Tth

A patient with which typ of amelogenesis imperfecta will have teeth that demonstrate enamel that varies from thin and smooth to normal thickness with grooves, furrows and/or pits?

. Type | (H.vpoplastic) .
Ty-pe

II (H|pomqturation)

. Type III (Itvpocalcified)

11

CopyriShr C

20ll'101:

- Denral Dccks

Intrnal resorption is an unusual form oftooth resorption that begins centally within the tooth. Resorption ofthc dentin ofthe pulpal walls may be seen as part ofan inflarnmatory response to pulpal injury, or it may be seen in cases in which no apparent trigger can be idcntificd. Most cases of intemal resorption present no early clinical symptoms. The first evidence of th lesion may bc thc appcarancc ofa pink-hued area on the crown of thc tooth, which reprcsents the hyperplastic, vascular pulp tissuc filling thc rcsorbed areas. Ifthe condition is discovered before perforation ofthe crown or root has occured, endodontic therapy may be canied out with thc cxpectation ofa pretty high success rate.
Idiopathic external resorptive lsions arc charactcrized by thc invasion ofthe ccrvical rcgion ofthe root by fibrovascular tissue which progressively resorbs dentin, enamcl and cementum. Thc sourcc of the resorption is in thc attachmcnt apparatus. This is the key to understanding diagnosis and treatment. The dental pulp remains protected by an intact layer of dentin and predentin until late in the process. Since the source of the lesion is not in the pulp, endodontic treatment by itselfis ofno assistance in dealing with the sourcc ofthe problcm.

Arrow points to external root resorption of the maxillary


left cntral incisor

Crown size varies from Varies fiom thin and smooth to small to normal. small teelh normal rhickness with grooves, nay Iack proximal confirrrows andlor pits lacts, color varies from normal to opaque white-ycllow Varies flom creamy opaque Normal thickness with enamel to marked yello\r',/brown. that ofien chips and ab8des surfacc ofteeth sofl and easily rough. dental sensitivity and open bite are common
Opaque nhite to yellowbrow_n. soft rough enamel surface. dental sensiti.i,ity and open bite are common. hea\,1 calculus formation is

Enamel has normal to slightiy reduced contrast

Enamel has contrast

similar to or less than


clcntrn. uncrupted crowns

have normal morphologv Enamel has contrast

Normal thickness with enamel that ofien chips and abiades easily

similar to or less than


dentln, unerupted crowns have normal morphology

See

pictures #15, #16, and # l7 in booktet

Remember: L Amelogenesis imperfecta is a hereditary ectodermal defect, unlike dentinogenesis imper_ fecta which is a hereditary mesodermal defect. f. In all three types ofame)ogenesis imperfecta, the dentin, pulp and cementum are unaffected by the disease process itself /anlte dentinogenesis imperfecta).

. Normal

. Somewhat smaller
. Extremely large

. Completely obliterated

't2
Cop)right O 201l-2012 - Denral Decks

Leukemi! is a group ofbone mrrrow diset$es involving an uncontrolled incrase ln:

. Red blood cells (Erythocytes) . Platelets

Plasma cells

. White blood cells (Leukocytes)

13 Coplri8hr O
201 |

'2012 - Dnral Decks

Dentin dysplasia is another autosomal dominant trait that affects dentin. AII teeth ofboth dentitions are affected. This condition has not been associated with any systemic connective tissue disorder. This is a rare condition that has been subdivided into type I or
radicular type and a more rare type II or coronal type:

. Type | (radicular

cl.,-.splasia): more common type

. Both dentitions

are normal in color and shape . The teeth are generally mobile, frequently abscess and can be lost prematurely . Teeth show greater resistance to caries than do normal teeth Radiographic features: . Extremely short roots . Obliterated pulp chambers and root canals before eruption . Residual fragments of pulp tissue appear typically as horizontal lucencies
(

cnevrons)
c.vst-r')

. Periapical radiolucencies (granulomo,s or


. Type

around the defective roots

. Color of primary teeth is opalescent . Color of permanent teeth is normal . Coronal pulps ofpermanent teeth are lilled rvith globules of abnormal dentin

Il

(coronal dy.splasia)

r"rsually enlarged ("thi,\tle

lube") a

J arc

Radiographic features: Deciduous teeth are similar in appearance to type I, but permanent teeth exhibit enlarged pulp chambers ("thistle nbe") in appearance . .\bsence of neriaoical radiolucencie"

is a lomr ofcancer that begins jn the blood-forming cells ofthe bone marrow (the soft, innet part o.l'1he botret. Undcr nomal circumstances, the blood-fonning, or hematopoictic, cclls of thc bone marrow make leukocytes to defend thc body against infectioLls organlsms such as r inrses and bacteria. But il'sot'ne leukocytes are damaged and remain in an inmlature fbrm' they becornc poor int'ection fighters that multiply excessivcly and do not dic olfas thcy should.

Leukemia

The leukcmic cells accumulate and lesscn the production of oxygcn-carrying red blood cclls /eryrlr r or r les/. blood-clo tting cells (plaletets), and normal lcukocytes. If untrcated, the surplus leukemic cells or er$helm the bone marrow, enter the bloodstream. and eventually invadc other parts ofthe btrdr. such as the lymph nodes, spleen, livcr, and central nervous syslern (brcin, "^Pfualcoldr' In this * ar. the bchar ior of leukcmia is different than that of other canccrs, which usually bcgin in major organs and ultimatcl), spread to thc bone marrow.

Thcrc are more than a dozen varictics ofleukenia, but thc follorving four types arc thc most comnlon:

L -\cute lymphocytic lcnkcrlta (ALL). most common type in children le]rlemia (AML.): most malignant type j. Chronic lymphocytic lcukernia fczlr: least malignant type -1. Chronic myclogenous leukcmia (CML)t 2 distinct phases, invariably latal
1. .\cute myelogenous

lmportant: . Leukemia is classified by thc dominant cell type . Leukemia can modify thc inflammalory reaction
and by the

duration from onset to death

. Leukemia's affect on teeth/gingiva: Duc to a decrease in immune responsc, periodontal problenrs can be aggravated. Gingival enlargement can be a finding rvith leukemia Additionally, deficits with platelets can bc scen intraorally with increased hemorrhaging,/bruising."

BId Dis

A 48 year old female patient walks into your office. She states that she is diagnosed with some disease which she can't remember the name of. Her physician wants her to follow up with you, her dentist, regularly to watch out for cancer of the tongue and throat. She also has a bald tongue, and states that her fingernails "look funny." What disease does she have?

. Aplastic anemia . Plummer-Vinson syndrome


. Pernicious anemia . Cushing's syndrome

14
Copvrighr ali 2011'201: - DenralDecks

ORAL PATHOLOGY

Btd Dis

What disorder is a result of a genetic mutation causing the substitution of glutamic acid by a valine and results in dental radiographs with enlarged marrow spaces?

. Cystic fibrosis . Muscular dystrophy


. Polio

. Sickle-cell anemia

15

Copvrigl'r

aC

201l'2012

' DenlalDecks

Plummer-Vinson syndrome characterized by an iron-deficiency anemia, atrophic


changes in the buccal, glossopharyngeal, and esophageal mucous metnbranes, koilonycha (spoon-shaped./inger rail.s), and dysphagia. The dysphagia is due to an esophageal stricture or web. SCC of the tongue and throat are complications. lt is most common in middle aged women, rarely in the male. The etiology unknown. Because ofthe predisposition to the development ofcarcinoma ofthe oral mucous membranes, it is essential that the di-

agnosis be established early so that treatment can be given ASAP This includes administration of iron, vitamin B complex and a high protein diet.

Aplastic anemia is a form of anemia in which the capacity of the bone marrow to generate red blood cells is defective. Two types:

l. Primary: unknown cause, affects young adults. The signs and symptoms include pallor, weakness, malaise, dyspnea (dilfcul4, breathing), headache and vertigo. Oral symptoms include spontaneous bleeding,bnstsing (petechlaeT and gingival infections.

It

is usually fatal. 2. Secondary: caused by exposure to toxic agents, such as radiation, chemicals or drugs (fbr exanple, chloramphenicol). It can occur at any age. Symptoms are the same
as primary. Prognosis is good once you remove the cause.

*** Aplastic anemia is the most serious and life-threatening blood dyscrasia associated s ith drus toxiciw.

Sickle-cell anemia is an inherited disease in which the red blood cells, normally discshaped, become crescent shaped, As a result, they function abnomally and cause small

blood clots. These clots give rise to recurrent painful episodes called "sickle cell pain crises." Sickle-cell anemia (also called sickle-cell disease) is the result ofthe production ofabnormal hemoglobin (Hemoglobin S/ due to a genetic defect. It is carried as a trait by 10% of African Americans and 0.2olo have sickle-cell anemia. lt is more common in females and usually clinically manifests itselfbefore the age of30. The typical signs ofanemia are present. The patient is weak, short ofbreath and easily fatigued. Muscle and joint patns are common.

Denlal radiographs ar oftn of diagnostic value: marrow spaces are markedly enlarged because ofthe loss ofmany trabeculae; the trabeculae, which are present, are often abnormally prominent. Occasionally, osteosclerotic areas are noted in the midst of large radiolucent marow spaces. However, the lamina dura and the teeth are unaffected. See picture #18 in booklet

\ote: The gene defect is a known mutation ofa single nucleotlde (th),minelbr an adenine) ofthe beta-globin gene, which results in glutamic acid to be substituted by valine. Sicklecell anemia occurs when a person inherits two abnormal ger'es (one front each parent). lf a person inherits one abnormal gene for the disease, they have what is called sickle cell trait. The life span ofred blood cells is reduced from 120 to 20 days.

ORAL PATHOLOGY

Bld Dis

The category of pigmented (usually purple) lesions in the skin caused by xtravasation ofblood from the capillaries is known as:

Petechiae

. Ecchymosis
. Varicose veins

16

Copyrighr al 2011-2012 - Dental Decks

ORAL PATHOLOGY

Bld Dis

All of the following statements are true EXCEPZ one. which one is the rxcEPZl0M

. .

Acute leukemias have a slow onset and progression Acute leukemias are characterized by the appearance of immature, abnormal cells in the bone marrow and peripheral blood and frequently in the liver, spleen, lymph
nodes, and other parenchymatous organs

The clinical picture of acute leukemias are marked by the effects ol anemia, which is usually severe (/ittigue, molaise), an absence of functioning granulocles (proneness to inlbction and inflammation), and thrombocytopenia (hemorrlrugic diathesis)

The spleen and liver usually are moderately enlarged, while enlarged lymph nodes are seen mainly in acute lymphocytic leukemia. Fever and a very high ESR are
found

Leukocyte counts vary greatly in the acute leukemias


17 Copyrighr /(, 2011'2011 Dental Decks

Purpura spots (pinpoint spots) are purplish discolorations in the skin produced by small bleeding vessels near the surlace ofthe skin. Purpura may also occur in the mucous membranes (such as the lining oJ the mouth) and in the intemal organs. Purpura by itselfis only a sign of other underlying causes of bleeding. Purpura may occur with either normal platefet counts hrcnlhrombocftopenic purpuras) or decreased platelet counts (thromboq)topenic purpltra.rr. Platelets help maintain the integrity of the capillary lining and are irnportant in the clotting process. Note: Large purpura spots (> 3 mn) are called ecchymoses.

Major kinds of purpura: . Thrombocytopenic purpura (Werlhol's tlisease):

a bleeding disorder characterized

by a deficiency in the number olplatelets. This resr.rlts in multiple bruises, petechiae, and hemonhage into the tissues . Thrombotic thrombocytopenic purpura (TTP): a severe and frequently fatal form characterized by a low platelet count in the blood due to consumption of platelets by thrombosis in the terminal arterioles and capillaries ofmany organs

Oral manifestations of thrombocytopenic purpura: . Severe and profuse gingival hemorrhage . Petechiae occur commonly on the palate

Important: Tooth extractions are contraindicated due to the tendency for excessive
bleedin s.

***

This is false; acute leukemias have a rapid onst and progression.

-{cute leukemia is characterized by malignant proliferation olwhite blood cell precursors I hle.\ts) in bone marrow and lymph tissue and their accumulation in peripheral blood, bone rnanow, and body tissues. Leukemic cells inhjbit normal bone marrow production rrf en throcytes. platelets, and immune function.
Orher important features ofacute leukemia: . -A.brupt onset (fe11,noltlsl with sudden high fever, weakness, malaise, severe anemia, and lymphadenopathy; bone and joint pain common in children. -ueneralized . Principal organ involved: bone marrow (along witlt the spleen and liver) . Petechiae and ecchymoses in skin and mucous membranes, hemorrhage from various sites: bacterial infections common. . Laboratory findings: leukocytosis 30,000-1000,000 per cu.mm. with immature forms r nn eloblasts and 1.,-mphoblasts) pred,ominating. . ln 7-i9i ofthe cases olacute lymphocytic leukemia, the lymphocytes are neither B nor T-cells and are called "null cells. " . L ntreated patients die within six months; with intensive therapy (chemotherap,-, rutliation, and bone marrow transplanls) remissions lasting up to five years may be obtained; death is usually due to a hemorrhage (broin) or a superimposed bacterial

infection.

. Thromboc)'topenic purpura

. Agranuloc).tosis

. Sickle-cell

anemia

. Peutz-Jeghers syndrome

18 Copynghr @ 2011,2012, Dental Decks

The term used to describe a leukemia where leukmic cells rppear in the blood but there is no significant lncrease in the number of white blood cells is callod: .,

. Aleukemic leukemia

. Subleukemic leukemia

Stem cell leukemia

't9
CopFight O 20ll-2012 - Dental Dects

Agranulocytosis is an abnormal condition of the blood, characterized by a severe reduction in the number ofgranulocytes (particL arly neutroprilr./. Note: It may also be caused by the antithyroid drugs fi.e., propylthiouracil, methimazole, qnd carbimazole).

Clinical features:

. Sudden onset of high feveq chills, jaundice, weakness . Oral infection with rapid periodontal destruction . Oral ulcers and gingival bleeding

and sore throat

The most characteristic feature of this condition is the presence of infection, particularly in the oral cavity. The signs and symptoms develop very rapidly, usually within a fev days. and death may occur soon afterward.
The oral lesions are an important phase ofthe clinical aspects ofagranulocytosis. They appear as necrotizing ulcerations of the oral mucosa, particularly the gingiva and palate. These lesions appear as ragged necrotic ulcers covered by a gray membrane. One imporranr aspect is that there is littl or no apparent inflammatory cell infiltration around the

lesions. Histologically, this is pathognomonic of agranulocytosis.

\ote: Cl clic nutropenia

is an unusual form ofagranulocytosis. These patients typically erhibit severe gingivitis. The severe ulcerations usually seen in agranulocytosis usually dLi not occlrr. See picture #19 in booklt

Other leukemia terms to know:

. ".A.leukemic" leukemia is a term used to describe a lorm of leukemia in which there are leukemic cells present in the bone maffow, but the circulating white blood cells are
nither immature nor increased in number. . Stem cell leukemia is a form of leukemia that is characterized by abnormal cells that are poorly differentiated but are considered to be precursors of lymphoblasts, myeloblasts. or monoblasts. Note: these cells are too immature to classify.

. A "leukemoid" reaction is a term used to describe a marked increase in the number of circulating granulocytes. This condition is seen in a variety of disorders inch.rding cluonic infections and neoolasms.

. Denuded gingiva . Glossitis . Edematous buccal mucosa .


Severe gingivitis

20
Cop),righr O 201 l-2012 - Dental Dcks

In your ofllce, you see a 6 month old child whose lirst teeth are eruptlng and whose mother iJ concrned about the color. The mandibular incboB do show a brownlsh-blue hue, You are golng to ask the mother about which
of the following conditions during her pregnancy:

. Sickle-cell

anemia

. Erythroblastosis fetalis
. Patent ductus arteriosus

. Low-weight preterm birth

21

CopFiglr O 20ll-2012 - Dental Decks

Pernicious anemia is a relatively common, chronic, progressive, megaloblastic anemia. It is caused by the lack of secretion of the intrinsic factor in normal gastric juice. This lactor is necessary lor adequate absorption of vitamin 81.,, which is necessary for the maturation of erythrocytes. As a result, they produce fewer erythrocytes than normal. The onset of pemicious anemia usually is insidious and vague. As the condition progresses, there will be a sore, painful ton g.ue (atrophic glossilr.r/, angular cheilitis, a tingling numbness of the extremities. difficulty swallowing (d,,-sphagia), painful swallowing (od,vnophagia). See picture #20 in booklet

A Schilling 24-hour urine test is done to evaluate whether vitamin B 1.' is being absorbed by the body and is most commonly used to evaluate patients for pemicious anemia. Remember: Thalassemia major and minor are hernolytic anmias that result from a genetic defect. Both are characterized bv a low level of ervthrocvtes and abnomal hemoglobin.

Oral manifestat ions of thalassemia: . Oral mucosa may exhibit the characteristic anemic pallor . Flaring of the rnaxillary anterior teeth with malocclusion

The fetus'blood is Rh-positive because the father passed along an Rh-positive trait. rvhich is a dominant trait. The mother responds to the incompatible blood by producing antibodies against it. Thesc antibodies cross thc placenta into the fetus'circulation. whcrc thcy af

tach to and destroy thc fctus' red blood cells, leading to anenria
er.r

throblastosis fetalis.

this is called

\ote: It can also rcsult from blood typc

incompatibilities. For cxample, the mothcr may

har e ty'pc O blood and thc fetus has type A or B blood.

The most common form oferythroblastosis fctalis is called is ABO incompatibility, which can ran in its severity. The less common form is called Rh incompatibility, which more ottcn causes a very scvere anemia in the baby. The severity olthis condition can vary widcly. In some instanccs, the baby has no symptoms of the disease. In othcr cases, it can lcad to death of thc baby beforc or shortly after birth. lt can be treatcd in utero by intrauterine transt usion.

\\'hcn thc child is bom, signs may include an enlarged liver or spleen, generalizcd cdcma, laundice. and anemia. After birth, depending on the severity, a transfusion usually nccds to be performed. The most severc form of this disease, Rh incompatibility, can bc prcvcntcd if thc mothcr takes a medicinc called Rhogam at certain times during and aftcr prcgnancy.

Oral manifestations of crythroblastosis fetalis:


. Teeth appear to havc a grccn, blue or brown hue due to thc deposition ofblood pigment in the enamel and dentin. . Enamel hypoplasia may occur. If it does, it affects thc incisal edges of the anterior teeth and the middle portion ofthe deciduous cuspid and the first molar crown.

They have a rapid onset and progressron They have a shorter, more devastating clinical course than the acute leukemias They are characterized by proliferations of lymphoid or hematopoietic cells that are
more mature than those ofthe acute leukemias

They constitute

7 5o/o

of all leukemias

22
Copyright O 201l-2012 - Dental Deck

. Acute lymphocytic leukemia (ALL)

. Acute myeloid leukemia (AML)

. Chronic lymphocytic letkemia (CLL)


. Acute monoblastic

23
Coplright
@

20ll-2012,

Dnral Decks

Important: Chronic leukemias have a slower onset and progression. They also have a longer, less devastating clinical course than the acute leukemias, and they constitute about 507o of all leukemias.
Other important clinical features of chronic leukemias: .Insidious onset witl't weakness and weight loss: disease may be detected during exanination for some other condition (e.g., anemia, unexplained hemorrhages, or rccw're nt intrqctab le inlbctio t1 ) . Organ involvement similar to acute type: massive splenomegaly is characteristic of chronic myelogenous leukemia; lymph node enlargement is main pathologic finding in lymphocytic tyPe lymph. Petechiae and ecchymoses' recurrent hemorhages, bactedal infections anemia hemolytic by autoimmune complicated ocytic anemia may be . Laboratory findings: leukocylosis above 100,000 per cu. mm. with matur forms (gronulocltes ancl lymphocytes) predominating; Philadlphia chromosome and low levels of leukocyte alkaline phosphatase are corunon in chronic myeloid lukemia (c.\'tL) . \'ledian survival time for patients with chronic myelogenous leukemia /CMI/ is lour (CIl) l ears rvith death <lue to hemorrhage or infection; chronic lymphocytic leukemia treatment' years without even patients may survive older runs a variable course;

The peak age for ALL is around four years old, and it is the form of actte leukemia that is the most responsive to therapy. Cunent therapies for ALL include chemotherapy with lbllo\1.up radiation, and possible bone-marrow transplant after particularly high dose chemotherapy treatment or in cases ofrecurrence or ilnon-responsive to other treatments.

***

Acute myeloid leukemia (AML) and chronic lymphocytic leukemia (CLL) arc the

most common types in adults. See picture #22 in booklet

Allhough the exact cause ofmost leukemias remains unklown, increasing evidence suggests a combination ofcontributing factors. These factors include: familial tendency, congenital disorders (Dov,n syndrome, or the presence of Philadelphia chromosome chronic myeloid leukemia), viruses (e.g., HTLVI, herpes-like viral particle,s lnve been c ulared.fion potients ancl leukemic patients have high ontibod), titer to the Epstein-Barr lirr-st. ionizing radiation and the exposure to the chemical benzene and cytotoxins such
as alk) lating agents.

Important: Oral lesions are most likely to be observed in myelogenous leukemia. These oral lesions may be the initial manifestation ofthe disease. The oral lesions include gingivitis, gingival hemorrhage, generalized gingival hyperplasia, petechiae, ecchymoses. and ulcerations. See picture #21 in booklet

Bld Dis

You have a new patient in your dental oflice who has just moved from Denver. He srys his doctor told him that he has some disease caused by living at a high altitude. When conducting an intraoral xam, you lind that his tongue is a deep purple and his gingiva bleed easily. What disease is a likely cause ofthese findings?

. Polycythemia Vera . Hemophilia B . Thallesemia Major

. Porphyria

24
Coplrighl

e 20ll':012 '

Denral Decks

ORAL PATHOLOGY

Bld Dis

The translocation from chromosome 22 to chromosome 9 is a finding of which leukemia?

Acute lymphocyttc Ierkemia (ALL)

. Chronic myeloid leukemia (CMZJ

. Acute myeloid

leukerma (AML)

. Chronic lymphocytic leukemia (CLL)

25 Copright
ae

20ll l0ll

DenklDeck!

Polycythemia is the condition oftoo many red blood cells in the circulation. The blood can be too thick to pass easily through the small blood vessels ofthe body. This in tum leads to clot formation and blockage ofthe small vessels which can lead to a stroke
There are two types ofpolycythemia: I . Primary polycythemia (also called polycythemia vera or e4lhemia) occurs when excess erythrocytes are produced as a result ofturnorous abnormalities. This occurs in the tissues that produce blood cells. Usually accompanied by leukocytosis. Splenomegaly, as a result ofvascular congestion, is seen in 75% ofpatients. 2. Secondary polycythemia is an increase in the total number of erytluocytes due to another condition. For example. chronic tissue hypoxia of advanced pulmonary disease, high altitude fo.r ler's disease) or the secretion oferythropoietins by certain tumors.

Oral manifestations of polycythemia:

. Oral
red.

mucous membranes (especialb tlrc gingiva and tongue) appear deep purplish-

. The gingiva are very swollen and bleed very easily. . Submucosal petechiae (purplish spots), ecchymoses (same and hematomas are common.

as

petechiae, but bigger)

is a rare syndrome of paroxysmal vasodilation with buming oain. incrcascd skin tcmpcrature, and redncss ofthc feet and, less often, thc hands.

\ote: Erl-thromelalgia

\lmost 90% of patients with chronic myeloid (mteloctttic, n|'elogenous, granuloc,-tic) leukemia have the philadelphia chromosome, an abnormality in which the long am of chromosome 22 is translocated. usually to chromosome 9. Radiation and carcinogenic chemicals may induce this chromosomal abnormaliry
characterized by the abnormal overgrowth ol granulocytic precursors rn^elctblqsts and promvelocvtes) in bone marrow, peripheral blood and body tissues. C\tL is most common in young and middle-aged adults and is slightly more common in men than in women: it is rare in children.
The npical symptoms of

C\IL is

CML include: . spongy bleeding gums . t'atigue . fer er


. \\ eight loss

. moderate splenomegaly . joint and bone pain . repeated infections

\ote: Acute myeloid leukemia (AML) rs a malignant disease of the bone marrow in s hich hematopoietic precursors are arrested in an early stage of development. AML is distinguished ftom other related blood disorders by the presence of greater than 30% blasts in the blood and/or bone rnarrow. These blasts /m|eloblasts) contain Luer rods in therr cvtoDlasm.

Which of the following is { troublesome librobLstic neoplssm that is locally sggrer$ive rnd infiltrrtive?

. Peripheral fibroma

. Traumatic neuroma
. Nodular fasciitis . Fibromatosis

26
Copynghr Q 20ll-2012 - Dntal

Deck

While in the OR on a general surgery rotation, a 3 month old is brought in with alarge (20cm) flaid-lilled mass on her neck. The diagnosis is r cystic hygroma (hygroma coli).This lesion is under whlch umbrella oflesions, which also ' contdns enlarged tissue on the postcrior rnd latral border of the tongoe?

. Angiomas . Lymphangiomas . Schwanaomas . Fibrosarcomas

27
Coplright O 201l-2012 - Denlal Decks

Tumor

Etiolog)

Clinical Charictenstics
Most common sit ovI

Trcatmenl and Prognosis

mentll

foramen m edentulous moufts;


nodule or swelling, wbich may be painful to digilal pressure Encapsulated mass lhal prese.ts
a5 an

[xcision wjth snall proximal podion ofinvohed neNei

Consenati!e e{cisioni

a*mptomatic lump. Tbe

longue ls the most common location.

siy it

is denved from

asympromaric ndule, occurs on tongue. buccal mucosa and


1.

Solitlry nelrollbromr

L Soli(ary: surSical excision 2. Neurofibromatosis: removal is impractical. wrtch lbr hiSh rate ofmalignant

2.

iltultiple

lesions as pad

oflhe
Su.gical excivont rccuncncc

sFdrome neurofi bromatosis


P.esms as {irrn mass; xhibits rapid gowlh; pain and

Tbe mandible and condguous soi lissues are most frqunlly

Aggressive sursical approach is recommended. Rccurrence's

involved intrno.ally. LocaLly


aggressive and infi ltrarive.

\ traumatic ncuroma is a lesion causcd by lrauma to the periphcral ncnc. In thc oral catity, thc injury rnay be nr rhr rirmr olrrauma fiom u surgical procedure such as a toolh cxlraclion, from a local ancslhctic injcclion or fionl an .r..ideni. It is usually a vcry small nodulc 1/.\'r /,{utt 0.5 .r in didnrcter).In thc oral cavity it is most commonly sccn .! rIc mentat foramen. Il is firm, movable ard wcll cncapsulatcd. ll is painfut when pslpaled. Prcssurc applicd to rhr naurona clicits a resporsc oftcn dcscribcd as an "electric shock".
Hi!tologic features: . \bundant ncnc lissuc a'rd collagcnous fibrous tissuc in haphazard arangcnrcri . Chronic lillammalory ccll infilirate may be prcscnt . Sch\ ann cclls rri1l bc prcscnt

mphangiomas are benign bamartomas of lymphatic channels that develop early in life $ ith no sex predilection. They may occur on the skin or mucous membrane. ln addition to the tongue, they occur commonly on the lips and labial mucosa.
Ly

Clinical features: . Raised, diffuse. bubbly nodules or vesicles . Range in color from clear to pink. dark red, brown or black

'

.As)'mptomatic

. Soft. fhrctuant . \ aries in size . Usuall.v painless


Histologic features: four types of lymphangiomas: . Ll mphangioma simplex (capillory lymphangiono) lvmphatics
. Calernous lymphangioma - comprised ofdilated lymphatic vessels with surrounding adventitia . cl stic lymphangiom? (cyslic hlgroma or hygroma col, - consisting ofhuge, macroscopic lymphatic spaces with surounding fibrovascular tissues and smooth muscle .Benign lymphangioendothelioma facquired progressive lltnphangiont) lymphatic channels appear to be dissecting through dense collagenous bundles

composed of small, thin-walled

Important: Lymphangiomas do not undergo malignant change. Some lymphangiomas,

es-

pecially congenital types, regress spontaneously during childhood. Aspiration is manda-

tory before surgical excision ola lyn.rphangioma to prevent complications associated with
the similar-appearing hemangioma

. Lipoma
. Pyogenic granuloma

. Epulis granulomatosum
. Peripheral fibroma

28
Coplright O 20ll-201: - Dental Decks

A patint presents to your clinic with multiple exophytic masses covering the buccal mucosa, tongue rnd lips. A biopsy reveals that these are mucosrl neuromas. The most importrnt rerson this patient should be referred to a physician is bectuse ofthe risk of related:

. Squamous cell carcinoma ofthe tongue


. Pituitary hyperplasia . Medullary carcinoma ofthe thyroid

. Sipple's syndrome

29 Coplaiglit O 201l-?012, Denral Deckr

slate during pregnancy

l0

days

of

Remember: L Thc peripheral fibroma is a rcactive hyperplaslic mass that occurs in the Singiva and may be dcrivcd from connective tissue ofthc submucosa or the PDL. It presents as well-demarcatcd lbcal mass \\ ith eithcr a sessile or pedunculated base. Ii is similar in qolor to the suffounding connective tissue. It may be ulccratcd. The treatment for a pcriphcral fibroma is local excision. Recurrence is rare'

\ote:

Other variant forms ofthe peripheral fibroma includc: . The peripheral odontogenic fibroma: which is gingival mass composed ofa well-vascularized, flbrous connective tissuc. Thc distinguishing featlre ofthis variant is thc presencc ofstrands ofondorogenic epithelium, oftcn abundant, throughout the connective tissue. It is usrlally non-ulcerated. . Thc peripher.l ossifying fibroma: is a gingival mass in which calcified islands. presumed to be bonc. are seen. The bone is found qithin a non-encapsulated prolifcration of plump benign fibroblasts. The surface is often ulccrated. See picture #85 in booklet

Focal fibrous hyperplasia is hlperplasia oforal mucosa. It is also callcd traumatic fibroma, irrilarion fibroma. and hyperplastic scar. It is a rcactivc lcsion caused osually by chronic trauma to oral mucous membranes. The giant cell libroma is a focal fibrous hlT'erplasia in which connective tissue cells. many ofwhich are multinucleated, assume a stcllatc shapc.

:.

The multipfe endocrin neoplasia syndromes (olso called MEN Syndrome./ have been classified into three distinct syndromes, each is inherited as an autosomal dominant trait:

consists of tumors or hyperplasia of the pituitary parathyroids, adrenal cortex, and ofthe pancreatic islets. . \len Il (also c'alled Sipple's Syndrome ard sabn?e is characterized by parathyroid hyperplasia or adenoma, but no tumors to the pancreas. However, in addition, these patients have pheochromocltomas ofthe adrenal medulla and medullary carcinoma of the thyroid gland. . \Ien III (elso kno$n os sub\tpe IIB): rs characterrzed by mucocutaneous neuromas, pheochron'rocytomas ofthe adrenal medulla and medullary carcinoma ofthe thyroid gland.

. \ten I:

lr:

Important: MEN I and II are related to MEN lll in that patients with types I and II syndromes have neoplasms ofvarious endocrine organs, but do not have the oral manifestarions ofmucosal neuromas. These oral lesions are most common on the lips, tongue, and buccal mucosa.

\ote:

The most important aspect of this syndrome is the medullary carcinoma of the thvroid because of its ability to metastasize and cause death. Therefore, the detection of the rnucosal neuromas may alert the clinician for early diagnosis and treatment.

. Traumatic neuroma . Neurilemmoma fsc hwannoma) . Neurofibroma . Nodular fasciitis

. Fibromatosis

30

Coplrigh O 20ll-2012

- Dental Decks

. Neurolemoma . Neurofibroma . Neuroma . Fibroma

31

CopFight O 201l-2012 - Dental Deck

Neurofibromas may appear as solitary Iesions or as muitiple lesions as part ofthc syndrome neurofibromatosis 6,()// ne. klinghdusen:\ disease of skin). lr rs a benign ncoplasm, howcver the etiology of
solitary neurofibromas is unknown. Most researchers believe the cell oforigin is the Schu'ann cell; others believe dre pe neLlral fibroblasr is rcsponsiblc- For the solitary neurofibroma thc tongue. buccal mucosa, and vestibule are the oral regions most commonly affected. Tumor
Traumatic

Etiology pcnpheralnene

Clinical Chrracteristics
Most common sile over mental foramen in edentulous mouths; nodule or swelling, which may be painful lo digital pressure

Treatmntand Prognosis
Excision wilh small proxirnal ponion of involved nen,e; recurTence uncommon Conservalive excision;

Neurilemmoma

BcniSn neoplasm Schwann cells

of

Encapsulated mass that Fesents


as an aslmptomatic lump. The tongue is the mosl commot

localion.

Neurofibrona

Somc invcstigalors say il is derived liom the Schwann cell; olhers say it is derived l-.om

Two formsl 1. Solltary neurofibroma asymplomaiic nodule, occurs on tongue, bsccal mtrcosa and vestibule
2. Multiple lesions as pan ofthe s)mdrome Deurcfi brohalosis P.esents as firm mass; exhibits r8pid growth; pain and lendemess common

L Solitary: surgical excision


2. Neurofibromatosisl removal is impractical. wrlch for high rate ofmalignant

transformalion

fibroblasts

\odular fasciilis

Reacti!e lesion; lpserdosarconatous proliteration of fibroblasts Benign fibrous proliferation of fibroblasts

Surgical excision; recurrence

The mandible and conliguous sofi tissues are most liequenlly involved intraorally. Locally
aggressive and inliltrative.

Aggressive surgical approach is


recommended. Recurrence is

\bn

Reckfinghausen's disease (neut'oJibromatosis) is an autosomal dominant disease associated rvith loss ofa tumor suppressor gene (NF/ or NF2l. lt is characterized by multiple neurofibromas of the oral cavity and on the skin, and pigmentations of the skin r Ccrfi-au-luit .rpor.s/, iris freckling (Lisch spots), and axillary freckling Crou'e )s sign.). \ote: The presence ofsix or more caf6 au lait macules greater than 1.5 cm in diameter is generally regarded as being indicative of this disease until proven otherwise

Radiographic features: . \\ ell-demarcated unilocular or multilocular radiolucency . Possible root disturbance . Possible jarv enlargement Treatment:

. Surgical excision but may be impractical due to the number of lesions . Best 1eft untreated because multiple recuffences may be associated with malignant
transformation to neurogenic sarcoma

\ote:

The importance of the lesions is the high risk (5%' to

l5%o.1

of mtlignanl trans-

formation
Remember: The single neurofibroma presents at any age, it comnronly appears as a sessile, firm, pink nodule that commonly occurs on the tongue, buccal mucosa and lestibule. It is rerloved by surgical excision and rarely recur.

Con Tis Les

A 5S-year-old patient coms into your oflice for routine dental work. You se that he has a tooth fracture (due to decay) of tooth #31, A smooth, firm, asymptomatic lesion is noted on the lateral border ofthe tongue adjacent to the sharp enarnel oftooth #31. The patient states that the lesion has been there for years and is annoying because sometimes he will bite it accidenta[y. Name this most frequently encountered intraoral benign neoplasm of connctive tissue origin,

. A leiomyosarcoma
. A traumatic fibroma
. A leiomyorna

. A rhabdomyoma
32 Copynghr C 2011-l0ll - Dental Decks

ORAL PATHOLOGY

Con Tis Les

Scleroderma is a systemic disease that alfects many organ systems. The symptoms result from in{lammation and progressive tissue fibrosis and occlusion ofthe microvasculature by excessive production and deposition of:

. Types II and IV collagens

Types I and

lll

collagens

. Elastin

and reticulin

. All of the above

Traumatic fibroma, also know as initation fibroma, focal fibrous hyperplasia, and h)?erplastic scar, is
a reactive lesion caused usually by chronic trauma to oral mucorls mcmbranes. Ovcrcxubcrant fibrous connective tissue repair results in a clinically evident submucosal mass.

Reaclive lesion caused tmuma to oral mucous

Painless, broad-basd swelling lhar is lighter rhan surrounding ii$ue; fiquenlly fotmd on buccal mucosa, laleral border

oftongxe, snd tower lip


Orat liomyomas arc rare; present as slow-growlng, asymptomatic subrnucosal oasses, usualiy in the longue,
bard palate, or bucal mucosa

Wid surgical exc's'oni rc'

Oral leionyosarcomas have been reportd in all ag groups and most intraoral regions.
Rare bul have a prdilection for the soli tissues olthe had a..d neck Aoor of the nouth, solt palate. totryue, and buccal n!.a.ra). Presents as an asymptomatic. well-dfi ned Rare in head and neck; most commonly affecled oml siles ar th longu and soft palf,te. Presents as a mpidly $owing mass, which, ifrhere isjaw involvemnt, may caus pa'n.

Wide surgical excisioni

Excisioni

Combinition ofsurgery. radirtion, ard chemothempy.


melaslasis not common

*"* Otlrcr lcvels of nracromolecules found in the conncctivc tissue fe.g., glytosantinoglycans, :a,1u'(tn. lihronecti ) are also incrcascd. scleroderma is an uncommon. autoimmune disease. It all'ects the connective tissues $hich sur:rrund thcjoinls, blood vcsscls and intcmal organs bcneath the af}'ected area ofskin. Womcn arc afi.tcd thrcc to ibur tinres morc oftcn than men. The diseasc usually starts between thc ages of25 llnJ :0 It onl-'- oqcasionally bcgins in children or in the clderly. Although there is no curc. propcr lrearnlent and care can makc it possible for people rvith sclerodenna to lead full, productive livcs. \ote: \1a) occur concomitantly with other autoimmune diseases. Such as lupus erythemalosus, rheunaroid arthritis. dermatomyositis, and Sjogren's sundrome. \ote: Thesc changcs in thc conn.-iti\c trssue all'ect the fingers, trunk, face (producing u "purse-string" moil/i], and the nlore rr(]\]]'nal parts of the cxtrenlitieS.
Oral radiographs ol'a patient with scleroderma would show an abnormal rvidening of the peri..Jrrnral ligament. This space is created by a thickening ofthe periodontal-membranc as a rcsult of :n increase in size and number ofcollagen fibers. The enlarged space is almost uniform io width, .rlrounds the cntirc root of the toolh and makes the tooth appear as if it is being cxlrudcd rapidly :ilrir its socket. See picture #31 in booklet Note: Other oral radiographic l'eatures may include ]rlar.ral resorption ofthe angle ofthe ramus ofthe mandible or complete resorption 01-thc condylcs :nd or coronoid process ofthe mandible. Remember: The abnormal widening ofthc periodontal:rembrane spacc is also a radiographic finding in osteosarcomas.

Thcrc arc tlvo major types ofscleroderma: I Localized scleroderma; In this condition changes only occur in isolatcd areas of the skin and the tissues beneath it. [t is rclativcly mild and docs not aft'cct intemal organs. Cutaneous changes includc induration and rigidity. atrophy, and telangiectasias.

L Systemic scleroderma: With

this condition changes rnay occur in thc skin and also in a number of intemal organs. These might include blood vessels, joints, the digcstivc syst"em (esophagus, stomach and borlel/, and occasionally the lungs, hean, kidneys and muscles. Changes in the connective tissue may affect the function ofany ofthese organs.

ORALPATHOLOGY

Con Tis Les

A newborn baby girl was delivered via caesarian section due airway patency concerns. During ultrasound, there was th discovery of a tumor ofthe oral cavity. Upon delivery, the pink, comprssible tumor of the anterior maxilla was deemed to be a congenital epulis ofthe newborn. This lesion is composed ofcells that are identical to those ofthe:

. A traumatlc neuroma . A schwannoma . A granular cell myoblastoma . A lipoma

34
CopyriShr

201

l-201

- Dental Decks

ORAL PATHOLOGY

lnfl Jaw Les

An emergency patient walks into your o{fice with swelling ofthe left submandibular space. He says his lower left molar recently 'broke down" and has been very painful especially when something cold hits it or whcn he chews down on it. What is the most likely etiology of this swelling.

. Orthodontics
. Trauma

. Infection of the pulp ofthe tooth


. Periodontal disease

Copyrighr C

20ll'l0ll

- Dental Decks

The congenital epufis of the newborn (also called congenital gingival granular cell tumor).lt rsually appears on the gingiva (usually anteriof ofnewborns. It presents as a noninflamed, pedunculated or broad-based mass. The maxillary gingiva is more often involved than the mandibular gingiva, and ferrales are affected more than males. The treatment is surgical excision with little possibility ofrecurence.
See

pictur #24 in booklet

The granular cell myoblastoma is a rare neoplasm of unknown etiology. Most researchers believe its origin is from the Schrvrum cell. It presents as an uninflamed. asymptomatic mass less than 2 cm in diameter. The most common location in the head and neck region is the tongue. It may affect any age group and females seem to be affected more

than men. See picture #25 in booklet

Important: Both of these lesions are identical histologically. They both contain granular cells, however, the congenital epulis of the newborn does not exhibit overlaying pseudoepitheliomatous hyperplasia. The pseudoepitheliomatous hyperplasia of the or erlying epithelium is frequently seen in the granular cell myoblastoma.

This infection follows the carious involvement ofthe tooth. The cellular debris and/or rnfection rvhich caused the tooth pulp to die, slowly filters out of the tip of the root and produces an inflanmatory reaction around the root tip

***

\ote: ,\ periapical abscess can

also occur after traumatic injury to a tooth, which results in necroiis of the pulp, and in cases ol irritation of the periapical tissues, either by mechanical manipulation or by the application of chemicals in endodontic procedures'

Clinical features: . If acute, presents

as an

abscess:

See

picture #26 in booklet

Tooth is extremely painful to percussion - May feel slightly extruded from its socket - Tooth will exhibit mobilitY If chronic, presents as a granuloma or cyst. There are usually no clinical features of

svmptoms

Radiographic features: . If acut, only a slight thickening ofthe periodontal membrane is noticeable

.If

chronic (granuloma or q,st), there will usually be the involved tooth. See picture #27 in booklet

radiolucent area at the apex

of

Treatment: Establish drainage either by opening the pulp chamber or extracting the
tooth.

Note: If

a periapical abscess is

not treated, it can lead to serious complications such as

osteomvelitis. cellulitis and bacteremia.

A [ealthy patient comes into your ollice for an inltial exam. On the full mouth series of radiographs you see a radiopaque lesion perlaplcal to tooth #19. Tooth #19 has a deep amalgam restoration with rcurretrt decay underneath. You csn Nee the entire outline ofthe mesial root oftooth #19 - the lesion seems to stem frorn the tooth. What is the most llkely diagnosis of the lesion?
N.

. Focal sclerosing osteomyelitis

. Cementoblastoma . Cementoma
. Fibrocementoma

CopyriShr O 201 1,2012 - Dental Dcks

may be psrt ofthe etiologic picture, although been the most frquently cited.

. Bacteroides and campylobacter

. Clostridia

and corynebacterium

Staphylococci ard streptococci


and lactobacilli

. Enterococci

37
Copyrighr O 201 l-2012 - Dental Decks

Focal sclerosing osteomyelitis is a relatively common phenomenon that is believed to represent a focal bony reaction to a low-gnde inflammatory stimulus. [t is usually seen at the apex of a tooth in rvhich there has been a long standing pulpitis. Note: Synonyms for focal sclerosing osteomyelitis include bony scar, condensing osteitis, and sclerotic bone. The term focal periapical osteopetrosis has also been used to descdbe the lesions associated with normal
caries-free teeth. Focal sclerosing osteomyelitis may be found at any age but is typically discovered in young adults. Patients are usually asymptomatic, and most lesions are discovered on routine radiographic examination. A majority are found at the apices ofmandibular first molars. The periapical x-ray demonstrates the pathognomonic, well-circumscribed radiopaque mass of sclerotic bone surrounding and extending below the apex ofone or both roots. The entire root outline is almost always visible, an important litature in distinguishing it from the benign cementoblastoma, which radiographically, it may resemble. The tooth with this lesion may be treated or exftacted, since the pulp is infected and the infection has spread past the immediate penapical area. The sclerosing bone constituting the osteomyelitis is not attached to the tooth. and remains after the tooth is treated or removed. DifTuse sclerosing osteomyelitis represents an inflammatory reaction in the mandible or maxilla. believed to be in response to a microorganism oflow virulence. lmportant in the etiology and progression ofdiffuse sclerosing osteomyelitis is chronic periodontal disease, which appears to provide a portal ofentry lorbacte a. The condition tends to occur most ftequently in middle-aged black females. The disease is typified by a protracted chronic course with acute exacerbations of pain, swelling, and occasional drainage. Radiographically, this process is dld'ure. 9*pically affecting a large part ofthejaw The lesion is ill-defined. Treatment consists t idetermining and addressing the cause. Antibiotics are the mainstay of trcatment. Low-dose .onicosteroids have also been used with some success. Hyperbaric oxygen therapy may prove :.r be a r aluable adjunct.

Acutc inflammation ofthc bonc and bone manou'ofthe mandible and maxilla rcsults most fiequently from rtension of a periapical abscess. The second most common cause of acute osteomyelitis is phlsical injury as secn u,ith fractlte or surgery. Most cases of acute osteomyelitis arc infcctious. Staphr-lococci and streptococci are the most frequcntly cited.

Pain is the primary feature of this inflammatory process. Fever, painful lymphadenopathy, leukocytosis. and other signs and symptoms ofacutc infcction arc also commonly found. Paresthesia ofthe lotler lip is occasionally seen with mandibular involvcmcnt. Important: Unless the inflammatow process has bccn prcscnt for more than I wcck, radiographic cvidcnce ofacutc ostcomyelitis is usualll_ not present.

\\'ith timc. diffuse radiolucent changes bcgin to appcar. Treatment includes antibiotics and drainage.
Chronic osteomyelitis (chronic osteitis) fiay bc one ofthc scquclac ofacule osteomychlis /ei er ultI eatetl o, in.tdequateb lrealed), or rt may represent a long-tenr, lorv-grade inflammatory reaction that nevcr $ ent rhrough a significant or clinically noticeable acute phase. Most investigators believe that bacleria /e.g., staph ococci, \treptococci, bacleroiles. acli on\rer'/ are responsible for thc vast majoriI ol chronic osteornyelitis cases. The mandible, especially the molar area, is much more lrequently atl'ected rhan js thc rnaxilla. Pain is usually present, s\relling ofthe jaw is a comrnonly encountered sign; loose teth and sinus tracts are less frequently seen. Anesthesia is very uncommon. Radiographically, chronic osteomyclitis appears primarily as a radiolucent lesion that may show focal zones ofopacificarion. The luccnt pattem is often described as "moth-eaten" because ofits mottled radiographic appearance. Treatment includes antibiotics and scqucstrectomy.

Gar16's osteomyelitis or chronic osteomyelitis with proliferative periosteitis, is cssentially a subtype of chronic osteomyclitis in which there is, additionally. a prominent periostcal inflammatory reaction. It mosi olien results from a periapical abscess of a mandibular molar in a child. The child characterisricali.v presents with an asymptomatic bony hard swelling with normal-appcaring overlying skin and mucosa. Radiographicallli thc lesion appears centrally as a mottled, predominantly lucent lesion in a pattem consistcnt uith chronic osteomyelitis. The featrre that provides thc distinctive difference is the periosteal reaction. This appears as an expanded cortex, oftcn uith concent c or parallcl opaque layels. Trabeculae perpendicular to the "onion skin" layers may also be present. Treatment includes tooth extraction and antibiotics.

. First to third week

. Fourth to sixth week . Eighth to tenth week


. twelfth to fourteenth week

38
Coplright O 20ll-2012 - Dental Decks

. Relatively normal torso

and long arms and legs

Short torso and long arms and legs

. Long torso and long arms with short legs

. Relatively normal torso and short

axms ajld legs

39
Coplrighr O 201l-2012 - Denral Decks

Cleft palate occurs in the eighth to tenth week of embryonic life. Isolated clefts olthe palate are more common in females. It is characterized by a fissure in the midline ofthe palate, resulting from the failure ofthe two sides to fuse during embryonic development. The most severe handicap imposed by cleft palate is an impaired mechanism preventing normal speech and swallowing. Note: It effects approximately I in 2000 births.

Cleft lip results when the medial nasal procss fails to fuse with the lateral portions of the maxillary process olthe first branchial arch. Fusion normally occurs during the sixth and sevnth weeks of embryonic development. The maxillary lip is most commonly affected. It may be bilateral (20a/o) or lunilateral (80%). Clefts ofthe lip are more frequent in males. Lip clefi involvement is more frequent on the left sid than the right. Note: It effects approximately I in 1000 births, but varies with race.

\ot<

L Speech problems associated with both ofthe above are usually the result of the inability ofthe soft palate to close airflow into the nasal area. 2. It is not unusual for teeth especially the lateral incisor to be missing in the

cleft area. 3. More than 250 syndromes have been identified that may be associated with cleft lip and palate, or cleft palate alone. 4. Lip pits are rare anomalies that can occur in the upper lip, lower lip, or the oral commissure. Although lip pits may be seen near the oral commrssure or midline upper lip, most occur on the Iower lip and are associated with van der \lbude syndrome. This syndrome is an autosomal dominant condition with 80% to 90% penetrance consisting of lower lip pits and cleft lip and/or cleft
Dalate.

.\chondroplasia is the most common type of dwarfism. The upper an.ns and thighs are more shortened than the forearms and lower legs. Generally, the head is large, the forehead is prorrinent, the nose has a saddle-like appearance and the mandible exhibits progn

al

ism.

Potential problems in children with achondroplasia include overcrowding of the teeth, speech problems (articulatio ), and frequent ear infectior:Ls (otitis media). Dental malocclusion is treated \\. ith orthodontics. All children with achondroplasia should be evaluared b\ a speech therapist by two years of age because ofpossible problerrs with the de\eiopment ofclear speech. Articulation problems may be caused by onhodontic probIems. Due to the abnormal shape of the eustachian tube in an individual with achondroplasia, they are very prone to ear infections (otitis media).

\ote: The teeth are of nomal size but there is limited space within the maxillary and mandibular arches for them to erupt into which causes overcrowding and subsequent malo!'cluS ron.

.oI
. Marfar's syndrome

. Ehlers Danlos . Cystic Fibrosis

40
Coplrighl O 2011,2012 - Ltental Decks

Hypophosphrt.sia is a genetic metabolic disorder of bone mineralizstion caused by a deficiency in:

. Acid phosphatase

. Vitamin K . Alkaline phosphatase

. Phosphorus

41

Copyrighl c' 2011,2012 - DentatDcks

The characteristic features olosteogenesis imperfecta (OI) vary greatly from person to person and not all characteristics are evident in each case. The chief clinical characteristic ofosteogenesis imperfecta is the extreme fragility and porosity ofthe bones, with a proneness to fracture. Other features include: pale blue sclera, deafness due to otosclerosis, abnormal teeth, loosejoints and low muscle tone. a triangular face, and a tendency toward spinal curyature.

Clinically the teeth have: . Crowns that are bulbous: with a cervical constriction . Pulps that are obliterated, either partially or completely . Roots that are narrower and shorter

***

The deciduous teeth are more sverlv affected that the permanent dentition

T! pes of ostogenesis imperfecta: . Type l: most common and mildest fonn . Type ll: most severe lonn resulting in multiple fractures just from birth process . Type lll: most severe form beyond the perinatal period . Type IV: mild to moderately severe bone fragility
There is no known cure for osteogenesis imperfecta. Treatment is directed toward pre\ enting or controlling the symptoms. The cause of osteogenesis imperfecta is believed to be due to a genetic defect that causes imperfectly-formed, or an inadequate amount ofcollagen. The affected person has either less collagen than normal, or a poorer quality of collagen than normal, leading to \r eak bones that fracture easilY.

\ote:

Hr pophosphatasia is an inherited metabolic (chemical) bone disease that results from low lerels ofan enzyme called alkaline phosphatase. This enzyme is essential to the calcification ofbone tissue. The severity of hypophosphatasia is remarkably variable from patient to patient. Some patients have blue sclera that resembles ostognsis imperfecta. There may be delbrmity ofthe arms, legs and chest. Frequent bouts ofpneumonia can occur as well as

rccurrent tiactures.

J h pes of h) pophosphatasia:

. \eonatal:

severe manifestations, respiratory failure, marked hypocalcification of the skeletal structures . Infantile: h-vpercalcemia, premature loss of deciduous teeth, skeletal malformations,

failure to grorv

. Childhood: short stature. frontal bossing, usually normal calcium and phosphate levels . Odontoh! pophosphatasia: children and adults rvho have only dental problems

Important: The premature loss of teeth in children and adults is usually characteristic. These teeth also exhibit hypocalcification. Radiographically, the teeth display enlarged pulp chambers and pulp canals, deficient root development as well as alveolar bone loss. Rememtrer: Patient's with Paget's disease also have high levels of serum alkaline phosphatase.

. Gigantism

. Acromegaly
. Achondroplasia . Dwarfism

Coplrighi C

201

42 l'2012 - Denral Decks

A new 6 year old pediatric patient walks into your operatory with hls mother, You initirl physical assessment nots ! prominent foreh""d ;;;;;;;;;. ""d The prtietrt initirlly seems to have no eyebrows but you lrter rerlize thst the hair is just very fine and sptrse. When you shake her hand and she smiles, you llso notice that she is n.lssing teeth and the ones she has are cone shaped. Whst fu her most likely systemic condition?

. Piene Robin syndrome . Ectodermal dysplasia

. Cleidocranial dysplasia
. Peutz-Jeghers syndrome . Osteopetrosis
43
Coptrighr O
201

l-2012 - Dntal Decks

In over 90% of acromegaly patients, the overproduction ol GH is caused by a benign tumor of the pituitary gland, called an adenoma. Whether or not the epiphyses of the long bones have lused with the shaft is the main detenninant of whether gigantism or acromegaly will occur when there is oversecretion of growth hormone by the pituitary
gland.

Remember:

. Gigantism: tumor prior to adolescence (non-fusion o.f epip\,ses)


. Acromegaly: tumor after adolescence (fusion of epiphyes)

Oral manifestations ofacromegaly and gigantisrn include: enlarged tongue, mandibular prograthism, spacing of the teeth rvhich are usually tipped to the buccal or lingual side, oNing to enlargement ofthe tongue. Roots may be lorger than normal. See picture #33 in booklet.

\ote: Dn arfism (pttuitary dvtar/i)

is characterized by arrested growth. Frequently these people have limbs and features not properly proportioned or formed. It is caused by undersecretion ofgfowth homone. Oral manifestations include: eruption rate and the shedding ofthe teeth are delayed, clinical crowns appear smaller as do the roots of'the teeth, rhe dental arch as a whole is smaller causing malocclusion and the mandible is underder eloped.

Ectodermrl dvsplasia is an x-rinked recessive condition charactedzed by abnormar deveropmenr ofthe .i1n 3nd associated struct]lfes lhdir noi!!, and teeth, ond s$eat gldnd.rl. It involves all structures which are ::nr ed tiom rhe ctodcrm. It affects mares more than females. common clinical findings include hvpothrichosis /./e./?d.re in hait,). ho s\\,eat gland.r, leoding to h(,at iuh )l..r,La. c/lanodontia t,r oligodontia konplere or partiar^nhidrosis absenk of teeth). depresscd bridgc of nose, Iack of salivary grands l.,l the child appears much oldcr than rvhat he or shc is. There is no treatment lor the disease. hoNever
:-r r..!rmmodate the palient's

;::rrures can be fabricated for these patients. Keep in mind that they will need to be replaced periodicallv jaw growth. Scc picturcs #28 and #29 in booklet

Clcidocranial d]splasia is an autosomal dominant condition ofbony developmenr characterized by hypoplasia or aplasia ofthe clavicles' cranial bossing, ocular hypertelorism, and dental abnonnalilies which ::rrlrJe retaincd primar) teeth, malaligned leeth, the presence of multiple supcrnumerarv teeth and unerupted reeth- lmportant: The dentition itself, as obsen,ed by radiographs alone, often suggests the di:5o.rs. Sec picturc #30 in booklet Pierre Robin svndrome is an inherited disorder that presents the fblowintl in the neonate: severe micrognsthia and mfndit ular hypoplasia. se\ ere glossoptosis //),,, ariot rlt,fLttenent ofthe tongtrc), and high-arched or cleft pal:tte. This condition is characterized by respiratory problems.
Peutz-.Icghrs syndrome /PJSI is a genetic condition rnarked by hyperpigmentation (r"rfltrg ofthe lips .rnd \ometlmes other parts ofthe face, hands, and feet followed by the development ofbenign polyps called

arn'anomas throughout the intestines but primarily in lhe small intestine. See picture #35 in booklet

osteopetrosis /a&o.a lled Albers-Se honberg disease or narble bone./iredr., is an uncommon bone conJiiron thal mav be inherited as an autosomal dominant /&rr--reno .!, or recessive trait 6drle seri.n/s./. The .r3racteristic feature ofosteopetrosis is an abscnce ofphysiorogic bonc resorption owing to rcduced os(eoclastic activity' The lack ofbone resorption results in accumulation ofbone mass and manifesls itself :. ikeletal disturbances. including bone cavity occlusion. decreased hematopoietic activity, and growth re1:rrdation. Bone pain is the mosl liequent symptom. Blindness and deafness from sclerosis ofostia. anemin riom sclerosis ofbone marroq and osteomyeliris due to diminished vascularity are aiso seen. Dental findings include delayed enrprion. congenitally absent teeth. unerupted and marormed teeth, and enamer hyooDldsla,

ORAL PATHOLOGY

M&GJawDis

An old patient presents to your oflice with ill-fitting dentures. Radiographs reveal hypercementosis on roots and the patient is speaking embarrassingly loudly to you that she is more conscious ofhis baldness now that his hat doesn't fit anymore." These signs all point to what possible diagnosis?

. Paget's disease . Osteosarcoma . Fibrous dysplasia

. Albers-Schoenberg

CopyriShr C 2011

:0ll

Denral Decks

ORAL PATHOLOGY

M & GJawDis

You are consulting on a pathological case for a fellow dentist. The biopsy of the lesion sho\fls multinucleated giant cells and perivascular collagen cuffing. After asking about the clinical signs, your colleague mentions that the young patient seems always to be (staring offinto spac. and that she has.puffy cheeks." The most likely diagnosis of this case is:

. Aneurysmal bone cyst

. Central Giant Cell Granuloma . Tumor of hyperparathyroidism . Cherubism

45
Coplrighr ie

20ll-l0l:

Denlal Decks

booklet Paqet's tlisease ofbone is also called Osteitis Deformans' See picture #32 in appears Paget's discase ofbonc is a chronic, slouly progressive condition ofunknown ctiology(it cranium' pelvis' l'elnur' thc spine' include ofinvolvemenl sitcs frecluent most The toie lanitiatl. and fbrand stemum. Tbe bone may become dcnse, but liagilc, because of cxcessive breakdou'n of 50. Thc thc age ovcr adults affccts mation ofbonc. The disease is morc conmon in males and area' thc irffected in tlre bone of arca, defomlily signs and symptoms inclucle pain in thc aflectcd arca affccted ifthe loss hearing and headache and arca' su"sceptibility to fractures in lire affected slowly' dcvelop is the skull. Note: Thesc symptoms

***

When the jarvs are affected:

n-Iaxilla progresPatient who rvears a maxillary denture cotnplains that thcy do not fit aS thc sively cnlarges . Ultimatelyl the alveolar ridge wiclens. r,irh a rclative flatrening ofthe palatal vault . When tecth are prescnt, incrcased spacing as wcll as loosening is notcd . Hypcrcemcntosis of tooth roots, loss of lamina dura, oblilcralion of the PDL space and re-

sorption ofthe roots

]licroscopically, ostcoblasts
adr ances. dense bone

wlth

and multinuclcated osleoclasts arc lbund in abundance As the lcsion numeious reversal or growth lines are sccn' giving the tissue a mosaic

parrem.

Other important features of this disease include: . Palicnts may also give a history ofprogrcssively increasing sizc ofhats or nc'$ dentttrcs being
made at progiessively more frequent intervals. Note: This is due to bony changes' . Boncs are warm to touch due to incrcased vascularity

.\-raysofthesku|landthejawsdcmonstlatethctypical''cotton-wooll.appearance . Lab iests: Drastically increased serum alkaline phosphatase. Scrum phosphale and calcium
are normal. Urinary

calciun and hydroxyproline are increased' . Treatment: Recently. thc use ofcalcitonin and bisphosphonates as parathofmone antagonlsls latal ha\ e bcen eilective in suppressing bone resorption and dcposition lt is seldom

Cherubism is a benign, autosomal dominant condition ofthe maxilla and mandible, usuall-,- found in children by 5 years ofage (it olfects males 2:i). The vast majority ofcases occur in the mandible. The bony expansion is most frequently bilateral, although unilar eral involvement has been reported. The clinical appearance may vary from a barely discemible posterior swelling ofa singlejaw to marked anterior and posterior expansion of both jau s. resulting in masticatory, speech, and swallowing difficulties. Intraoralh', a
hard. non-tender swelling can be palpated in the affected area. There are no associated sl sterric manifestations. The deciduous dentition may be spontaneously shed prematurely, beginning as early as three years of age. There is often delayed eruption of the pennanenr dentition uhich is olten defective with the absence ofnumerous teeth and displace-

mcnt of those present.

Radiographicalll', the lesions characteristically appear as multiple, well-deltned, multilocular radiolucencies ofthejaw. The borders are distinct and divided by bony trabeculae. \ote: An occlusal radiograph of the maxilla may give a "soap bubble-like" picture
rr rrh

m.rrillrq

anrrum obliteration.

Histologicalll, the lesions bear a close resemblance to those seen in central giant cell granulomas. There are nlrmerous fibroblasts and rnultinucleated giant cells with prominent nuclei. A distinctive feature is eosinophilic perivascular cufling of collagen surrounding small capillaries throughout the lesion. Although this is not always present, perirascular collagen cuffing is regarded as pathognomonic for cherubism.
The treatment is cautious waiting as there is spontaneous regression of the tumors at around 25 - 30 years ol age. With increase in age and size of the patient. the defotmity
produced is less noticeable.

. Vitarnin A . Vitamin C . Vitamin D . Vitamin K

Coprighr O

201

45 l'2012 - Dental Decks

. Toxic nodular goiter


. Graves'disease . Hashimoto's disease . Addison's disease

17
Copyright @ 201 t-2012 -

De

al

D4ks

Hypoparathyroidism is a rare disorder associated with insufficient production ofparathyroid hormone, the inability to make a usable form ofparathy'roid hormone, or the inability ofkidneys and bones to respond to parathyroid hormone production.
Hypoparathyroidism can result from congenital disorders, iatrogenic causes (e.g., drugs, rcmoval ol the porathyroid glands during thyroid or parath!-roid sto'qery, radiation), rnfiltration ofthe parathyroid glands (e.g., metqstatic cqrcinoma. Il/ilson disease. sarcoid), suppression of parathyroid function, HIV/AIDS, or idiopathic mechanisms.

Hypocalcemia is the most important consequence of hypoparathyroidism. Symptoms occur when ionized calcium level drops to less than 2.5-3 mg/100 mL. The clinical manifestation is tetany. A positive Chvostek's sign (nitching oJ the.fdcial muscles v'hen
rapped on the facial nen e near the parotid gland) is characteristic ofhypoparathyroidism.

lmportant: The dental manifestations of hypoparathyroidism (i.e., delayed eruption, euantel ht,poplasia and blunted root apices) may be prevented by early treatment with vitamin D.
Remember: The term muscular dystrophy refers to a group ofgenetic diseases marked by the progressive weakness and degeneration olthe skeletal, or voluntary muscles, which confiol molement. Oral manifestations include an increase in dental disease iforal hygiene is neglected. weakness in the muscles of mastication leading to decreased maxillary biting ibrce and a higher-incidence of mouth breathing and open bite.

The rerm hr..perthyroidism refers to any condition in which there is too much thyroid hormone tln rorr'r, in the body. This most commonly results fron a generalized overactivity ofthe entire ihlroid gland. a condition also known as diffuse toxic goiter or Graves'disease. Altematively, one or nrore nodules or lumps in the thyroid may become overactive, a condition known as toxic nodular goiter or Plummer's disease, The primary role ofthyroxin is to stimulate cellular merabolism. grorvth and differentiation of all tissues. In excess, it leads to high basal metabolism, tilrgue. \! eight loss, excitability. elevated temperature and generalized osteoporosis. Oral manifestations are not too remarkable, but ifthe disturbance begins in the early years of life, the F.en'rature eruption of the teeth and the prernature loss of the deciduous dentition are common

dndings. T! pes of hi perthJ-roidisml. (Note: Besic symptoms of htperth,'-/oidisn dre present as h,ell as

lJdi|io

aI \nptonts). . Gra\'es'disease: is the most common form, occurs most frequently in women under - goiter knlarged thyoid which moy catrse a bulge in the netkl

50

erophthalmos is common thickened skin over the shin area . Plummer's disease (toxi. nodular goiler; afibcts both genders usually over 50 Erophthalmos is rare
Ofien uni-system, may present with only cardiac disease
causes temporary hypcrthyroidism, usually followcd with hypothl roidism. Thyroiditis is an inflammation ofthe thyroid gland.

lmportant: Thyroiditis

Therc are three main types of thyroiditis:

. Hashimoto's thyroiditis . Subacute granulomatous thyroiditis . Silent lymphocytic thyroiditis

. Dwarfism
. Mlredema . Cretinism
. Acromegaly

48 Coprighr C
201

l-2012 - Dental Decks

The clinical features ofthe primiry form of which disease is classically described as "stones, bones, groans, and moans?"

. Paget's disease . Hypophosphatasia

. Hyperparalhyroidism
. Hyperthyroidism

49
Coplright O 20ll-2012 - Denral Decks

Hypothyroidism refers to a condition in which the amount ofthyroid hormone in the body is below normal. This is the most common form ofthyroid function abnormality, and is far more common than hyperrhyroidism. This condition is considerably more comn.ron in women than in men. The most common cause of h)?othyroidism is Hashimoto's thyroiditis. The second most common cause is the treatment of hyperthyroidism. Hypothyroidism is characterized by pufiiness of the face and eyelids and swelling of the tongue and larynx. The skin becomes dry and rough and the hair becomes sparse. The individual has a low basal-metabolic rate and a low body temperature. The affected individuals also have poor muscle tone, low strength and get tired very easily. Mentally they are very sluggish. The treatment of hypothyroidism is straightforward and consists of administering thyroid hormone (t hyt oxin). Se\re hypothyroidism in a child is called cretinism. Due to a lack of thyroid hormone, there is a retardation of growth and an abnormal development of bones. Mental retardation is caused by the improper development ofthe CNS. If this condition is recognized early, it can be markedly improved with the use of thyroid hormones. Note: Extreme hypothyroidism in adults is called myxedema.

Dental findings in a child with hypothyroidism include an underdeyeloped mandible with an overdeveloped maxilla, enlarged tongue which may lead to malocclusion. delayed eruption ofteeth and deciduous teeth being retained longer.

\ote:

H) perparathyroidism (von Recklinghausen's disease of bone) is a tnctabolic disorder in which the parath.lroid glands produce too nruch pamthyroid honnonc. Too much parathyroid horrnone causes loo much calcium to be released from bon. It may be caused by a f'unctioning parathyroid tunor or conlpensatory parathyroid hyperplasia due to renal lailure, malabsorption, or vitamin D deficrency. Thcrc is a ferralc prcdilection and it affccts middlc agcd adults. Thc symptoms include: Ioss ol'appetite, increasing thirst, lrequent urination, Jethargy and fatigue, muscle weakness, joint pain and constipation. Important: pathologic fraettre (due lo the marked rcsorplion of borc) mal bc thc first symptom ofthe disorder Intraorally, thcrc is diffuse bonc loss causing malocclusron and shifting

ofthc teeth.

The chie I radiographic finding is the appcarance ofwell-defined cystic radiolucencies ofthejaw, *hich mav be unilocular or nultilocular Partial loss ofthe lamina dura is seen around thc tccth. See picture #34 in booklet

lmportant: Histologically, multinucleated giant cells are scattered within a dclicate fibrocellular
Accumulalions ofhcmosiderin and extravasated red blood cells also rc present. As a result, thc ussue-i appear rcddish-brown, accounting for the term "brown tumor." These lesions arc miuroscoprcall) idcntical to qentral giant cell granulomas.

'iroma.

Thc discasc spectrum ofprimary hyperparathyroidism rangcs from asynptomatic cases ldiagnosetl fion routine senon calcium determiralrotrs/, to severe cases oflcthargy and occasional coma. Earl) synrptoms include faligue. weakness, nausca, anorexia, polyuria, thirst, depression, and constipation. Frequcntly, bone pain and headaches are present. There are several clinical f'caturcs associated primary hyperparathyroidism, classically described as "stones, bones, groans, and moans." Lcsions ofthe kidneys, skclctal system, CI tract, and nervous systcm are responsible lor this syndrome complex. The rcnal component includes the prcsenc ol'renal calculi.
Management of primary hyperparathyroidism is aimed at eliminating the parathyroid pathology. Surgery is the treatmcnt ofchoice.

. Vitamin A

. \4tamin D

. Vitamin

. Mtamin E

50
Coplrighr
@

2011,2012 - Dntal Decks

. Difficulty with mastication and swallowing . Higher incidence ofperiodontal disease and caries . Attrition ofthe teeth

. Multilocular radiolucencies of the jaws

Coprrishr

201l-2012, Denral Decks

Osteomalacia involves softening of the bones caused by a deficiency of vitamin D or problems with the metabolism ofthis vitamin. This softening ofthe bones occurs because the bones contain osteoid tissue which has lailed to calcily due to the lack of Vitamin D. Note: Osteomalacia may occur as a complication ofsteatorrhea secondary to chronic pancreatrtls. Clinical findings: . Osteopenia

. Bone softening/deformity: hourglass thorax, bowing of long


. Increased fractures. biconcave vertebral bodies

bones

. Mottled skull All


bones are affected, specifically their epiphyseal growth plates. Osteomalacia appears to be more common in women. This condition mav be asvmDtomatic until fracture occurs.

Blood tests may show: . Lou levels of vitamin D . \la) also show low calcium and phosphorus levels . .\lkaline phosphatase levels can be high Rickets is osteomalacia in children. It causes skeletal deformities. It is usually accompanied bl listlessness, initability and generalized muscular weakness. A child with rickets may hare borvlegs and develop a pigeon breast and a protruding stomach. The teeth in a child l ith rickets are affected as follows: delayed eruption, malocclusion and developmental abnormalities ofthe dentin and enamel alone with a hi{:her caries rate.

Cerebral palsy is a term used to describe a group ofdisorders affecting body movement and muscle coordination. It is due to an insult to or anomaly ofthe brain's motor control centers. This damage interferes with messages from the brain to the body, and from the bod\ to the brain. The effects vary widely from individual to individual. Cerebral palsy rs characterized primarily by spastic paralysis or impaiment of control or coordination
!r\ er \ oluntary muscles and is often accorrpanied by mental retardation, seizures and dis!rrders of Yision and communication.

\ote: No intraoral

conditions /r/rose /isted on the normal population.

anomalies are unique to persons with cerebral palsy. However, several the ltont ofthe card) are more comnton or more severe than in

Remember: Down syndrome is a congenital defect caused by a chromosomal abnorrnaliy.- (trisomy 2 l). It is marked by various degrees of mental retardation and characteristic phvsical features such as a short, flattened skull, slanting eyes, a thickened tongue r.tissured), broad hands and feet and other anomalies. Other oral rnanifestations include mandibular prognathism. increased incidence ofperiodontal disease, delayed eruption of teeth. higher incidence of congenitally missing teeth, malocclusion, and enamel dysplas

ia.

. Sjdgren's syndrome . Cystic fibrosis . Cerebral palsy


. Down syndrome

52
Cop,righr O
201

1,2012 - Dental Dcks

. Hepatitis A . Hepatitis B

. Hepatitis

CoP)rght O 201l-2012 - Denlal Decks

There is a significantly reduced caries rate in patients with cystic fibrosis. This is probably the result of alterations in saliva and the long{erm use of antibiotics.

*** Important:

Cystic fibrosis is a congenital metabolic disorder that causes the exocrine glands (vhich are glands thqt seqete.fluids into a duct) to produce abnormal scretions, resulting in several symptoms, the most important of which affect the digestive tract and the lungs. In some glands, such as the pancreas and those in the intestines, the secretions are thick or solid (an ext essivef;, viscous mucous/ and may block the gland completely. The mucousproducing glands in the airways ofthe lungs produce abnormal secretions that clog the airrvays and allow bacteria to multiply. The sweat glands secrete fluids that have a high sodium and chloride content. Note: The staining olthe teeth is most likely due to the lact that patients with cystic fibrosis are usually subjected to large amounts of tetracyclines

during childhood.
Si mptoms of CF include:

. . . . . .

Poor growth despite good appetite Malabsorption and foul, bulky stools: steatorrhea Chronic bronchitis (COPD) with cough Recurrent pneumonia: respiratory infections Clubbing offingers and toes Banel-chested appearance

l4tlLlhighly contagious infectious disease involving the liver It is usually transrnined by dre fecal-oral route. However, it may also be transmitted parenterally. tlepatiris -{ usually rcsults liom ingestion ofcontaminated food. milk. or water. Many outbreaks of tl'ris tvpe are traced to ingestion ofseafood from polluted water. It is caused by an RNA enteroYirus. [t most often occurs in young adults. The initial symptoms (bver malaise, ,tbdoninal pain. anorexia, jaundice.)of Hepatitis A appear after an incubation period of l{epatitis A is
a -1-6

eeks.

Darnage to the liver cells. also results in increased serum levels of enzymes, such as transaminases, nonnally active in liver cells. The detection ofincreased serum levels of rhese enzlrnes is used in diagnosing this disease. In most cases ofHepatitis A, the infecrion is selt'-limiting and recovery occurs within 4 months.

Oral complications: The only oral complication associated with hepatitis is the potential for abnormal bleeding in cases ofsignificant liver damage. Ifsurgery is required, it is advisable to check with the patient's physician prior to scheduling the sLrrgery.
I . The presence of surface antigen (A or B) in a patient's serum indicates that the patient is potentially infectious for Hepatitis (carrier state). ...,.:-,,,a:. 2. Hepatitis viruses are very heat-resistant (morc so than the AIDS virus). 3. Autoclaving properly will kill these viruses.

\otei

. Hepatitis A . Hepatitis
C

. Hepatitis D . Hepatitis E

54 Coplright O 201 l -20


12

- Deotal Decks

Atl of the following aid in wound healing Excepr one. Which one is the EXCEPTIOM

. Highly vascular areas (i.e., tongue)

. Hyperthermia
. Younger
age

. Cortisone

Copyrighr O

20ll-2012,

Dental Decks

The Hepatitis B virus (HB V) rs a double-stranded DNA virus with worldrvide distribution, transmitted by parenteral and sexual contact. Risk factors include multiple sexual partners, intravenous drug abuse, and rcccipt ofblood products. Its incubation pcriod is 40 to 100 days, and it can be recovered from all body fluids, most importantly, blood, brcast milk, and amniotic fluid. The signs and symptoms are similar to hepatitis A (fever abdominal pain, nausea, etc.) but there is a longer incubation priod f6-8 ueefs). The symptoms arc slorver in devcloping but are ofa longer duration. Most patients rccover fully, however, some develop chronic liver disease.

Previously, viral hepatitis that was not caused by the type A or type B virus was callcd "nonA. non-B hcpatitis." Rccently three more viruses havc becn identilied that causc some of these non-A. non-B infections.
These new Yiruses includ:

. Hepatitis C: is a serum hepatitis that is caused by a virus antigenically different from Hepatitis viruscs A and B. Most cases olpost-transfusion hepatitides are ofthis type. It is usually much milder than A or B but is otherwise clinically indistinguishablc from thcm. Thcrc is a higher incidence of chronic disease (chroni< lrcp41llrt, cirrhosis and hepatocellular carcinoma. Note; Hepatitis C is now the most common reason for liver trirnsplantation in the U.S. . Hepatitis D is found only in patients with acute or chronic episodes of Hepatitis B, and it makes the Hcpatitis B infcction morc sevcrc. Drug addicts are at relatively high
. Hepatitis E: is transmittcd cntcrically, much like Hcpatitis A. It causes occasional epid.'nics similar to those caused by Hepatitis A. So far these epidemics have occurred only in undcrdcvcloned countries.

Other factors that influence the rate ofhealing: . Location of the wound: wounds in an area in which there is a good vascular bed
hc'al considerably more rapidly than wounds in an area which is relatively avascular . Phl sical factors: severe trauma to tissue is a deterent to rapid woLlnd healing. The

local temperature in the area of a wound influences the rate of healing. ln environ rnental hyperthemia, rvound healing is accelerated; while in hypothermia, healing is
dela1,ed.

. Circulatory factors:
of q ounds

anemia and dehydration have been found to delay the healing persons heal considerably more rapidly than

. -{ge of patient: wounds in younger r ounds in elderly persons

.Infection: bacterial invasion will retard healing . Hormonal factors ACTH and cortisone are substances that have been known to interlere rvith the healing of wounds. Diabetes mellitus (in.sulin deJiciencl) is one ofthe
most $ idely recognized diseases in rvhich there is significant, clinically evident. retardation in repair of wounds.

\utritional factors

such as the amount ofprotein a patient is consuming is one ofthe most rmpofiant factors which may effect the speed ofwound healing. Hypoproteinemia has been shos'n to delay wound healing, while having a high protein diet has been shown to accelerate u ound healing. Vitamins, especially Vitamin C, have been shown to be important in proper wound healing.

. 3 mm well-encapsulated fibroma
. Necrotizing sialometaplasia ofthe hard palate . 2 mm papilloma of left commissure of lips

. Aneurysmal bone cyst

Copyrighr O 20ll-2012 " Dental Deckr

. Nasopharyngeal carcinoma

. Oral hairy leukoplakia


. Burkitt's lymphoma . Koplik spots

Copright O 201l-2012 - Denral Decks

A biopsy is a procedure perfonred to remove tissue or cells from the body for examination under a microscope. Biopsies are usually performed to detemine whether a tumor is malignant or to detemine the cause of an unexplained infection or inflammation.
When the entire tumor is removed, it is called excisional biopsy technique. tion ofthe tumor is removed, it is called incisional biopsy technique.

lfonly

por-

An incisionaf biopsy (also called diagnostic biop.st) is done when lesions are too large to excise initially without having established a diagnosis or are of such a nature that excision would be inadvisable.
An excisional biopsy is preferred ifthe size ofthe lesion is such that it may be removed along u ith a margin ofnomral tissue and the wound can be closed primarily. Erample: A l -cm exophl-tic mass flfrich is a lesion that grovs ouf,rordfrom an epithelial surface) on the cheek.

\ote*

is the most reliable technique to diagnose soft tissue lesions. 2. The fixative ofchoice to preserve biopsy specimens lor routine histological

l. Biopsy

examination is l0%o buffered formalin. 3. The scalpel is the instrument of choice since it cleanly removes the tissue and does not dehydrate it as cautery or the high-frequency cutting knife may. .1. The rationale for surgical removal and biopsy of a large periapical lesion suspected to be of inflammatory origin is that a clinical diagnosis can be confirmed microscopically. This is the only way to distinguish between a granuloma and a cvst.

The Epstein-Barr virus (EBV) is a member olthe herpes virus group. It causes infectious mononucleosis and has been associated with the subsequent development of two forms of cancer: Burkitt's lymphoma and nasopharyngeal carcinoma. EBV is also associated rr ith hairy leukoplakia, an opportunistic infection resulting in white patches ofthe Iareral tongue. Hairy leukoplakia is a nonmalignant lesion seen almost exclusively in AIDS patients. The virus specifically infects Blymphocytes and some epithelial cells. Ir is associated rvith the production of atypical lymphocytes and lgM heterophile antibodies identified by the heterophile test (also callecl the mononucleosis spot lest). This antibody eventually appears in the serum of more than 80% ofthe patients with irteL-rious mononucleosis, hence it is highly diagnostic of the disease

\otes

l.There are no specific oral manifestations ol infectious mononucleosis, although secondary Iesions do occur 2. Neck srvellings are cl.raracteristic ofinfectious mononucleosis, Hodgkin's disease and tuberculosis.

Remember:

. Rubella viruses cause German measles (rubella),which present with 'ash (flat, pink spots on theface and then spreads to other bodl,parts).

characteristic

. Paramyxoviruses can cause measles (rubeola) and mnmps. Rubeola is characterized by fever. malaise and by the formation of Koplik's spots in the oral cavity. These spots are small. bluish-white lesions sunounded by a red ring. They cannot be wiped offand occur opposite the molars. Mumps cause enlargement of the parotid glands. Serious complications include deafness in children and orchtlis (in/lammation ol the testis) in
males past puberty.

Misc.

Epithelioid cells and giant cells are derived from macrophrges and are important in the development of:

. Initial inflammation

. Granulomatous inflammation
. Acute inflammation

Subacute inflammation

CopyriSht O 201l -201: - Dental Decks

ORAL PATHOLOGY

Misc.

A patient you saw yesterday had minor swelling of the submandibular space associated with a carious #31. You prescribed amoxicillin and sent him home. II called today to say the swelling has gotten worse. You squeeze him into the schedule and notice that he has trouble breathing. You call an arnbulance to escort him to the ER and tell the paramedics that he has Ludwig's angina, Ludwig's angina is a severe and spreading infection that involvs the:

. Submental

and sublingual spaces only

. .

Submandibular, submental, and sublingual spaces unilaterally Submandibular and sublingual spaces only
and sublingual spaces bilaterally

. Submandibular, submental,

59
Copyrighr (}

20ll

20ll

DentalDecks

Granulomatous inflammation is a subtype of chronic inflammation and is characterized by granulomas, which are nodular collections of specialized macrophages referred to as epithelioid cells. A rirn of lymphocytes usually sunounds granulomas. Granulomas are produced by multinucleatd giant cells (Langerhans giant cells and./brcign bod1, giant cells). All the other cell types characteristic of chronic inflammation, including plasma cells, eosinophils, and fibroblasts, may also be associated with granulomas.

Note: Granulomatous inflarnrnation is characteristically associated with areas of caseous necrosis produced by infectious agents, particularly Mycobacterium
tuberculosis,

. Tuberculosis:
giqnt cells).

caused by the inhalation of Mycobacterium tuberculosis. Oral nonhealing chronic ulcers lollow lung infection. Important: Characterized by caseating granufomas with multinucleated giant cells (Langerhans giont cells andJbreign body

Etiologic agents associated with granulomatous inflammation;

Infectious agents - TB and leprosy, which are mycobacterial diseases

. . .

Fungal infections (blastomyco,sis, histoplasmosis, qnd coccidioidom)'cosis) Spirochetes (Treponema pallidum, which causes syphilis) Cat scratch disease (caused h; an unnamecl gyam-negdive organisnt) Foreign material (e.g., suture or talc) Sarcoidosis (unknown etiology; it is non-necrotizing) Crohn's disease (il ls non-caseating)

Lud$ ig's angina oflen results fiom an odontogenic infection. As a result, the bacteriologl of theses infections generally involves oral flora, particularly anaerobes. Other recounized etiologies of Ludwig's angina include poor oral hygiene, IV drug abuse, trauma,
and tonsiliitis.

It

. Rapid onset . The three facial spaces are involved bilaterally . Board-like swelling offloor olthe mouth and no fluctuance is present . Tlpical "open mouthed" appearance
. FIP\ siiAn of the tnnore

is characterized by:

. Drooling. trismus, and fever . Ditllculty eating, swallowing, breathing . Tachvcardia. increased respiration rate
. Can lead to glottal edema; asphyxiation

-\inr av management, massive antibiotic coyerage (IV),


are the mainstays oftreatment.

and surgical incision and drainage

Verv important: The most serious complication of Ludwig's angina is edema of the glottis (i'hich is a slit-like opening behreen the true vocal corcls).

. Inflammation . Administration of hydroconisone . Necrosis

. Suppuration
. Pregnancy

60
Coptrighr O
201

l-2012 -

Dflld

Decks

. l-2 mglkg
. 5- l0 mg/kg

.8-10 mg/kg

. t2-15

mg/kg

61

Copright O 20ll-2012 - Denbl Deck

The ESR is the rate at which red blood cells settle out in a tube of unclotted blood, expressed in millimeters per hour Blood is collected in an anticoagulant and allowed to sediment in a calibrated glass column. At the end ofone hour, the lab techniciar measures the distance the erlthrocytes have fallen in the tube. Elevated sedimentation rates are not specific for any disorder but indicate the presence of inflammation. Inflammation causes an alteration ofthe blood proteins which makes the red blood cells aggregate, becoming heavier than normal. The speed with which they fall to the bottom ofthe tube corresponds to the degree of inflammation. ESR rises during: . lnflammation

. Tissue degeneration

Suppuration

. Necrosis
l\Jote: Certain non-inflammatory conditions, such as pregnancy, are also categorized by hish sedimentation rates.

Death may result from ingesting as little as 2 g offluoride in an adult and l6 mg,&g in children. Symptoms may appear with 3-5 mg/kg offluoride.
S)

***

mptoms of acute fluoride poisoning include nausea, abdominal pain, vomiting, diarrhea, Monitor vital

conr ulsions. and hypotension.


The treatment for acute fluoride poisoning includes: 1) Call poison control center, 2) iigns. 3) Initiate basic life support as needed and 4) Get patient to the hospital

Fluoride poisoning may be ac\te (caused b1' a single larye close offluoride) or chronic (caused hr long-term ingestion offluoride). The characteristic signs ofchronic fluoride poisoning are:

l. Osteosclerosis of the bones: which results fiom long-term ingestion of water with l0 to 15 ppm offluoride. :. Dental fluorosis (enamel hypoplasia): which is due to fluoride intake during the calcification stage oftooth development. This can occur in permanent and deciduous teeth.

\ote: [t has been estimated that the average American diet contains about 0.2 to 0.3 mg of fluoride per day. lf I ppm of fluodde is added to the drinking watel about I to 2 mg of fluoride will be added to the diet daily. Balance studies have shown that when quantities offluoride ingested do not exceed 4 to 5 mg daily, little is retained by the body. The finding indicates the safety of the preventive dentistry programs based on the addition of fluoride to drinking water in concentrations of approximately 1 ppm.

Important: Fluoride normally accumulates slowly in bones as a person ages. However, if ingested in very high amounts, it accumulates rapidly. The intake of calcium in high doses
rvill reduce the absomtion ofdietarv fluoride.

. Hereditary

. Medications . Tooth decay


. Mouth breathing

62
Coplright O
201 1-201 2

, Dental Decks

. Tongue

and mandibular alveolar ridge

. Buccal mucosa and pharyngeal pillars . Palate and maxillary gingiva . Mucobuccal fold of lower lip

63 Copyriglr
@

201l-2012 - Dntal Decks

Xerostomia (dry nouth) is not a disease, however, it can be a symptom of certain diseases. Many times xerostomia is caused by failure ofthe salivary glands to function normally, but the sensation can also occur in people with normal salivary glands. Xerostomia can cause health problems by affecting nutrition, as rvell as psychological health. At its most extreme, it can lead to rampant tooth decay and periodontal disease.

of xerostomia is medication. The main culprits are antihistamines, antidepressants, anticholinergics (e.g., atropine and scopolamine), anorexiants, antihypertensives, antipsychotics (e.g., chlorpromazine and prochlorperazine), anti-P arl<tnson agents, diuretics and sedatives. The most common disease causing xerostomia is Sjiigren's syndrome /S.9/, a chronic inflammatory autoimmune disease that occurs predominantly in postmenopausal women. Sarcoidosis and amyloidosis are other chronic inflaurmatory diseases that cause xerostomia. Other systemic diseases that can cause xerostomia include rheumatoid arthritis, sl stemic lupus erythematosus, and scleroderma. Remember: Xerostomia is the most common toxicity associated with radiation therapy to the head and neck. Comrnercially available saliva substitutes in general, contain an agent to lncrease vlscosiq. such as carboxymethylcellulose or hydroxymethylcellulose, minerals such as calcium and phosphate ions and fluoride, preservatives such as methyl- or propylparaben, and fla\ oring agents. Examples include: Xero-Lube,i Salivarti and Optimoist.s

Perhaps the most prevalent cause

createst numbcr

\lalignant melanoma accounts for only 4% of all skin cancers; however, it ofskin cancer rclatcd dcaths worldwide.

causes the

Clinical features: . Malc' morc oftcn than fcmalcs

. L sually palalc or rnaxillary gingiva


. 5th decade or older . A. B. C. D's of melanoma:

-\ = asymmctry B = border inegularity C : color variability (brown, black, blue, gray, pink) D = diameter of /o/ief greater than l/4 inch
Excessive exposure to UV radiation from the sun may bc the primary cause ofmclanoma. \falignant melanoma has bcen linked to both a lot of sun exposure over a lifetime and to painful sunbums during childhood.

\{alignant mclanoma is an uncommon neoplasm ofthe oral mucosa. It exhibits a definite predilcction for the palat and the maxillary gingiva/alveolar ridge. Unfortunately, oral
mucosal melanomas have a dismal prognosis. See picture #36 in booklet is a mole. Almost all moles are normal. Atypical (dl,splastic) nevi are unusual moles that are gencrally larger than normal moles and are eithcr flat or have a flat part. They have irregular bordcrs and oftcn are variable shades olcolor, particularly brown. The presence ofdysplastic nevi may mark a greater risk ofmalignant mclanoma developing on apparently normal skin. See picture #46 in booklet

\ote: A nevus

Name the malignant, epithelial cell tumor thrt chancteristically begins as a papule and enlarges peripherally, developing a central crater that erodes, crusts and bleeds. An example ofthis tumor is shown below.

Copyrighr C 20ll-2012 - Dental Decks

ORALPATHOLOGY

A 62-year-old patient comes into your oflice complaining of his loose lower molars. The health history reveals recent lower back pain. On a hunch, you send him down the hall to the orthodontist to take a lateral skull radiograph which reveals "punched-out" radiolucencies. You will refer this patient to the physician suspecting a diagnosis of:

. Non-Hodgkin's lymphoma

. Hodgkin's lymphoma . Multiple Myeloma


. Langerhan's Cell Disease

Basal cell carchoma is a common low-grade skin cancer that rarely metastasizes, but the local invasion by direct extension destroys the underlying and adjacent tissue. lt frequently develops on the exposed surfaces of the skin, face and scalp in middle-aged or elderly persons, it is very rare in mucosa. The primary cause ofthe cancer is excessive exposure ofthe sun or to x-rays.

Clinical features: . Presents as a nonhealing, indurated chronic ulcer


Males are more affected than females 4th decade of life or older . Located primarily on sun exposed areas ofhead and neck with the nose being the most common srte

. .

Risk factors: childhood sun exposure, blistering sunbums, fair skin, blue eyes, blonde
or red hair.

Important: It is locally destructive and metastases are exceptionally rar.


The treatment for basal cell carcinoma is eradication ofthe lesion, often by electrodessication or cryotherapy. Recurrence is unconmon ifproperly treated. r'-ote: MOHS micrographic surgery is the most effective and state-of-the-art procedure even if the skin for skin cancer today, which offers the highest potential for recovery cancer has been previously treated by another method and recurred. With the MOHS technique. physicians are able to see beyond the visible disease, to precisely identify and remoVe the entire tumor layer by layer while leaving the surrounding healthy tissue intact and unharmed. As the most exact and precise method of tumor removal, it minimizes the chance ofre-growth and lessens the potential for scarring or disfigurement.

f.r lso known as "Plastna Cell Mveloma') is a disease ofabnormalplasma cells that mosr often build up in the bone marrorv. The cells form tumors in many bones ofthe body.

\Iultipfe mleloma
Clinical Features:

. \Ien 2:l

(40-70 rears o/r// Vcrtcbrae, ribs, and skull are most frcquently involvcd; pain in lumbar or thoracic region is a common early symptom. . Jaws are rarely a primary site, but become involvcd in 70% of the cases, molar ramus area
most common site. Symptoms include swelling, pain, loosening ofthe teeth, and paresthesia.

Radiographic features: variable; slight demineralization to extensive dcstruction, characteristic finding rs multiplc. small, discreet "punched out" radiolucencies in involvcd bones ln a patient
:u:pccted ofhaving multiple myeloma. a lateral skull radiograph is best to confirm the diagnosis.

Laborator-'is usualll

fi

ndings: important in establishing diagnosis:


a

. Abnormal monoclonal immunoglobulin protein


ofthe IgG or IgA class. with

peak known as an M spike. The immunoglobulin monoclonal Jight chain cofrponent.

. Lrinary monoclonal light


. Anemia

chains, so-called Bence Jones

proteinu

Treatment and Prognosis: chemotherapy, radiation; prognosis poor.

Important: A form ofamyloidosis occurs in l0% of multiple myeloma patients. It is due to the formarron of complex proteins in which immunoglobulin light chains are precursors. Amyloid ptotein is deposited in various organs an can lead to organ dysfunction fspeciallr kidnq', hedfi, GI ltucl, liver and
I Plasmacytoma is a localized collection ofmonoclonal plasma cells. The discasc is dividcd

-i.';$t:l: ,.:rr' ofthe diagnosis restswith thc potential forthese disorders to progress to multiple myeJoma. 2 Primary presentation of intraoral non-Hodgkin's lymphoma is uncommon. In gencral,

\61*,-'intoprimaryplasmacytomaoftheboneandextramedullaryplasmacytoma.Thcimportance
the oral manifestations occur secondary to a morc widespread distribution throughout the bodY 3. Hodgkin's disease involving the oral cavity is also considered a raity, but it still can appear here initially.

. Superficial spreading
. Lentigo maligna . Acral lentiginous
. Nodular

Cop"ighi

@ 201 I

66 '2012 - Dental Decks

. The presence of Nikolsky's sign

. The presence ofnodules . The presence ofregional lymph node involvement . The presence of erythroplakia

CopFight O 20ll-2012 - Dental Dcks

Malignant melanoma involves the cells /melqnoc),tes) that produce pigment (zrelarirl, which is responsible for skin and hair color Melanoma can spread very rapidly and is the most deadly form ofskin cancer. lt is the leading cause of death from skin disease. Melanoma may appear on normal skin, or it may begin at a mole frevusl or other area that has changed in appearance. Some moles present at birth may develop into melanomas. The development of melanoma is related to sun exposure, particularly to sunbums during childhood, and is most common among
people with fair skin, blue or green eyes, and red or blonde hair. There are four major types ofmelanoma;
I . Superficial spreading melanoma is the most common type of melanoma. It is usually flat and irregular in shape and colol with varying shades ofblack and brown. lt may occur

at any age or site and is most common in Caucasians. 2. Nodular melanoma usually starts as a raised area that is dark blackish-blue or bluish red,

although some lack color Poorest prognosis, 3. Lentigo maligna melanoma usually occurs in the elderly. It is most cornmon in sundamaged skin on the face, neck, and arms. The abnormal skin areas are usually large, flat, and tan with intermixed areas of brown. DeveJops from pre-existing lentigo maligna
,1.

(Hutchinson freckle). Acral lentiginous melanoma is the least common form of melanoma. It usually occurs

on rhe palms, soles, or under the nails and is more common in African Americans.

Important: Malignant melanoma exhibits either initial radial (do not metastqsize) or yertrc^l lnetastasis mdf occur) gtowthpattems within the skin. Radial growth phase melanomas include the superficial spreading, lentigo maligna, and acral lentiginous melanomas. Vertical qro\\th is characteristic of nodular melanoma.

\ote: Skin cancer is a very common malignancy in the United States. Olthe different types, basal cell carcinoma is most common followed bv souamous cell carcinoma and the least common is malisnant melanoma.

Clinical staging of malignant neoplasms: T:-Nl method of assessing the prognosis and thempy of malignant neoplasms is based upon 1) the size ofthe primary tumor, 2) the presence ofregional lymph node involvement and 3) the presence of distant metastases. This is represented as:
The

.T

. .

\I :

size of the primary tumor = presence ofregional lymph node involvement presence ofdistant metastases

The use of this classification system is becoming less common. A major shortcorring ofthe TNM classification is the lack ofquantification of depth/penetration ofthe

\ote:

umor.

. Dorsum
. Ventral surface

'Tip
. Posterior lateral border

68
Coplrighr O 201l-2012 - Dnral Decks

Squamous cell carcinoma

ofthe palate

. Squamous cell carcinoma of the nasopharynx .


Squamous cell carcinoma Squamous cell carcinoma

ofthe oropharynx ofthe maxillary sinus

69 Coplaight
@

201l'2012 - Dental Dcks

Squamous cell carciloma i'SCC) accounts for greater than 9070 oforal cavity and oropharyngeal malignancics. Lip carcinomas account for 25,q/o b 30o% ofall oral cancers. Approximately 909/0 occur on the lower lip due to increascd sun exposure. IntraorallJ', the most common site is thc posterior lateral border of the tongue, it is particularly uncommon to have lesions develop on thc dorsum or in the tip ofthe tonglle. Metastases from tongue cancer are relativcly common at thc time ofprimary trcatmcnt. In gcneral, metastatic deposits from SCC ofthe tongxe are found in the lymph nodcs ofthe neck. usually on the ipsilateral side. The first nodes to become involved are the submandibular orjugulodigastric nodes at the angle ofthe mandiblc. Noter The floor of the mouth is the second most common intraoral lesion

of SCC.
f,liolog/
Lip
surlighl- pjpe

Clinic!l Chrrrcterirtics
NIen betwee. 50 10.

rermil-

Lo*er liP,
Upper lip
Fair;

lion oflower lip. chronrc

nonhe!liig ulcer, keratotic


Tongue Men oler 60. posterio. Irter|l border . indurared non healing ulcersith leukoplak'a, erytbroplakia

indurated ulcer: leukoplakia.

Me! in their 70
and girgiva

s:

painless

exophyl'c'nas verrucons
Soft palate-10-20 %

asympbmatic. red or sh'te


plaqres as ul.erared and

Locrtion
\asopharyrL\

Incidence

Etiology

Clinical Charrcteristics
Men 30-40 years old, roof or latcral wall is most common site, cervical mass, earache sore throat, nasal

Treatment
Su.gery and

Less than 2% of all Tobacco, alcohol cancers in fhe U.S.

irradiation

obsnucion
Palate

l0% ofalloral

Tobacco, alcohol,

dntue irritation

Men over 60, soft palatc is more common than hard palate, painful ulce., Ieukoplakia, cxoph'1ic mass Men over 50. sore throat, dysphagia, paintul ulcer, cewtcal mass Men over 40, chronic sinusitis, butging of the palate, loosening of teeth, paresthesia in cheek

Surgery and

inadialion

Orophartr{

l0% ofall hcad and neck cancers

Tobacco, alcohol

Surgery and irradiation Surgery and

\laxillary sinus 30% ofallhead


and neck cancers

Unknown

iradiation

. "Horizontal" growth

phase

. "Circular" growth phase

. "Vertical" growth

phase

. "Radial" growth

phase

70 Cop)'righl O 20ll-2012 - Dental Decks

' Lip
. Tongue
. Floor of mouth . Buccal mucosa

71

Coprrighr O

201 |

-2012 - Dental Decks

Melanomas develop initially as a flat phase without competence for metastasis called the "radial growth phase". This refers to the initial growth of a melanona in a horizontal plane. It is clinically macular or only slightly elevated. They then may evolve focally an elevated part, the vertical growth phase, with metastatic competence. The "vertical" growth phase is the phase that begins when neoplastic cells populate the underlying dermis. lt is characterized clinically by an increase in size, a change in color, nodularity and, at times, ulceration. Metastasis is possible once the melanoma reaches this phase. Overall, patients with vertical growth phase melanomas have a cure rate of 7070. Note: This phase predoninates in nodular melanoma.

Radial growth phase melanomas, although invasive, have a cure rate that approaches
with surgery alone. Radial growth phase melanomas include the superficial spreading. lentigo maligna, and acral lentiginous melanomas.
1009'o

Generally, the radial growth phase is described by the A,B,C,D's of melanoma:

: asynmetry : border irregularity C : color variability (brolvtr,


A B

black, blue, gray, pink)

D = diameter of (olexl greater than 1/4 inch


Remember: Malignant melanoma is an uncommon neoplasm of the oral mucosa. It exhibits a definite predilection for the palate and the maxillary gingiva/alveolar ridge. Untortunately. oral mucosa melanomas have a dismal prognosis. The five-year survival rate for such tumors is < 20olo (/br skin lesions it i,s > 65%o).

Cancer ofthe tongue causes more deaths than do malignant lesions in other regions ofthe head and neck. This has been attributed to the fact that it is a highly mobile organ that is richly endorved rvith lymphatics and blood vessels which facilitate metastases. IntraorallY. the most

c'ommon sjte is the posterior lateral border of the tongue,

it is panicularly uncommon to

har e lesions develop on the dorsum or on the tip ofthe tongue. Remember: Metastases tiom ronsue cancer are relatively common at the time ofprimary treatment. In general. meiastatlc deposits from SCC ofthe tongue are found in the lymph nodes ofthe neck, usuaily on the ipsilateral side. The first nodes to become involved are the submandibular or j ugulodigastric

nodes at the angle ofthe rnandible.

Cancer ofthe lips: lip carcinomas account for 25 to 30% ofall oral cance$. Approximately 900 o occur on the lower lip due to increased sun exposure. Pipe smoking has also been imphcated in the etiology. Cancer of the lloor of the mouth: is the second most common intraoral location of SCC. lt occurs most comlronly in the anterior segment on either side ofthe midline, near the orifices ofthe salivary glands. Metastasis to submandibular lymph nodes is not uncommon for lesions tbund here. Note: Prognosis is very poor for lesions found here.

Cancer of the buccal mucosa: generally occurs along the plane of occlusion, midway anteropostenorly.

Cancer of the gingiva: is more common in the mandible than in the maxilla, and posterior
siles are seen more frequently than anterior

Note: Overall the 5-year suwival rate for SCC is 45% to 50%; with neck metastasis it is 25%.

ORAL PATHOLOGY

A 47-year-old

patient is referred frorn his physician to you because ofan exophytic growth in the maxillary lft mucobuccal fold. Unable to find an odontogenic source, a triopsy was done. A starry-sky morphology was found and a dismtl diagnosis of Burkitt's lymphoma was made. Which virus is thought to be responsible for this lymphoma?

IIIV*

. Herpes virus

. Epstein-Barr virus . Cytomegalovirus . Human papillomavirus

72
CopyriShr

lil20l

l l0l:

DenralDecks

ORAL PATHOLOGY

Neo

At the

age ofnine, poet Lucy Grealy was diagnosed with Ewing's srrcoms the jaw. Although rare in fernales, and rare in the jaw, Ewing's sarcoma most oftn presents radiographically as:

of

. Multiple

radiolucent/radiopaque lesions resembling "cotton ball" or "cotton wool"

appearance

. Multiple "punched-out" radiolucencres . "Moth-eaten" destructive radiolucencies ol medulla and erosion of the conex witlr
expanslon

. Lytic lesion that rnay

be ill-defined or sharply defined

73 CoDriShr C

20ll l0ll ' Dental

Decks

Burkitt's lymphoma is a high-grade, non-Hodgkin's lymphoma that is endemic in Africa and occurs only sporadically in North America. It is manifested most often as a
large osteolytic lesion in the jaw (African

fom)

or as an abdominal mass (Non-African

form).

Burkitt's llmphoma is the first human cancer with strong evidence of a viral etiology. The Epstein-Barr virus (t herpes-4tpe virus) has been isolated from cultures of tumor cells and patients with Burkitt's lymphoma have high titers ofantibodies against EBV Also, an
antibody against a surface antigen on the tumor cells has been demonstrated,

Note: The Epstein-Ban virus is also associated with infectious mononucleosis, oral hairy leukoplakia, and nasopharyngeal carcinoma.
Two forms of Burkitt's lymphoma:
1.

2.

African: younger (mean age 3), male predominance, typically involves the jaws. \on-African: older (mean age I I), no sex predilection, presents most often as an abBoth forms are histologically identical.

dominal mass.

***

jaw lesions usually present as expanding intraoral masses on the palate and ringir,a. Lesions appear as soft tissue nodular masses and many are hemorrhagic. See picture #37 in booklet. Radiographically, there is a moth-eaten, poorly marginated destmction ofbone. See picture #38 in booklet

\ote:

The

Eling's sarcoma is an uncommon, highly lethal, round cell sarcoma ofbone ofuncertain origin. The most common sites for Ewing's sarcoma are the pelvis, the thigh, and the trunk ofthe body. When the jaws are involved, there is predilection for the ramus ofthe mandible. Ninety percent of Ewing's sarcoma occur between the ages of5 and 30 years. and over 607o affect males. See picture # 39 in booklet
Pain. usually ofan intermittent nature, and swelling ofthe involved bone are ollen the earliest clinical signs and symptoms of Ewing's sarcoma. Involvement of the mandible ol maxilla may result in facial deformity, destruction of alveolar bone with loosening of reerh. and mucosal ulcers. Feveq leukocytosis, raised ESR, and anemia are also present.

Radiographically. the most characteristic appearance is that ofa moth-eaten dstructive radiolucency of medulla and erosion ofthe cortex with expansion. A variable periosteal "onion-skin" reaction may also be seen. See picture #40 in booklet Histologically, it is often difficult to distinguish this tumor from a neuroblastoma or a reticulum cell sarcoma, however, the cells of Ewing's sarcoma contain glycogn.
The highly malignant nature ofthis sarcoma is reflected in its propensity for metastasis, especially to lungs, other bones, and the lymph nodes. Multiple method treatment protocols. including surgery or radiation for local control and chemotherapy for systemic micrometastases have dramatically improved the formerly dismal l07o 5-year swvival rate to a 60olo 5-year suryival rate.

Important: The most common osseous malignancies are osteosarcomas, followed by


chondrosarcomas, fibrosarcomas and Ewing's sarcoma.

ORAL PATHOLOGY

Neo

A {ifteen-year patient presents to his physician because of localized pain in his right femur and rapidly enlarging swelling. A radiograph ofthe area shows a "sun-ray" apparance, Which is a likely diagnosis for this patient based on incidence?

. Chondrosarcoma . Osteosarcoma . Sclerodema


. Chronic osteomyelitis

74
Copyrighr..C

20ll':012

- Dental Decks

ORAL PATHOLOGY

Neo

While doing a dental mission trip in f,thiopia, you notice purplisb-brown nodules on th hard palate of a 32-year-old female patient. You notice more of these spots on her arms and legs. Given that HMnfection has an almost 59lo prevalence in urban Addis Ababa, what neoplasm are you suspicious ol?

. Nicotinic stomatitis

. Hemangioma . Kaposi's sarcoma . Leukemia

Cop).right ia:

20ll-:012 - Dentd

Decks

Osteosarcomas bl.so called osteogenic sdrcomas) account for approximatcly 20Yo of all sarcomas and, alier plasma ccll rnycloma, arc thc most common primary bone trnnors. Conventional ostcosarcomas involving thc rnandible and maxilla display a predileclion for malcs f629l,,/. Thosc arising in the jaws have a mean age of34 years. The majority (60o/o) ofnrandibular osteosarcomas arise in thc body ofthe mandible. In the maxilla, there is nearly equal incidcnce ol-tumors involving the laveolar ridge and maxillary antrum, u,ith feu, a1'ficcting the palate.
Osteosarcomas involving the mandible prcscnt most commonly with swelling and localized pain. ln some cases, there may bc loosening and displacement of teeth as well as paresthesia duc to involvement of the inl'crior alveolar nerve. Maxillary tumors display similar clinical symptoms but nlay cause paresthesia ofthe infraorbital nervc, epistaxis. nasal obstnlction, or cyc problems.
nray be characterized by localized widening ofthe periodontal ligament space of onc or two tccth. Advanced tumors can be visualized as moth-eaten radiolucencies or irrcgular. poorly marginated radiopacities. See picture #42 in booklet

Radiographicalln early ostcosarcomas

Osteosarcomas are best treated by radical mandibulectomy or maxillectomy, with radiothcrapy and chemotherapy for rccuncnces, soft tissue extension, or metastatic diseasc. As with most maLgnanl jaw tumors. initial radical surgery resulls in superior survival ratc of 807o compared with l'o D sunival lbr local surgery. Osteosarcomas ofthc jaws frcqucntly recur l/.;/0 lo 709'0), \Nith as n'relastalic rate of 25 to 50o/o. The most common sites ofmetastases arc lung and brain. \ote: Prognosis is better for mandibular tumors than lor maxillary tunors.

\ote: Chondrosarcomas arising in the mandible and maxilla are extren'tely mrc and havc countcd li'r ipproximately l9lo of all chondrosarcomas ofthe entire body. Cartilagc is produced by tumor .ells. The most common signs are a painless swelling and expansion ofthe affected bones, re:rltrng in loosening ofteeth or ill-fitting dentures. Radiographically, the lesions vary fiom mothlaten radiolucencies that arc solitary or multilocular to diflusely opaquc lcsions. The trealment is ir rdc Iocal or radical excision. Note: They are radioresistant.

Kaposi's sarcoma is a malignant ncoplasm ofcndothelial cells. It is characterized by abnormal vascular profiferation (it is o concer oftl1e lining of blood t'esselsi. It occurs on multiple sites, cspecially thc lo\ er extremities. lnitial lcsions are srnall, red papules, which enlargc and fuse to form purplc_to-broq'n. ipLrngl nodules. It spreads to lymph nodes and intemal organs. Note; Human herpes virus 8 has an eti-

oloclc role.
Kaposi s sarcoma is a uniquc form of angiosarcoma and is the most common cancer associated AIDS patients and has a predilection for the palate. Tbree different clinical patterns:

*ith

. Clas\ic lfediterranean type . Endcmic African type

'

mmunodeficiency rJpc (AIDS-tetuted)


Classic Type

Africeo Typc Afiica


Endenic Children and adul$ Extremilies

lmmunodeficienc! T\pe

Gcogmphy

Mediterranean basin
Rare

Relari\ely (ommon

Older nren Skin tesions Oml lesions Olher organs Coulse


Prognosis
Rare

Rare

Common Frequently

Occasionally Indolent

Occasionally
Prolonged

l:air

Fair

\ote:

Inrraorally. the hard palate is the most common localion, followcd by the gingiva and buccal muis

RmemberAIDS is caused by the RNA refovirus, HIV (d&o tdllel HTLV-lll). The HIV infection
acqnircd with IV drrrg use
as we1) as

by scxual conlact (homosexual and heterosexnal) and contaminated

blood Droducts.

. Lipoma . Ranula . Lymphoma . Oral lymphoepithelial cyst

76
Coplrighr O 201l-2012 - Dnral Decks

. They may be completely asymptomatic . The patient is usually aware ofslight discomfort or pain . The maxilla is affected far more frequently than the mandible . The molar region is predominantly involved

77 Coptright
@ 20 t 1,20

t2 - Denral

Deck

Almost all oral lymphoepithelial cysts arc lcss than 0.6 cm. in diameter at thc timc ofdiagnosis, which is usually during the teen years or the third decade oflife. The site most colnmonly affected is thc floor of the mouth, but the lateral and ventral tongue are not uncommon sites ofoccurrcnce. nor is thc soft palarc, espccially lhe mucosa abovc thc pharyngcal tonsil. This cyst has a clinical appearance similar to that ofan epidermoid cyst or a dcmroid cyst ofthc oral/phanngcal mucosa, but its growth potential is much lcss than ihc othcr cysts. Lymphomas arising rvithin the oral cavity account for less than 59/o oforal malignancies. In the head and ncck. most lymphomas start in the rcgional Iymph nodes or rvithin cxtranodal lymphoid sites in areas known as mucosa-associated lymphoid tissues (MALT h'nphonrur./. Within thc oral cavity. lymphoid tissue is chiefly represented in Waldeyer's ring fu ring oflvnphdtic tissue fonied br lhe tvo palatine tonsils. Ihe phanngeal tonsil,theli gual tonsil, dnd inteh^ening htnphoid tissre). Ofimponancc rclative to lymphoma is that in both nodal and extranodal sitcs, neoplasia can occur A ftrndamental aim is Io separatc lymphomas into two grollps:

. Ilodgkin's t)pe:

is charactcrizcd by painless enlargcmcnt of lymph nodes or extranodal tissue. \\'ithin Ihc oral cavity, tonsillar enlargement. usually unilateral. may be seen in the carly phascs. His-

tologicalll-, cominon to all forms ofHodgkin's diseasc is the prcscncc ofmalignant l)'rnphoid cells and non-ncoplastic inflamrnatory cells. including Iymphocytes, macrophages, eosinophils. and plasma cclls. Ofgreatest significance is thc identification ofthe Reed-Sternberg cell, which must be present fbr thc diagnosis of Hodgkin's disease to be established. Note: The Ann Arbor system of clinical stsging is uscd to determine the clinical extent ofthe disease and is an impofiant factor fbr dcciding !he tJ-pe and intensity oftherapy and helps dcterminc thc prognosis. . \on-Ilodgkins tlpei when primary oral soft tissues lesions are present (\'hich i.t rare). thcy are charactcnzed by an absence ofsymptoms and by a relatively soft charactcr, oftcn $'ilh overlying r.rl.erations. If bone is the primary site. alveolar bone loss and tooth mobility are oftcn prcscnting .rgns. S\!elling. pain, numbncss ofthc lip. and pathologic fracture may also be associatcd with bone lesrons. In general. thcrc arc t\r'o groups ofnon-Hodgkin's lymphona noddar (bllicular"/ and diffuse ibrms. For non-Hodgkin's Iymphomas. numerous classification schemes have cvolved. Thcsc in;lude the classifications ofLuke and Collins, the WHO, Rappaport, and thc NIH International Worklns Fonnulation.

***

This is false; the mandible is affected far more ftequently than the n.raxilla.

The most cornmon malignancy affecting skeletal bones is metastatic carcinoma. Howe\ er. netastatic disease to the mandible and maxilla is unusLra'l (onl.v about I %o). Most importantll, a tumor of the jaws may be the first evidence of dissemination of a kno$'n rurlrrr from its primary site. \letastases to the jaws most commonly originate from primary carcinomas of the breasr (ade o(arcinona), kidney (renal cell carcinona).lur'g (carcinona), colon (adetr()Ltr(i .) t.t). proslale (adenocarcinona) and thyroid gland (ode ocarcinonn), in decreasing order of fiequency.

\ote:

Clinical features of metastatic jaw lesions:

. Ofder adults (average age is


ceral neoplasm

561,ears)

. The mechanism ofspread to the jaws is usually hematogenous from the primary visthe jar.v, the angle and body of the mandible are more commonly involved Bone pain, loosening of teeth, lip paresthesia, bone swelling, gingival mass, and pathologic fracture may be clinically evident

. \\'ithin

. The radiographic appearance of mostjaw metastases is poorly marginated' radiolucent, irregular, moth-eaten, expansile defects

\ote: Microscopicalll',
noma cells.

r erified u,ith an immunoperoxidase stain

the diagnosis of metastatic carcinoma in difficult cases can be for cytokratin, which is present in all carci-

Important: The prognosis for patients with metastatic carcinoma to the jav's is grave, rvith a disrnal l0olo 5-year survival and over two-thirds dead within a year.

Neo

Explain what is meant by a carcinoma ofthe oral cavity having the following TNM designation:

Tl,

N2,

Ml

7A

Coptrighr e 2011 201? Denral Decks

ORAL PATHOLOGY

Neo

A 65-year-old patient ofEast Indian origin presents to your clinic with a complaint of a lesion on the maxillary alveolar tuberosity. The lesion is a thick white, exophytic mass with a cauliflower appearance. A social history reveals that this woman has been chewing a betel-nut concoction from her native India for over 40 years. \Yhat is the likely diagnosis of this lesion?

. Papilloma . Erythroplakia . Vemrcous carcinoma

. Hyperkeratosis

79
Cop)righr O

20ll 201?

Denral Decks

Clinical Staging of Carcinoma of the Oral Cavity


. T = Size of the primary tumor TX: Primary tumor can not be assessed TO: No evidence of tumor

Tis: Carcinoma in situ T1: less than 2 cm in greatest diameter T2: 4 cm in greatest diameter T3: greater than 4 cm in greatest diameter

. N = Regional lymph node involvement - NX: Regional lymph nodes can not be assessed NO: No clinically palpable lymph nodes, or lymph nodes palpable but metastases
not suspected Nl: Palpable homolateral lymph node(s), not fixed but metastases suspected N2: Palpable centralateral/bilateral lymph node(s), not fixed but metastases suspected N3: Palpable lymph node(s), fixed metastases suspected

\I

= Distant metastasis

- !tX: Presence of distant metastasis can not be assessed lfo: No distant metastasis fI1: Clinical and/or radiographic evidence of metastasis other than
ll mph nodes

regional

-\ r'errucous carcinoma is a well-differentiated squamous cell neoplasm ofsoft tissue of the oral or laryngeal cavity. The lesion may invade or infiltrate the borders of adjacent structures but it rarely metastasizes. Verucous carcinoma may transform into an invasile fom of carcinoma or coexist with other squamous cell carcinomas. It is often
misdiagnosed histologically as a benign lesion.

Clinical Features: . \{ale predilection . Broad-based, exophytic, indurated lesion . Diffuse, whitish, cauliflower or coral papillary mass . \{andibular mucobuccal fold, alveolar mucosa and palate . Slo*-growing, continuous enlargement . Painless
See

picture

#,13

in booklet

\ote: Tobacco and human papillomavirus


tors. It

(subtypes 16 and 18)may be etiologic fachas a good prognosis. is treated by surgical excision and

Neo

A 73-year-old patient prsents to your office with complaint of a lasting ulceration on th right side of his tongue. H has a history olhypertension and high cholesterol and a 40-year pack history of srnoking. The patient takes anti-hypertensives and anti-lipidemics and has no allergies. You cannot lind any sources oftrauma. After two weeks, the ulcer has grown in size. What is the likely diagnosis of this most common malignancy ofthe oral cavity?

. Adenoid cystic carcinoma


. Mucoepidermoid carcinoma

. Basal cell carcinoma

Squamous cell carcinoma

80 CoDrighr
Lq

20ll l0ll

Denral Decks

ORALPATHOLOGY

Nrv & Mus Disord

While attempting to give in inferior alveolar nerve block, ifyou inject the anesthetic solution into the capsul of the parotid gland, you may cause a Bell's palsy like feeling for the patient by anesthetizing the:

Triuern

inrl newe

. Glossopharyngeal nerve . Hl.proglossal nerve

. Facial nen'e

81 Copynghr C 201 l,:01 2 - Dentat Decks

Squamous cell carcinoma is a malignant neoplasm ofstratified squamous epithelium that is capable oflocally destructive growth and distant metastasis. It is the most common type oforal cancer, accounting for over 90%o ofall malignant neoplasms ofthe oral cavity. lt is two times more prevalent in m ales (40-65 years of4ge./. See pictures #44 and #45 in booklet.

Possible sites: . Lower lip (most common site) . TottgU.e (posterior leteral border is the most common loccttion, dorsum least common locqtiotr) . Floor ofthe mouth (least favorqble plognosis) . Sofl palate (u conmon) . Gingiva/alveolar ridge

Buccal mucosa

Clinical fatures: . Early presentation ofleukoplakias and erythroplakias . Painless ulcer, tumorous mass, or verrucous (papillary gro\tth) . Occasional loosening or loss ofteeth . Possible paresthesia ofthe teeth and lower lip
Risk f'actors identified include smoking, alcohol consumption, painful and ill-fitting dentures, chronic inflammation and the use ofsmokeless tobacco. Important: Tobacco use is the primary risk factor Note: Causes ofgenetic alterations which results in loss ofcell cycle control. abnonnal signaling, increased cell survival, and cell motility include tobacco, human papillomavirus (subq,pes 16 and l8), and heredity. Remember: The most reliable histologic criterion for a diagnosis oforal squamous cell carcinoma is invasion,

Bell's palsy is a form offacial paralysis resulting from damage to the facial nrve.It can strike at any age; however, it disproportionately attacks pregnant women and people who have diabetes, influenza, a cold, or some other upper respiratory ailment.
a unilateral paralysis ofall facial muscles with loss ofeyebrow and forehead wrinkles, drooping ofthe eyebrows, flattening ofthe nasolabial furrow, sagging of the comer of the mouth and the inability to frown or raise the eyebrows. The upper and loler lips may also be paralyzed on the side affected.

Clinical signs include

.{tier its sudden onset the paralysis begins to subside within two or three weeks, and gradual. complete recovery occurs in over 85yo of patients.

Triggering events related to Bell's palsy are acute otitis media, atmospheric pressure change. exposure to cold, ischemia of the facial nerve near the stylomastoid foramen, \lelkersson-Rosenthal syndrome, and multiple sclerosis. Note: Melkersson-Rosenthal slndrome is the term used when cheilitis occurs with facial palsy and plicated tongue.

A 2s-year-old college student comes into your olfice cornplaining that when she wakes up she has trouble opening her mouth. When conducting a TMJ exam, you note tenderness ofthe right latral pterygoid and non-reciprocal clicking of the right TMJ. What is the most likely cause of the patient's myofacial pain?

. Trauma . Muscle spasm . Periodontal disease . Tumor

a2
Copyrighr O

20ll'2011-

Dental Decks

ORAL

PATHOLOGY

Nrv & Mus Disord

A S3-year-old patient comes to your o{Iice and notes that sometims when he swallows, he gets a sharp 'jolt" on the right side of his throat. He says the pain is seyre and he can even feel it in his ear. The most likely diagnosis is:

. Postherpetic neuralgia
. Orolingual paresthesia

. Frey's syndrome . Glossopharyngeal neuralgia

83 Coplriglrt C

20ll ?012

Denral Decks

Such spasm may arise in one ofthree ways: muscular overextension, muscular over- contraction or muscle fatigue. The most frequent cause ofthe spasm seems to be muscle fatigue. This syndrome is seen predominantly in women, usually in the 20 to 40 age range, and generally occurs unilaterally.

There are four cardinal signs and symptoms ofthe syndrome:


1. Pain 2. Muscle tendemess 3. A clicking or popping noise in the TMJ 4. Limitation in jaw motion (especially in the morning)

The pain itself is usually unilateral and is described as a dull ache in the ear or preauricular area, rvhich may radiate to the angle ofthe mandible, temporal area or lateral cervical area. The muscle most apt to exhibit tenderness is the lateral pterygoid muscle.

\ote:
droDte)

There are no radiographic findings associated with MPS (mtofacial

pain syn-

Treatment: Most cases are selflimiting. Soft diet. limited talking, no gum chewing, moist heat. NSAIDs and Diazepam help relieve symptoms.

Glossopharyngeal neuralgia refers to pain similar to that of trigeminal neuralgia, which arises frorn the glossopharyngeal newe (CN IX).lt is not as common as trigeminal neuralgia. but the pain may be as severe when it does occur. It occurs in both sexes, in middle-aged or olcler persons and is described as sharp, jabbing, electric, or shock-like pain located deep in the throat on one side. lt is generally located near the tonsil although the pajn may extend deep into the ear. It is usually tiggered by swallowing or cherving. It is
almost ahvays unilateral.

Frel 's sl ndrome (also called the auriculotemporal syndrome) ts an unusual phenomenon. * hich arises as a result of damage to the auriculotemporal nerve and subsequent reinnen ation of the sweat glands by parasympathetic salivary hbers.The syndrome follo* s some surgical operation such as removal ofa parotid tumor or ramus ofthe mandible, trr an infection of the parotid that has damaged the auriculotemporal newe (a branch of ---l I /.

ofFrey's syndrome. The patient rypically exhibits flushing and sweating of the involved side of the face during eating.

lmportant: Gustatory sweating

is the chiefsymptom

This syndrome is not a common condition. Postherpetic neuralgia is a persistent burning, aching, itching and hyperesthesia along distribution of a cutaneous nerve following an attack ol herpes zoster. It may last for a ferv rveeks or many months. lnvolvement of the facial nerve and geniculate ganglion produces the Ramsey Hunt Syndrome, which is characterized by facial paralysis and
otalpia karache).

ORAL

PATHOLOGY

Nrv & Mus Disord

Which of the following is a relatively rare autoimmune disorder of peripheral nerves in which antibodies form against acetylcholine (ACh) nicotinic postsynaptic receptors rt the myoneural junction?

. Myasthenia gravis

. Myelofibrosis
. Multiple sclerosis

. Graves'

disease

E4

Copynghr C

201

l'201 2 - Dental Dects

ORALPATHOLOGY

Nrv & Mus Disord

A 34-year-old dentophobe is your patient for the morning. After giving a very hesitant health history, you decide to begin your oral exam, As you reach towerd her face, she immediately flinches and puts her hands up. She lets you know that ifyou tough a particular point above her lip, she gets sharp, stabbing jolts ofpain. You let her know that a neurologist can work her up for:

. Glossopharyngeal neuralgia

. Trigerninal neuralgia
. Postherpetic neuralgia

. Diabetic neuralgia

Copy.ighr (]

20ll ?011

Denral Decks

Myasthenia gravis is an autoimmune disorder in which antibodies form against acetylcholine (ACh) nicotinic postsynaptic receptors at the myoneural junction. The muscles are quickly iatigued with repetitive use. lt is tlpical for a myasthenic patient to have a flattened srrile and droopy eyes. with slow papillary light responses. Xerostomia and rampant caries may accompany myasthenia gravis. The acetylcholine that is necessary for the proper transmission ofnerve impulse is destroyed, with the result that salivary glands do not receive adequate stimulation. Note: Head and neck manifestations include inability to focus eyes, drooping eyelids, double vision, difficulty in chewing and swallowing, and slurring ofwords.

Multiple sclerosis is a chronic, often disabling


lhrain
ancl spinal cord).

disease that randomly attacks the CNS

It is believed to be due to an autoimmune response in which the

inxnune system attacks a person's own tissue. Twice as many women as men have MS, s ith the onset of symptoms occuring most often between the ages of 20 and 40. Symptoms may range from tingling and numbness to paralysis and blindness. Patients with multiple sclerosis sometimes have facial andjaw weakness. In addition, both Bell's palsy and trigeminal neuralgia may develop more frequently in patients with MS.

Trigeminal neuralgia is an excruciating painful illness in which the afl-licted f'eels sudden stabJike pains in the face. The pains usually last only moments, but are among the most severe pains humans can feel. The pain is provoked by touching a "trigger zone," n picallv near the nose or mouth. lt is caused by degeneration ofthe trigeminal nerve or
b1 pressure being applied to it. Pain distribution is unilateral and follows the sensory dis-

tribution of cranial nerve V, typically radiating to the maxillary (I'-2i or mandibular / I --l/ area. At times, both distributions are affected. The momentary bursts ofpain recur in clusters. lasting many seconds. Paroxysmal episodes ofthe pains may last for hours.
\eccssary criteria for diagnosis of trigeminal neuralgia: . Onser ofpain initiated by a trigger point . Pain extreme. paroxysmal, lanclnatlng . Durltron is less thrn 2 minutes . -{ refractory period experienced for several minutes after attack . Pain limited to krown distribution of one or more branches of trigeminal nerve with no motor deficit in that area . Pain diminished with use ofcarbamazepine

Spontaneous remissions occur lasting tnore than six months during the early phase the disease

of

The dnrg ofchoice for treating trigeminal neuralgia is carbzmazepine (Tegrelor. lt is an analgesic and anticonvulsant. It is also prescribed in the treatment ofcefiain seizure dis-

orders. When used for the trcatment of trigeminal neuralgia, it usually rclieves the pain u'ithin 48 hours aftcr treatment is startcd.

. Median alveolar cyst . Globulomaxillary cyst


. Nasolabial cyst
. Nasopalatine cyst

86

Copliiglt

O 20l l-2012 -

Dotal Deck

Afl ofthe followlng cysts are congenit EXCEPT one. Which one is the EXCI'PIIOM

. Thyroglossal duct cyst

. Branchial cyst . Globulomaxillary cyst . Dermoid cyst

87
CopynShr O 20ll-201? - Dental Decks

The nasolabial cyst has been called, inaccurately, nasoalveolar cyst. This latter designation is inappropriate because the entity is not a true cyst ofthe maxilla. Rather, it represents a soft tissue cyst without involvement ofthe alveolus, hence the prelerence for the designation "nasolabial cyst." Note: Because thjs cyst is extraosseous, it is not likely to be seen on a radiograph.

Ouick reference for develoomental cvsts:


Median AlYeolar - IAIE

- teeth vital

*Nasolabial
- in sofi tissue

Nasopalatine - most common


- heart shaped Hard Palate

lI
I

Median Palatal
- rare

Thc thl roglossal duct cyst, which may arise fiom any portion of the thyroglossal duct. This is therefore lound in a midline position and is usually dark in color It may bc vascular as to rcsenrblc a hemangioma. One lrequent important symptonr is hemorrhage into the mouth, resulting from thc rupture of the overlying veins. Complctc cxcision of the tracl to the base of the Iongue. frequently including a portion ofthc hyoid bone, is nccessary tbr a curc. Note: Whcn mirration ofthc thyroid gland lails and the gland rernains in thc basc ofthe tongue. it is called lingual thl roid or eclopic lingual thyroid. Lingual thyroids are lour tjmes more comnon in females than in rrales.
cy st

. The branchial /.1eli/ cyst or cervical lyrnphoepithelial cyst is located in the latcral portion ofthe ncck. usuallv anterior to the sternocleidomastoid muscle. Thcy may also appear in the submandrbLrlar area. adjaccnt to the parotid gland. or around lhc stcmocleidomastoid muscle. The majorily rr: lhese t-vpes arise ftom remnants ofthe second branchial cleft or pouch. This cyst is lincd with .rlified and slratified squamous epithelium and conlains a l11ilky or mucoid fluid. The treatment aon5isrs ofconlplele surgical excision. Note: The branchial cyst has an intraoral countcrpart known a: Ihe lt mphoepithelial cyst. The floor ofthe moulh is the most common site lor thcse lcsions, follo\\ ed bv the tongue
is rclativcly uncommon in the oral cavity. This cyst frequcntly contains hair, \ebaceous and sweat glands, as well as tooth structures. The most common site is thc midline of the floor ofthe mouth ifabove the mylohyoid muscle.lt appcars as a mass in the upper neck if belol\ the mylohyoid muscle. The treatment is the surgical removal ofthe entirc tulnor

. Thc dermoid cyst

Important: Globulomaxillary cysts are thought to dcvclop fiom epithelial rcmnants remaining tbllos ingjoining ofthc globular portion ofmcdian nasal process with thc maxillary process. They arc characterized by a large "pear-shaped" radiolucency between the maxillary lateral incisor and
cuspid. Thcy arc asymptomatic, all regional teeth are vital. They tcnd to cause divergence ofthc roots. However, thcrc is considerable controversy as to whether this cyst actually exists. Many of these are, in reality apical cysts associated with non-vital lateral incisors. Olien thcsc apical cysts havc a tcndcncy to cxtcnd betwccn thc two tccth to simulatc a globulomaxillary cyst.

All of the following cysts are developmenttl (or ftssural) EXCEPT one. Which one is the EXCEPTIOM

. Nasopalatine duct (canal) cyst


. Nasolabial (nasoalveolar) cyst . Branchiogenic cyst . Median palatal cyst

. Median alveolar cyst

88
Copynghr

q 20ll'2012

- Dental Decks

ORAL PATHOLOGY

N-O Cysts

The circular radiolucent area seen in this radiograph is clinically seen as a marked swelling in the region of the palatine papilla. It is situated mesial to the roots of the central incisors. The pulps of the anterior teeth in this patient tested yital. These findings would be compatible with what diagnosis?

89
Copyrighr C 2011'2011- Dental Decks

. Nasopalatine duct (canal): "hearrshaped" radiolucency in midline, most frequent


type of nonodontogenic cyst. Usually asymptomatic or may produce an elevation in the anterior part ofthe palate.Teeth are vital, Treatment is enucleation. Note: It is caused by cystification ofthe nasopalatine duct remnants. Remember: It is also known as incisive canal cyst and may be located within the nasopalatine canal or within the palatal soft tissues at the point ofthe opening ofthe c anal (in this loccrtion it is called a cyst ol the palatine papilla).

. Nasolabial (nasoalveolar): is superficially located in soft tissues of the upper lip. This
is an extraosseous cyst. Treatment is surgical excision.

. Nledian palatal: rare, may occur any'rvhere along median palatal raphe. May produce
elling on palate. Treatment is enucleation. Note: Many oral pathologists now believe that this cyst represents a more postedor presentation of a nasopalatine canal cyst .
s$

. \Iedian alveolar: rare. occurs in bony alveolus between central incisors. Distinguished liom periapical cyst by the fact that adjacent teeth are vital. Treatment is enucleation.

Of the cysts ofthejaw, those that arise from epithelial remnants in the incisive canal are the most common type of maxillary developmental cyst. Histologically, this cyst is lined rvith vessels, nerves and mucous glands in the wall. They most often remain limited as to size and are asymptomatic. Some of them, however, become infected or show a tendency to grow extensively. When tl'tis occurs, surgical intervention is indicated.

\ote:

The soft tissue (ancl.far less common)

v^ri^nt of the nasopalatine canal cyst

is the

cr st of the palatine papilla.

\\'hen making

a diagnosis

ofthis cyst, the following two cysts should be ruled out:

. The globulomaxillary cyst: usually appears between the roots of the lateral incisor and those ofthe canine. lt is "pear-shaped" and often causes the roots ofinvolved reerh to diverge. See picture f87 in booklet
. The median palatal cyst: usually situated in the midline of the hard palate, posterior to the premaxilla. Clinically, Ihis lesion presents as a firm swelling, which is usually painless. Note: Some investigators now believe that this cyst represents a more posterior presentation ofa nasopalatine duct cyst, rather than a separate cystic degeneration ofepithelial rests at the line ollusion of the palatine shelves. See picture ff48 in booklet

The soft tissue, and far less common, varlant the nasopalatine canal cyst is the:

of

. Median mandibular cyst . Nasolabial cyst

. Cyst ofthe palatine papilla


. Aneurysmal bone cyst

90

Coprighr O 2011,2012, DeninlDecks

An &year-old girl who looks like she is 14 years old comes with her fathor into your ollice. Her frther states during the health history that she hNs Mccune-Albright syndrome. Which of the following would you NOT EXPECT the p.tient to have?

. Heart

disease

. Polyostotic fibrous dysplasia . Cafe-auJait spots

. Endocrine dysfunction

91

Coplright O 201 I -20 l2 - Denral Decks

The nasopalatin dvct cyst (NPDC) is a developmental non-odontogenic cyst that only occuIS in the anterior maxilla from the embryonic remnants ofnasopalatine ducts. lt is usually located between teeth #s 8 & 9 but can be as posterior as the mid-palate and as anterior as the incisive papilla. It is the most common non-odontogenic oral cyst of the oral caviry lt occurs at any age, but is most common in the fourth to sixth decade oflife and is more common in males. It is usually asymptomatic and is discovered during routine dental examination. If infected, the patient complains ofpain with swelling and even drainage; however, swelling is not common. When present, swelling is often in the incisive papilla area in smaller lesions but can be in the mid-palate in larger lesions and may be buccal and mid-palatal in very large lesions. Radiographically it usually presents as a bilateral, well-circumscribed, round, ovoid or heart-shaped radiolucency. Tooth displacement or diverging ofthe roots ofthe central incisors is commonly identified. Depending on the size and clinical symptoms, treatment ranges tiom no treatment to surgical curettage ifthe cyst is infected or interlering with a prosthetic appliance. Recurence is rare and prognosis is good. Note: The cyst of th palatine papilla is identical to that of its intrabony counterpart.
The median mandibular cyst, like the globulomaxillary cyst, was once considered a fissural c.vst. Ho$'ever, embryonic evidence rcfutes thrs (there are no epithelial lined processes).It is a \ ery rare cyst that appears in the midline ofmandible. Most of them are periapical, lateral periodontal cysts or odontogenic keratocysts.

Remember: The nasolabial cyst has been called, inaccurately, nasoalyeolar cyst. This lar ter designation is inappropdate because the entity is not a true cyst ofthe maxilla. Rather, it represents a soft tissue cyst without involvement ofthe alveolus, hence the preference for the designation "nasolabial cyst." Note: Because this cyst is extraosseous, it is not likely to be
seen on a radiograph.

-\lbright's syndrome (also cqlled McCtne-Albright s-vtdrcrre/ is the most severe form of poll-ostotic librous dysplasia. It affects young people (males andJbmales equally). It
is characterized by Caf-auJait spots on the skin, and endocrine abnormalities (the most conunon ol u,hich is precocious sexual development infemales). The extent to which each ofthese problems exist in those with the syndrome is quite variable. The hallmark of Albright's syndrome is premature puberty in the female. Early sexual development in the nrale is less comnon than the female.

Clinical features: . Early childhood . \{ultiple, slow-growing, painless expansile bone lesions confined to the craniofacial
area or throughout the skeleton

. endocrine manifestations; in females often sexual precocit-v .lrregular shaped Cafe-aulait spots on the torso and sometimes intraorally . Disfiguring . Increased level ofserum alkaline phosphatase

*** Pathologic fractures

are frequently associated with this syndrome.

There is no specific treatment for this syndrome. Drugs that inhibit estrogen production, such as testolactone. have been tded with some success.

Important: An additional complication is the malignant transformation potential of


both the polyostotic (nainly) and monostotic fibrous dysplasia into osteosarcomas.

weeks. There are no

An &year-old chlld has complained of pain on the left sido ofhis head for 5 abnomal findlngs on physical examination. A prnoramlr radiograph reveals multiple radiolucent lesions on the left side ofthe moxillo. The lesions give ahe appearance of teeth thot rre "floating in space." Tbe lesions are sharply circumscribed, with a punched-out appearance. Which of the following is the most likely di.gnosis?

Langerhans cell disease

. Hyperparathyroidism . Cherubism . Paget's


disease

92
Cop)rishr O 201I -2012 - Dental Decks

. Is a common lesion and appears more frequently than does the peripheral giant cell granuloma

. Is found predominantly in children


. Affects males more than females

and young adults

.Is

present almost exclusively in the small bones

ofthe hands and feet

93
CoDright A 20ll-2012 - Dental Decks

ldiopathic histiocytosis or Langerhans cell disease, also formcrly known

as

histiocylosis X,

is a disorder charactcrized by a prolifemtion of cells exhibiting phenotypic charactcristics of Langerhans cells, The clinical manifestations of this process range from solitary or multiplc bone lesions to disseminated visccral, skin, and bonc lcsions.

Traditionally, idiopathic histiocytosis has been used to encotrlpass thrcc disorders: eosinophilic granuloma, Hand-Schullcr-Christian syndromc, and Lellcrcr-Si$,e disease. . Eosinophilic granuloma (cltronic loculized Jbnn) has refened to patients with soJitary or rnultiplc bonc lcsions only . Hand-Schuller-Christian syndrome (chronic listelnitvl?d lbrn.) has represented a specific clinical triad of lytic bone lesions, exophthalmos, and diabetes insipidus. Many ol thesc paticnts also cxhibit lymphadenopathy, dermatitis, splenornegaly, or hepatomcgaly. . Letterer-Silve disease (.rcute dissemitvted fonn) has been charactcrizcd by a rapidly progrcssir e. usually latal, clinical coursc. Widcsprcad organ, bonc, and skin involvement by the prolifLrdti\e lrocc:'s rn Inl'anls has been the comnton presenlalion.

ldiopathic histiocytosis
changes may bc the
a

is generally regarded as a condition

ofchildren and young adults. Oral

initial presentation in all fonns ofthe disorder. Tenderness, pain and srvelling arc liequcnt patient complaints. Loosening ofthe teeth in thc arca ofthe affected alvcolar bone is
conlnlon occulrcncc. Thc gingival tissues are frequently inflamed, hypcrplastic, and ulceraled.The ja\\ s mal exhibit solitary or multiple radiolucent lesions. The lesions frcqucntly ali'ect the alveolar bonc. rcsu)ting in the appearance of teeth that arc "floating in space." Bone lcsions rvitb a .h:rrpiy- circumscribcd, punched-out appearance may also occu. in the central aspects of the :randiblc or naxilla.

\licroscopicall!, eosinophils
rnuc

are mixed with thc tumor Langcrhans cells. some

ofwhich

arc

mult-

lci

d.

Prognosis is Very good when the disease is localized, however, the disseminated form is usually
:...te1.

The central giant cell granuloma is a benign process that occurs almost etclusively within the jaw bones. The tumor typically prescnts as a solitary, radioluccnt lesion ofthc mandible or maxilla- It is an uncommon lesion and occurs less frequently lhan does the peripheral giant cell granuloma. It is loL)nd prcdominantly in children and young adults. Fcmalcs are atltcted lnorc frcquently than males. Lcsions occurmorc frequenlly in thc mandible than in themaxilla. These lcsions tend to involvc thcjaws entarior to thc molar teeth. Thc ccntral giant coll graiuloma typically produces a painlcss cxpansion or

.\\ elling ofthc affected jaw (-entral giant cell granuloma is classified into aggressive and non-aggressive types; the aggressrvc

lre

tends to occur in younger patients and is known to cause disfiguration, especially after surgcry.

Radiographically, it consists ofa multilocular or, less frcqucntly. unilocular radioluccncy ofbonc. The r::rgins of the Iesion are relatively well demarcatcd. often presenting a scallopcd border. Roots of the i3.th nlay br displaced and. less commonly, rcsorbed. See picture #,ll in booklet

Important: Thc lcsion is composed ola proliferation ofspindled libroblasts in a slroma contain ing \ ari.rb.c anlounts of collagen. N{ultinucleated giant cells are ptesent lhtoughout the conneclivc tissuc

\otei Thc differential diagnosis includes .l !t. and aneurysmal bone cyst.

amcloblastoma. odontogenic mlxoma, odontogenic kerato-

Thc usual treatment is surgcry however. treatment altematives to surgery have emcrged with success_ 1u1 results ranging from stcroid injections to calcitonin injcctions or nasal spray to interferon alpha-2a injections . \\,hich are administered 2-J times pcr rvcck for several months.

. \otei' . --' .

L The microscopic appearance ofccntral giant ccll granulona is vimrally identical to ihe giant cell lesion /B/?rtri /rrrol/ associated lvith hyperparath] roidism. However. blood tests \r'ill shotv an increase in serum calcium and alkaline phosphatase and a decrcasc in serum phosphorus in hypcrparathyroidism. 2. The giant cell tumor of bone may present with similar clinical and mjcroscopic features. However, the Siant cell tunor is regardcd as rare in thejaws in comparison to the cenffal giant ccll granuloma. They occur most frequently in long bones. See picture #96 in booklet

ORAL PATHOLOGY

N-O Tirm

A 21-year-old male patient is home from colleg and came to you becaus his "bite seems off " A quick physical assessment seems to indicate that his chin is deviated to the right. Taking a panoramic x-ray and comparing to the previous panoramic radiographs you have in his chart, you notice that the lft condylar neck seems to have elongated. What condition does this patient most likely have?

. Condylar

agenesis

. Condylar hyperplasia . Condylar hypoplasia

. Hemifacial microsorria

94 Coplaight
aq

2011-:01l - DenitlDecls

ORAL PATHOLOGY

N-O Thm

A 6-year-old boy is a patient in your practice who has been hospitalized multiple times for broken bones. During routine lab tests, it was noted that his alkaline phosphatase levels were quite high. Dentally, you notice multiple impacted teeth. If this patient has a form offibrous dysplasia, which radiographic feature may you lind?

. The lesions are usually radiolucent, well-circumscribed, and may have a "cotton wool"
appearance

. The lesions

are usually a saucer-shaped radiolucency

. The lesions

are usually radiopaque, not well-circumscribed, and may have a "groundglass" appearance

. The lesions are usually well-demarcated unilocular or multilocular radiolucencies


95
Copyrighr'-C 2011

20ll

- Denlal Decks

Congenital and developmental anon.ralies ofthe temporomandibularjoint, although relatively rare, are important to identiry early to reestablish normal midface growth centers. The more common entities include condylar agenesis, condylar hypoplasia, and condy-

lar hyperplasia.
Condylar agenesis is the absence of all or porlions of the coronoid process, condylar process, ramus and mandibular body. Other first and second arch abnormalities are commonly seen. Early treatment is indicated to limit the degree of deformity, with the primary objective being to re-establish the condylar growth center. This is best done with a costochondral graft with or without orthodontic surgery and facial plasric augmentation.
Condylar hypoplasia may be congenital, but is usually the result of trauma or infection. The most common facial deformity is shortness of the mandible with deviation of the chin towards the affected side. Treatment ofthe child involves the placement ofa costochondral graft. In the adult, treatment involves either shortening of the normal side or lengthening ofthe involved side. Both result in an acceptable cosmetic and functional result. Onhodontic therapy is necessary in all cases to establish proper occlusion. Condylar hyperplasia is an idiopathic disease characterized by a progressive, unilateral o\ergro\rth olthe mandible. The chin is deviated towards the unaffected side. Presentation is common in the second decade. Radiographic findings are usually a normal condlle but an elongated neck- Treatment depends on whether the condyle is still growing. lts growth is occurring, condylectomy is the treatment- If growth has ceased, orthognathic surgery is performed.

Fibrous dysplasia is an idiopathic condition in which normal mcdullary bone is gradually replaced by
an abnormal fibrous connective tissue proliferation. The mesenchymal tissue contains varying amounts

of ostcoid and osseous material that prcsumably arises through mctaplasia.

The discase most commonly presents as an asymptomatic, slow enlargement of thc involved
bcrne.

It may involve one bone or several boncs concomitantly.

Forms of fibrous dysplasia:

. \Ionostotic: is the most common, comprising 70o% ofcases. There is an equal male to fcmalc ratio, and this fclrm is the most likely to quiesce at puberty. A t!?ical monostotic lcsion will involve thc femur tibia. or ribs, uith 25% occuring in the bones ofthe skull. See picture #49 in booklet . Pol] ostotic fibrous dysplasia applies to cases in rvhich nrore than one bone exhibits evidcncc ofthc disorder. It is relatively uncommon; however, many patients have lesions of the skull, facial bones, t r.ia$ s. as a componcnt ofthe condition. Note: Alkaline phosphatase may bc elevated in up to 30% oipatients $ith polyostotic fibrous dysplasia, and a dramatic dse may herald malignant degeneration.
Calcium and phosphorus tcnd to be normal.

\otes-

l. Craniofacial librous dysplasia is a form offibrous dysplasia charactcrized by maxillary lesions that extend to involve the maxillary sinus, zygoma, sphcnoid bone. and floor ofthe
orbit.

Itfccune-Albright syndrome is a designation that has been applied to patients with polyostotic fibrous dysplasia, cutaneous melanotic pigmentation fcafe au lait macules), and endocrine abnormalities (most commonlr precocious sexual development infenales). 3. Jaffe-Lichtenstein syndrome has been used to describe patients with multiple bone lesions offibrous dysplasia and skin pigmentations. 4. Osteoblastoma is an uncommon primary lesion of bone that occasionally arises in the rnaxilla or mandible. It is a benign process that may exhibit a seemingly rapid onset and cause pain. The mandible is the most liequent head and neck sitc. Young adult malcs arc most commonly affected. Radiographically, is a well-circumscribed lesion that varies fiom radiolucent to radiopaque. May have a "sun-ray" paftcm ofnew bone production.

. In the retromolar pad region

. On the lingual surface ofthe mandible, most often in the premolar region . On the lingual surface ofthe mandible, inferior to the mylohyoid ridge
. Along the midline ofthe hard palate

96

CopriglrtO 20ll-2012

- Dnral

D4ks

All of the following are clinical features of the ossifying


fibroms EXCEPT one. Which on is the EXCEPTION!

. Slow growing expansile lesion


. More often in maxilla . Asymptomatic . Common in young adults around 35 years ofage

97
Cop}.right O 201 l-2012 -

De

al Decks

Tori and exostoses are nodular protuberances of mature bone whose precise designation depends on anatomic location. These lesions are of little clinical significance; they are non-neoplastic and rarely are a source ofdiscomfon. The precise etiology ofthese lesions remains obscure, although evidence has been presented to suggest that the torus may be an inherited condition. Note: The etiology ofexostoses is also unklown. It has been suggested that the bony growths represent a reaction to increased or abnormal occlusal stresses ofthe teeth (i.e., bruxism) in the involved areas. Mandibular tori (also called torus mandibularis) are bony. exophytic growths that occur along the lingual surface ofthe mandible superior to the mylohyoid ridge.

Vandibular tori may occur singly, however, there is a marked tendency toward trilateral occurrence, and the lesion is not necessarily confined to the premolar region. Unlike palatal tori. the mandibular tori are more readily demonstrated radiographically.

forr huoxillary

and mandibulor) are ofno pathological significance and rarely are they erf clinical significance r.vhile the normal teeth are still present. If, howeyer, a complete denture needs to be made, they should be carefully removed.

***

This is false; these lesions, with rarc cxccptions, arise in tooth-bearing regions ofthc iau s. most ofien in the mandibular premolar-molar area. See picture #89 in booklet Thc ossifling libroma is a bcnign fibro-osseous lesion of the jaws that is considercd by nran\ in\estigators to dcvclop fiom undiffercntiated cells ofthe periodontal ligament. The :imilaritics between this lesion and thc ccmcntifying fibroma are numerous. Both tumors occur in similar age groups and locations and manifest comparablc clinical characteristics.
The ossif-ing fibroma is a slow-growing, expansile lesion that is usually asymptomatic when d:scorered. Thcsc lcsions, with rare exceptions, arise in tooth-bcaring regions ofthejaws, most oflen in the mandibular premolar-molar area. They are uncommon lesions that tend ro occur in fcmales during the third and fourth decades of life. The most important radiographic feature ofthis lesion is the well-circumscribed radirrluccncy \r'ith a sharply defined border Thcy prcscnt a variable appearance dcpcnding on thc maturation or thc amount olcalcification present. As thc lcsion matures, it eventually becomes a relatively uniform radiopaque mass. The tumor consists ofa collagenous stroma that contains varying numbers ofunifbrm spindled or stellate cells. Irregular trabeculae of woven immature bone are most consistently rored in thse tumors. The treatment ofossifying fibroma is most often accomplished by surgical removal utilizing curettage or enucleation. Recurrence is uncommon.
has been described in children and teens. This rare lesion behaves in a more aggressive fashion than does the ossifliing fibroma, and it may require more extensive therapy when encountered.

.\ r ariant of ossifoing fibroma, thejuvenile ossifying fibroma,

A l?-year-old patient ofyours comes in for a routine examination. A head and neck examinrtion reveals multiple cysts of the skin. Her panoramic exarns have always shown multiple impactd teeth and today shows multiple radiopacities ofthe jaws especially at the angle of the mandible. You suspect Gardneris syndrome. What complication should she most be concerned with whn consulting her physician?

. Odontomas
. Osteomas

. Adenocarcinoma . Epidermoid cysts


. GI polyposis
9E Copyfl ghr
a., 201

1,201 2 - Dental Decks

ORAL PATHOLOGY

N-O Tirm

A l0-year-old boy comes with ber mother to the dental office because of a painless swelling ofhis maxilla. Radiographic exam reveals an irregularly shaped radiopoque mass with a ground glass appearance. No other bulges have been noted by the mother. A biopsy reveals fibrous tissue in the bone. What is the most likely diagnosis?

. Monostotic fibrous dysplasia . Polyostotic fibrous dysplasia


. Albright's syndrome

. Jaffe syndrome

99
Copyrighr,(] 20l

l'2011

Dental Decks

Gardner's syndrome is an autosomal dominant disorder and is characterized by intestinal polyposis, multiple osteomas, fibromas ofthe skin, epidermal and trichilemrnal cysts, impacted pemanent and supernumerary teeth, and odontomas. The most serious complication of Gardner's syndrome is the multiple polyps that affect the large intestine. The inevitable outcome ofthis disease is invasive colorectal ancr.

Clinical features: . Onset early puberty . Polyps ofthe colon ultimately change into adenocarcinoma by the fourth decade of life . Abnormality ofthe retina ofthe eye . Development olmultiple epidermal cysts usually on face, scalp, and extremities
The oral findings ofGardner's syndrome include:

. Vultiple impacted and supernumerary teeth . \4ultiple jaw osteomas which give a "cotton-wool" appearance to the jaws. These
osteomas appear as dense, well-circumscribed radiopacities. Osteomas most olten dev-

eiop first rvithin the angle ofthe mandible

. \4ultiple odontomas
\\rhen Gardner's syndrome is suspected based on oral findings, the patient should be referred to a gastroenterologist for consultation. Note: Multiple desmoid tumors t.tibt onatosis) and epidermoid cysts ofthe skin are also characteristic of the disease. Remember: Multiple impacted and supemumerary teeth are also seen in Cleidocranial dl splasia.

***

Fibrous dysplasia is an idiopathic condition in which normal medulJary bone is gradually rcplaccd by an abnormal fiborus connective tissue prolifcration. The mesenchymal tissuc contains \ arllng amounts ofosleoid and osseous mateial that presumably arises through metaplasia.

Thc discase most commonly presents as an asymptomatic. slo$' enlargcmcnt


bone. I! may jnvolve one bone or scvcral bones concomitantly.

of the

involved

. \Ionostotic

fibrous dysplasia: is the most common, comprising 707o of cases. There is an equal male to female ratio, and this form is the most likcly to quiesce at puberty. A typical monostotic lesion Nill involve the fcmur, tibia or ribs, with 2570 occu.ring in the bones ofthe skull flle mdiil/a r.r .atnnnlv involved). A panorex will show a radiopaque mass with inegular bordcrs that has a "ground glass" appearance. See picture #,19 in booklet . Poltostotic fibrous dysplasia applies to cases in which more than one bone exhibits evidence ofthe disLrrder. It is rclatively uncommon; horlcver, many patients have lesions of the skull, facial boncs, or ii\\ s. as a component ofthe condition. Note: Alkaline phosphatase may be elevated in up to 307o oipalicnts $ ith polyostotic fibrous dysplasia, and a dramatic risc may herald malignant degeneration. Cal.rum and phosphorus tcnd to be normal. Important: Fibrous dysplasia has a variable radiographic appearance that ranges from a radiolucent lesron to a densely radiopaquc mass. The classic ptesentation has been describcd as radiopaque with numerc'us bony nabeculae imparting a "ground-glasss" appearance. An important distinguishing feature L)ifibrous dysplasia is the poorly delined radiographic and clinical margins ofthe lesion The process iecms lo blend into the surrounding normal bone without cvidence of a circumscribcd border.

\lalignant degeneration occurs in less than l7o ofcases offibrous dysplasia. Malignancies are almost exciusi\,ely osleosarcoma. For unknown reasons, monostotic and craniofacial Iesions have the greatest potential formalignant dcgeneration, and radiation therapy has been found to incrcase the risk by 400-fold. The differential diagnosis of fibrous dysplasia of the jaws includes the ossilying fibroma, howeyer, radiographically thc ossifying fibroma has a well-circumscribed appearsnce.

\ote:

Remember: N{cCunc-Albright syndrome is a designation that has bccn applied to patients with polyostotic fibrous dysplasia, cutaneous melanotic pigmcntation fcate .r lait macules), and endocrine abnorrnaTlties (rnost cotwtronly precocious sexual

devlopuent in.lbmales).

ORAL PATHOLOGY

Odont Cyst

A new patient walks into your oflice. Your initial physical assessment reveals that his eyes are set wide and that he has multiple lesions of the skin. When shaking his hand, you notice that the skin ofhis palm is very thick and has palmer pitting. When doing a health history, he reveals that he sees a neurologist and that he has some calci{ied structures "in his brain." A panoramic radiograph may likely reveal:

. Osteomas

. Odontogenic keratocysts . Odontomas


. Dentigerous cysts

100
Copynghr i,e

20ll'2011-

Denral Decks

ORALPATHOLOGY

Odont Cvst

A mother brings her 2-year-old boy into the dental oflice because of a "swelling" on his alveolar ridge. Your exam reveals a smooth-surfaced bluish lesion with fluctuance where tooth #K will be erupting. The most likely diagnosis ofthis is:

. Dentigerous cyst . Eruption cyst

. Hematoma
. Hemangioma

10'l Coprrighl !, 2011-2011 - Denlal Decks

This patient has nevoid basal cell carcinoma syndrome, which is an autosomal dominant disorder characterized by oral, systemic and skeletal anomalies. It is an inherited group ofdefects which involve abnormalities ofthe skin, eyes, nelvous system, endocrine, glands and bones. The condition is characterized by an unusual facial appearance and a predisposition for skin cancer. It is also krown as the basal cell nevus-bifed rib syndrome, the basal cell nevus syndrome and the Gorlin and Goltz syndrome. Note: Recently, mutations in the human homologue ofDrosophila patched fPICIl), a tumor suppressor gene were identified as the underlying genetic event in this syndrome.
Possible abnormalities include:

. Cutaneous anomalies: including multiple basal cell carcinomas, other benign dermal cysts and tumors, palmer pitting, palmer and planlar keratosis and dennal calcinosis.

. Dntaf and osseous anomalies: including odontogenic keratocysts (o.ften multiple) mild n.randibular prognathism, rib anomalies (often bifid), and venebral anomalies. . Ophrhalmologic abnormalities: including hypertelorism with wide nasal bridge an
congenital blindness. . Neurologic anomalies: including mental retardation, dural calcification, agenesis of corpus callosum and congenital hy&ocephalus. . Sexual abnormalities: including hypogonadism in males and ovarian tumors in females.

Radiographic features:

. Calcification ofthe falx cerebri . Presence of odontogenic keratocysts

An eruption cyst is essentially a soft tissue variant ofthe dentigerous cyst. It is invariably associated with an erupting tooth (asaa lly prim^ry but o(casionally a permanent tooth). The effects are mostly limited to the overlying gingival tissues rather than bone. Clinically, the lesion usually appears as a smooth-surfaced, reddish-pink or bluish-black, tluctuant. Iocalized swelling on the alveolar ridge over the crown ofan erupting primary or permanent molar tooth. The intense bluish coloq which is often characteristic, is due ro an accumulation of blood. Due to this appearance, it may be mistaken for a hemangioma or hematoma.

\o

treatment is necessary

as the cyst often ruptures spontaneously. In a few

rare cases,

incision or even the removal ofthe overlying tissue may be necessitated by pain or tendemess associated with lhe lesion.

Odont Cvst

Upon viewing a panorex ofa l4-year-old patient, you see a well-de{ined multilocular radiolucency with scalloping around the roots on the left side ofthe mandible rpical to the canine and first premolrr. No clinical symptoms are present. Teeth are not carious and respond normally to vitality tests. Mdical history is unremarkable. Upon opening into the area, no fluid or tissue is evident. What is the most probable diagnosis?

. Dentigerous cyst
. Traumatic (simple) bone cyst

. Primordial cyst
. Residual cyst

. Stalne /s/a/i., bone defect


102
Copyflghr O

20ll 201:

Dental Decks

ORAL PATHOLOGY

Odont Cvst

A healthy l9-year-old patient presents to your olTice for a routine exam. Taking a panoramic radiograph, you see a well-corticated, unilocular radiolucency surrounding the crown of impacted tooth #17. The lesion is asymptomatic. What is the most likely diagnosis?

. OKC
. Dentigerous cyst

. Cystic ameloblastoma
. Central ossifying fibroma

103 Copyaghr,e

20ll

:01:

Denral Decks

Remember: This cyst may be completely devoid ofsolid or liquid material. lt occurc most frequently in younger persons rlith no sex predilection. The usual location is in the mandible bctween the canine and ramus. The regional teeth are vital.

Clinical features: . Children and adolescents. usually belbre the . Usually asynlptomatic . Prinarily seen in mandiblc

age

of20

. Slight ilun) conieal

bonc crpansion

. Associated with vital teeth, no displaced teeth


The dentigerous cyst (or lbllicular c.vst) contains a crown ofan unerupted tooth or denlal anonralv such as an odontoma. Enlarged dentigerous cysls can cause Drarked displaccmcnt oftccth. Prcssure of accumulatcd fluid usually displaces the tooth in an apical direction.
See

picture #50 in booklet

The primordial cyst diflers from the periodontal and dentigerous cysts in that it conlains no calcrlied structures. These cysts are lined by stratified squanous epithelium and may be either unilocular. multilocular or multiple. The residual cyst rcl'ers to a situation in which a tooth associated with a radicular cyst is cxtractcd but the c)st is lcft undisturbed, it persists within thejaw and this lcsion is called a residual cyst. \ote: \bu must curette the socket ofa tooth with a radicular cyst aftcr extraction. See pictures #

86 and #87 in booklet

Ihe st^Ine (stqtic)

bone defect is radiolucency ofthe mandible due to invagination ofthc lingual .urlace of the of the jaw. lt is locatcd in thc posterior mandible below the mandibular canal.

\ote:

henratopoietic bonc marrow.

Thc focal osteoporotic bone marrow defect is a radiolucency in the jaw that conlains 11 is olten seen in an extraction site.

Clinical features:

.lt

usually contains a crown ofan unerupted tooth . Usually not clinically visible rvithout radiographs . .\\\ nrptomalic, occasionally pain or swclling . Lsually involve uneruptcd mandibular third molars. olher frequenl sites includc ma\illary canines.

marillary third molars and mandibular second premolars Rrdiographic feature: \\'ell-circumscribed, unilocular radiolucency around crown ol'looth. =50 in booklet
Remember:
See

picture

The lateral periodontal cyst may bc dcfincd as a non-kciatinized, non-inflammatory dc\ clopmcntal cyst occurring adjacent o. lateral to thc root ofa tooth. See picture #91 in booklet

:.

The fissural cysts ltthich are also called developmental .,l.irtr) are non-dental in oiigin, thcy include nasoah colar. median palatal, and nasopalatine cysis.
-1. -1.

Th!- primordial cyst contains no calcified structures.

Thc traumatic (simple) bone cyst n].ay contain blood, 11uid, dcbris or be completely empt-l.', Comrnonly found in young persons, in the mandible between the caninc and ramus.

The residual cyst is often found in edentulous areas. This cyst refers to a situation in which a tooth * ith a radicuiar cyst associated with it was extracted. and the socket wasn't curelted. The radicular clst persists in thejaw as a residual cyst. The gingival cyst ofthe newborn has also been designaled as the dental lamina cyst ofthe newborn or Bohn's noduels. Such cysts appear typically as multiple nodulcs along the alveolar ridge in neonales. They are due to cystification ofrcsts ofthe dcntal lamina. In the vast majority ofcascs. these cvsts degeneratc and involute or rupture into the oral cavity. Note: Similar epithelial inclusional cysts may occur along the midline ofthe palate fpala/i/te q'sts ofthe nerborn or Epstein's 2earl9. Thcse are ofdevelopmental origin but are not dcrivcd from odontogenic cpithelium. No

treatlnent is necessarv.

. Henwig's epithelial root sheath . The reduced enamel epithelium

. Remnants ofthe dental lamina . A pre-existing osteoma

10,t CopFiShr O
201 I

'2012 - Dnral Decks

. Residual cyst

. Traumatic bone cyst


. Primordial cyst . Periodontal cyst

105 Copynght O 201l-2012 - Denhl Deck

Keratocysts dillcr fiom olher odonlogenic cysts in their nricroscopic appearancc and clinical behavior They may resemblc periodontal, primordial or follicular cysts. Usually they cannot be distinguished radiographically. See picture #88 in booklet

Clinical features: . Widc age range. peak

occurrence in 2nd and 3rd decades [,csions found in children are often reflective of nrultiple odontogenic keratocysts as a componenl ol'the nevoid basal cell carcinoma syndrome . More common in males than fcmalcs . The chrefsite ofinvolvcmcnt is thc mandible, in approximatcly a 2 to 1 ratio . In thc nrandiblc, most occur within the posterior portion ofthe body and rarnus rcgion . Typically asymptomatic

Radiographic features:

. well-demarcated area ofradioluccncy with . Unilocular or multilocular \licroscopicallyi . The lininging epithclium
dramctcr

a sca)lopcd, radiopaqr,re

margin

is thin and parakcratinizcd

. The basal layer is palisadcd u,ith prominent, polarized, and intensely staining nuclei ofuniform

. The luurcn may contain large amounts ofkeratin debris or clear fluid similar to serum transudalc . The parakeratotic type forms 85 10 95% ofall odontogenic keratocysts; thc balance is made op of the .'rthoker.ltinized vaiant. lmportant: The orthokeratinzed variant is less aggressive, \\ ith a rnuch lorver rate ofrccurrence and is not syndrome-associatcd.

Important: The most remarkable


rence.

feature of keratocysts is their great tendency toward recur-

A primordial cvst arises from cystic changes in a developing tooth bud before the formation ofenamel and dentin matrix. Since the primordial cyst arises from a tooth bud, the tooth \\'ill be missing from the dental arch unless the cyst arose from the tooth bud of a supenrumerary tooth. The mandibular third and fourth molar regions are the most common locations for a primordial cyst. lt is usually found in children and young adults

\elrrp. \lany investigators have reported that most prirrordial cysts have the same aharacteristic l'eatures as those of odontogenic keratocysts. However, until conclusive
prtrtri is established, primordial cysts and odontogenic keratocysts are considered separate iit trtles.

l0 ard 30 years of age. Radiographically, the primordial cyst is a circular radrtrlucencv * ith a radiopaque border and lound at the site where the tooth failed to debetri .-en

\\ h.n a tooth having a radicular cyst at its apex is extracted, the radicular cyst is left
behind in bone and is now called a residual cyst. A residual cyst can also rise from remn3nts ofthe epithelial rests after tlre extraction ofa tooth. This cyst occurs in older indir iduals. the average age is 50 years. The radiographic appearance is that of a circular radiolucency sunounded by a radiopaque border and occurring in an edentulous area. A residual cyst can easily be misdiagnosed as a primordial cyst. The latter arises in lieu of a looth r\hereas a residual cyst arises in relation to an extracted tooth.

. Lateral periodontal cyst


. Dentigerous cyst

. Odontogenic keratocyst

106 CoplriSht O 201


1,201 2

- Dental Decks

. Based on symptoms . Radiographically

. Histologically
. By an electric pulp tester

107

Copright O2011,2012

- Denral Decls

Cyst
Dendgerous

Clinical Characteristics

Radiogmphic Chrracteristics

{follicular cyt)

. . . .

Children and teenagers Mandibular third molar and maxillary canine area 00%o) in nandible) Associated with impact or unerupted teeth Sccond most common odontogenic cyst Usually occurs between the ages of 10-30 Often associated with an impacted tooth 50o/o mandibular third molar area Over 307o recurrence rate 95% mandibular canine-premolar area Apposition with rooi ofvifal tooth Usually symptomless

Well-defined usually unilocular RL associated with the crown of !n unerupted tooth Well-circumscribed RL with smooth margins lnd thin
radiopaque borden

Odontogenic kmtocysl

Lateral Periodontal cyst

Well-defined, round or teardropshaped RL with an opaque margin along lateral surface of tooth

\otcs,

L The lateral Dcriodontal cvst mav be deflncd as a non-keratinizcd. non-inflammatorv developmental cyst occurring adjacent or latenl to the root ofa tooth. 2. Thc gingival cyst ofadulthood is the sofl-tissue oounterpart ofthis lesion. 3. The origin ofthe lateral periodontal cyst is related to proliferation ofthc rests ofdental

lamina.
4. The calcifying odontogelic cyst (COC./ is believed to be dcrived from odontogenic ep-

ithelial remnants within thc gingiva or within thc mandible or maxilla. Ghost cell keratinization is the characteristic microscopic fe.ture of the COC. Radiographical)y, the COC may prescnt as a unilocular or multilocular radiolucency with discrctc. wcll-demarcated margins.

within

the radiolucency, ghost cells may undergo calcification and appear as


as

a "salt and peppcr"

typc of pattcm. Note: This cyst has a cutaneous countcrpart known

calcifu ing epithelioma of Malherbe or pilomatrixoma. 5. The glandular odontogenic cyst, or thc sialo-odontogenic cyst is rare cyst that occurs most often within the mandible. This lesion is locallv asgressivc and recurence is common.

Lesion
Dental gaanuloma (h is one ofthe mosl

Usual Location
Apex oftooth

Clinical and Radiogriphic Characteristics

Treatment
Root canal rcatment or extraction of involved tooth

. . . . . . r .

coamon ofall sequelae

o{p lpiti,
Radicula. cyst l..llso called apical Apex oftooth

Asymptomatic Circumscribed radiolucency at apex of tooth Tooth is non-vital May be sensitive to percussion

periodontal & periapical

fls,

It is the most common odontogenic cyst

Root canal treatment Asymptomatic Circumscdbed ndiolucency at with apicoectomy or extraction wilh apex of tooth curettage ofsocket Tooth i: non-vital May be sensitrve to percussion

\ecrotic pulp causes periapical inflammation: . lt acute. a periaprcal abscess forms . If chronic, a dental granuloma forms
cyst derives its epithelial lining from prolitbration of small odonrogenic epithelial residres (rest of Malcssez) within the periodontal ligament.

\ote: This inflammatory

ORAL PATHOLOGY

Odont T[rm

A 37-year-old patient comes into your office with the complaint of a slowlygrowing, painless swelling of his lower left jaw. A panoramic shows a multilocular radiolucency with well delined and sclerotic margins along the left mandibular molar-ramus area. All teeth test as vital. A triopsy is conducted and shows odontogenic epithelium. What is the most likely diagnosis for the most aggressive type of this turnor?

SoIid (multicystic or pol!"cystic)

. Unicystic
. Extraosseous
(p
e

rip heral)

108 Copyrighr'a 201 l-2011 - Dental Decks

ORAL PATHOLOGY

Odont T[m

A 30-year-old patient coms into your oflice complaining ofa painless swelling of his lower left jaw. A panoramic radiograph shows a welltircumscribed

multilocular radiolucency with a 'honeycombed" pattern at the location ofthe lower left molars. The teeth have been displaced. The pathology report calls this an odontogenic mlroma, This tumor:

. Is composed of large polyhedral, neoplastic, epithelial cells

. Is composed ofneoplastic epithelium

and mesenchyme

. Arises from follicular connective tissue resembling pulp tissue

. ls composed of spindle-shaped
giant cells

mesenchymal cells and aggregates of multinucleated

109

coplrighr ill 20ll-201: - Denral Decks

Ameloblastoma consisrs entirely ofodontogenic epithelium, which at sites shows the differentiation ofthc familiar, histologic layers ofthe enamel org.n. It is most often seen in the mandibular molar-ramus area. Clinically, it is the most aggressive odontogenic tumor. Important: The ameloblastoma is the most common epithelial odontogenic tumor.

Clinical features:
. No sex or racial predilection . Mean age is commonly between 35 and 45 years . 2 biologic-microscopic subtypes:
treatcd consen'ativcly. I ) Solid or multicystic: is more aggressive and high recunencc rate 2) Unicystic: is less aggessive and less likely to recu. A histologic variant ofthis type is the plexiform unicystic ameloblastoma.

if

. Slow growing .

and painlcss swclling

. Locally aggrcssivc . Capable ofcausing large facial deformities


Teeth are vital

\oter

The) exhibit

Rarely, extraosseous peripheral ameloblastomas arc found in tlre gingiva and buccal mucosa. a benign non-aggressive course.

Radiographic characteristics: Multilocular or unilocular radioluccncy with well defincd and slerotic margins. Notei In the mandible it appears similar to thc central giant cell granulona. See picture #51 in booklet Treatment: Each casc should be considered on its own basis. The solid hnullicristic or polvcl'stic) le.ion requires surgical excision. Rescction should be reserved for larger lesions. Unicystic Iesions usu alh require only enucleation. Thcy should not bc ovcr trcatcd. Note: You nccd to know thc .rmeloblastoma ertremely well for thc cxam!l!

\olei }lalignant

behavior by amcloblastoma is rarely encountered. These lesions occur in a younger age group (JOr/ and appcar in the mandible more frequently than the maxilla. Malignant lesions have been ll\ rded into nvo subtvpes: the malignant ameloblastoma and ameloblastic carcinoma.

The odontogenic myxoma is mesenchymal in nature and origin. mimicking microscopicall! the dental pulp or follicular connective tissue. The mean age in which this lesion appears is J0. Radiographically. this lesion is always radiolucent, although the pattem mav be quite variable. It may appear as a well-circumscribed lesion or as a diffuse lesion. h rs often multilocular. frequently with a "honeycombed" pattem- Important: Cortical exltnsion (rother than perforation) and root displacement (rqther lhan resorplion) are the rule. Surgical excision is the treatment ofchoice. These lesions exhibit some aggressiveness and have a moderate recurrence rate. See picture #90 in booklet

\ote: \\:hen relatively

large amounts ofcollagen are evident, the term used to designate the odontogenic myxoma.

"myofibroma" is

Ihe central odontogenic fibroma is a rare lesion that is regarded

as the central counter-

part to the peripheral odontogenic fibroma. It is seen in all age groups, and is found in both rhe n.randible and maxilla. It appears as a well-defined radiolucent lesion that is usually multilocular, often causing cortical expansion. It is treated with surgical excision and recurrence is very uncommon.

l. The calcifying epithelial odontogenic. (pindborg) ttrmor is composed oflarge polyhedral,neoplastic,epithelialcells. \otcr . t,,.,.r,rr' 2. The ameloblastic fibro-odontoma and ameloblastic fibroma are composed of neoplastic epithelium and mesenchyme. 3.The central giant cell granuloma is composed ofa proliferation ofspindled fibroblasts in a stroma containing variable amounts ofcollagen. Multinuclated giant cells are present throughout the connective tissue stroma.

. In

the mandible than in the maxilla. and more often Dosterior than in the anterior

reglons

. In the mandible
reglons

than in the maxilla, and more often anterior than in the posterior

. In

the maxilla than in the mandible. and more often Dosterior than in the anterior

regions

. In the maxilla than in the mandible,


regl0ns

and more often anterior than in the posterior

'fi0
Coplrighr O
201 1-201 2

- Dnral Decks

. A predilection for persons younger than 20 years old

. A predilection for middle-aged caucasian men


. A predilection for middle-aged black women . No age, racial or sex predilection

111 Coptrighr

201 l

'2012 - Dntal Decks

The cementoblastorna, also known as the true cementoma, is a rare benign neoplasm of cementoblast origin. It occurs predominantly in the second and third decades, typically before 25 years ofage. It is more often seen in the mandible than in the maxilla, and more often posterior than in the anterior regions. It is intimately associated with the root of a tooth, and the tooth remains vital. It may cause cortical expansion and, occasionally, lowgrade intermittent pain. Radiographically, this is a well-circumscribed radiopaque lesion that replaces the root ofthe tooth. It is usually surrounded by a radiolucent ring. Because ofthe intimate association ofthis lesion with the tooth root, this lesion cannot be removed without sacrificing the tooth. There is no recurrence.

Note: To distinguish this lesion from condensing osteitis the root outline.

in CO you can distinguish

represents a reactlve This lesion also known as periapical cemento-osseous dysplasta response of periunusual an to be appears lesion ,uth". thun a neoplastic proiess. This phenomenon that occurs aI apical bone to some local factor This is a relatively common

affected more than are men' the apex ofvital teeth. Women, especially black women' are the anterior peiiopp"urc in *iOat e age (around i0 ,earsl and the mandible' especially often' the apices More riapical region, is farrnor" .o.-only uffected than is the maxilla vital' As the are the teeth of trvo or r.riore teeth are affected. There are no symptoms and a mixed or mottled patcondition progresses or matures, the lucent lesion develops into The final stage appears as a solid opaque mass often surrounded

,. i"t. repair thin iucent ring. No treatment is required for this condition' by a "ttl.g

i"*

\ote: A rare condition called florid

osseous dys

plasit (FoD) appears to be an exuber-

ant fonrl ofperiapical cemental dysplasia'

the age of 40 There are The cementiffing fibroma occurs chiefly in adults around Iesion may cause tooth The fr.iii".tio* ior ine uody of the mandible antl for females' it is well-circumscribed and may aprr.ro. ement or cortical expansion. Radiographically, lt is treated with pea, ..tatively radiolucent, lucent with opaque foci, or diffusely opaque can be considered simcu..etag. o. e^cirion. Recurrence is very rare Note: This lesion ilar or identical to the ossifying fibroma'

. TuberosibT area . Maxillary anterior area

. Mandibular premolar area


. Molar

ramus area

112 Coplright O

20ll-2012

Denral Decks

s-,

Th smeloblrstic libromr and ameloblastic librordontoma eppeer to be variations of tbe same process. These neoplasms occur predominantly in:

. Adults with a mean age of 40

. Elderly people with a mean


. Young adults with
a

age

of

75

mean age of25

. Children

and young adults

.113

Copyright O2011,2012 - Dentat Deck

The mandible is affected twice as often as is the maxilla. There is a predilection for the molar-ramus region. although any site may be affected.
sheets of large polygonal epNote: A unique microscopic pattem typifies the CEOT v,ith of amyloid that have concentric calcified deposits ithelial cells are usually seen areas ([, ie s e g an g ri ngs ) tl]roughout.

Teens; females:

anteriorjaws; in association wilh the crowns ofimpacted


teeth

Well-circumscribed unilocular RL lesion; may have small opaque foci

Adults f,/0

l"dr'ti

molar-ramus region;

law expansron
Adolt qhears);
anterior maxilla and posterior mandible; no symptoms
Females over 60; mandible and maxilla

Resection ofaffcctcd Unilocular or multilocular RL lesior! may have opaque area; recurrence common if inadcquately treated foci

Well-circumscribed, oft en scmilunar RL lcsion


associated with the roots teeth

of
Resection of affected areai locally aggrcssivc; metastases lo lung and regional lymph nodes

E\cept for the presence of an odontoma, the 1wo lesions are tlte same. The mean age is lround 12 years, and the upper age limit rnay extend as high as 40 years. The mandibuiar molar-ramus area is the favored location for the two lesions. Radiographically, these lesions are rvell circumscribed and are usually surounded by a sclerotic margin. They mal be either unilocular or multilocular and may be associated wiih the crown of an impacred toofi. Important: An opaque focus appears within the ameloblastic fibro-odontona o\\ ing to the presence ofan odontoma. This lesion therefore presents as a cornbined lucent-opaque lesion; the ameloblastic fibroma is completely lucent radiographically.

\ote:
rare.

These lesions are treated conservatively by curettage or excision. Recurrence is

The other tumor of mixed, (epithelial and mesenchynal) origin is the odontoma. These ;alcilied iesions take one or two general configurations. They may appear as multiple miniature or rudimentary teeth, in which case they are known as compound odontomas, or they may appear as amorphous conglomerations of hard tissue, in which case thel are known as complex odontomas. As a group they are the most common odontogenic tumors.

How would you refer to the group of small radiopacities between the rnandibular canine and first premolar on the periapical x-ray below?

'114 Cop)rrghr
ae

:0ll :012

Dertal Decks

ORAL PATHOLOGY

A 4s-year-old African Amerlcan female presents to your oflice for a routine exam. Periapicals of the mandibular incisors show multiple radioopacities with radiolucent rims. Teeth #23 through #26 test as vital. There is no pain on percussion or palpation. Tfeatment for these lesions should be:

. Do nothing (obserue)

. RCT treatment lor

teeth #23 through #26

. Surgical excision of lesions


. None ofthe above

115 Cop)'right C 201 l-2011 - Dental Decks

Tumoa Complex

Clinical l'eatures
Children and young adultsi tendency to occur in the posreriofja\r's

Radiogrxphic Charactedslics Amorphous opaque


masses iYpically in a tooth-beanng ara, between roots or over tne

\{icroscopic Characteristics Nomlal appealng


enamel. dcntin. cementum. and pulp may be sccn.

Treatment
aud Prcgnosis Enuclealion;

impacted tooth

Compound Children and young adults; tendency to occur in thc anteriorjaws

Multiple small toothllke structures


typically in
a

tooth-

bearing area, berween roots or over the qown ofan impacted tooth

Normal-appearing enamcl. dentin. cementum. and PulP may bc seen.

Enucleation;
docs not rccur

Remember: A compound odontoma looks like a tooth, and a complex odontoma does not lit is a disorganized arrangement oJ tubular dentin, enqnrel' and thin la1'ers of cenemunl. See picture #52 in booklet for picture ofcomplex odontoma l.'ote: A rare variant known
as ameloblastic ondontoma has been described. This is essentially an ameloblastoma in which there is focal differentiation into an odontoma.

Periapical cemental dysplasia, also known

as

periapical cemento-osseous dysplasia (and

lornerly knov'n as cementoma) represents a reactive rather than a neoplastic process. The tenr "cernentoma" is a misnomer as thc opacities are not cementum but bone. While they appear lo arise lrom the teeth, the lesions apparently arise within the bone instead. It appears to be an unusual response ofthe periapical bone to some local factor (br example, It dunntic occIusion or infbction). Clinical features: . Occurs at the apex ofvital teeth

. A pre dilection fbr middle-aged black women . As] mptomatic, usually multiple, small periapical
lar incisorarea. See picture #53 in booklet

areas ofradiolucency in the mandibu-

\ote:

Depending on stage, it may appear mixed radiolucent and radiopaque or totally ra-

dropaque.

Important: Age. gender, Iocation, radiographic appearance, and tooth vitality considered together are diagnostic of this condition.
Three stages:
L Osteolytic stage; radiolucency appears on radiograph. Cementoblastic stage: beginning of calcification in the radiolucent area (mixed radiolucent and rqdiopaque appearance). 3. llature stage: radiopacity appears on radiograph with a thin radiolucent line around

l.

area.

\ote:

No treatment is required for cementomas. Once this stage is reached, the lesion stabilizes and causes no complications.

ORALPATHOLOGY

PigLesofOralCav

A S-year-old boy presents with his mother for his lirst dental exam. Your exam reveals a normally developing dentition but you notice multiple "freckles" on his lower lip and on the buccal mucosa. What condition should vou be concerned about?

. Gorlin-goltz syndrome . Gardner's syndrome


. Peutz-Jeghers syndrome . Cleidocranial dysplasia

115

Coplriglrr o

20ll-l0l:

- Denral Decks

ORAL

PATHOLOGY

Pig Les of Oral Cav

What most likely caused the discoloration of the gingiva as seen in the picture blow?

117 Copyrighr C

20ll'l0l:

Denral Decks

Peutz-Jeghers syndrome (also called hereditat! itesti al polyposis syndrome) is an unusual condition which is ofinterest to the dentist because of the oral findings. The pigmentations usually appear at an early age, often during the first decade of life and at this time are restricted to the oral region. Intraorally, these pigmentations may be located anywhere on the mucosa, but are most frequently seen on the buccd mucosa, gingiva and hard palate. The mucosal surlace ofthe lower lip is almost invariably involved. These spots or macules, while

of variable intensity. may mnge through shades of brown, blue and black. During the succeeding decades of the patient's life, pigmentations may arise elsewhere on the skin, especially on the extremities. In addition, intestinal polyposis is seen, most commonly in the small intestine (jejunum) and may produce signs and symptoms of abdominal pain, rectal
bleeding, and diarrhea. It should be noted that the pigmentations ofPeutz-Jeghers syndrome may occur without demonstrable evidence of polyps and, also that multiple pollps may be encountered without pigmentations. See picture #35 in booklet

Important: It is significant to note that although the oral pigmentations per

se are harm-

less. their presence is important in that they indicate a necessity for investigating the pos-

sible presence of multiple polyposis which may prove harmful. There is a strong tendency

for these multiple polyps of the colon to undergo malignant chang.

\ote: It

appears to be caused by a mutation ofa gene known as I 9 that encodes a multifunctional serine-threonine kinase.

LKBI ofchromosome

has been mistaken for a mclanin-pigmented lesion. The most common looations for amalgam tattoos are the gingiva. buccal mucosa and alveolar mucosa,

.\n amalgam trttoo is the most common oral pigmented lcsion The tattoo

Trauma and chemicals are frequent causes

oforal lesions:

occurs when patients placc the tablet against an aching tooth, allowing the check '\n aspirin burn or lip to hold it in position while it dissolves slowly. Within a few minutes a buming sensation ofthc mucosa will bc noted and thc surface becomes blanched or whitened in appearancc The caustic acrion ofthe drug causes necrosis ofthe oral mucosa, with subsequent sloughing ofthe necrotic epiiheliurn. \ote: Aftempt to locate amalgam on x_ray if tattoo is suspcctcd; ifyou can not locate any amalgam. a biopsy may be needed to rule out a melanocytic neoplasm See pictur #54 in booklet Remember: Chcmical bums may also be caused by hydrogen peroxidc. silver njtratc, phcnol' etc. . The use ofthe heary mctal bismuth is still common in treating certain dermatologic disorders as well as larious other diseases. Bismuth pigmcntation appears as a "bismuth line," a thin' blue-black line in the marginal gingiva which is sometimes confined to the gingival papilla Note: Lead ingestion also l.avcs dark marks on the gingiva. . Traumatic ulcer: very common; may mimic oral cancer and chronic infectious ulcers . Fo.al (frictionat) keratosisi common whitc lesion caused by chronic friction on the mucosa. Important: Differcntiated from idiopathic leukoplakia becausc cause is known . Linea alba: a type offrictional keratosis that appcars as a linear whitc line in buccal mucosa . Smoking-associated melanosis: caused by a chemical in tobacco smoke that stimulates melanin production. Note: If thc pigmentation is localized, an ulceration is present or thc lesion is elevalcd, a biops.v is necessary to exclude other pigmented conditions feg, nevi, melanoma). Although smoker's mclanosis is an abnormal deposition ofmclanin, the lesion itsclf is not associated with an increascd risk ofmclanoma or carcinoma. . Nlelanotic macufei most common mclanocytic lesion. Gingiva a commonlocatton (called an orul nelonotic macule - See picture #55 in booklet/ and the lips, most frequently on the lower lip (called a labial melanotic macrle). Can be associated with Peutz-Jegher's syndromc . Drug-induced pigmentation: Most commonly associated with minocycline' chloroquine' cyclophosphamide, and azidothymidine (AZT).

. Buccal mucosa
. Tongue

. Hard palate
. Alveolar mucosa

118

Copright O 201l-201: - Dental Decks

All ofthe following conditions demonstrate pigmentation of th intraoral mucous membranes .EXCfPl one. Which one is the EXCEPTION2

. Addison's disease

. Albright's syndrome

. Cushing's syndrome
. Peutz-Jeghers syndrome

't19 CoD,right C 201 I -20


12

- Denral Decks

Moles (nevi) are small, usually dark, skin growths that develop frotn pigmenlproducing cells in the skin (called melanocltes). While nevi are fairly common on the skin, intraorally they are quite uncommon. When present, they are usually on the hard palate, but can also be seen on the gingiva and lips. Congenital nevi (commonlt knou,r as birthmarks) are usually large (greqter thqn I 0 cm) and with the passage of time, may change ftom flat, pale tan macules to elevated, vemrcous, hairy lesions. Approximately 150/o occur on the skin ofthe head and neck. Congenital nevi have a higher incidence of malignant transformation (as opposed to acquiretl nevi).

Acquired nevi are microscopically classified into five subtypes:

l. Intramucosal nevus: most common in oral cavity 2. Blue nevus 3. Compound nevus: rare in oral cavity ,1. Junctional nevus: rare in oral cavity 5.Intradermal nevus: is the most common lesion of skin. known as the common mole
\ot: Acquired nevi ertraorally.
are much more common than congenital nevi both

intraorally and

Important: The B-K mole syndrome and the dysplastic nevus syndrome are both characterized by having numerous large, pigrrented atypical nevi which have a high risk for
der eloping malignant melanoma.

Addison's disease (qlso called chronic adrcnocortical itsuflicienq,) results from hypofunction ofthe adrenal cortex. lt is chamcterized by bronzing ofthe entire skin. Oral signs consist
of diffuse pigmentation ofthe gingiva, tongue, hard palate and buccal mucosa. Although curaneous pigmentation will most likely disappear following therapy, pigmentation ofthe oral tissues tends to persist.

-{lbright's s1'ndrome (also celled McCune-Albright syndronre) is a severe form of


pollostotic fibrous dysplasia, involving nearly all bones in the skeleton. In addition to the bone lesions there are brown patches ofcutaneous pigmentation (called caf6-au-lait
spots, and endocrine dysfunction, especially precocious puberty in girls. Important: There is an increased incidence of osteosarcoma seen with polyostotic fibrous dysplasia.

Peutz-Jeghers syndrome (also known as Hereditary Intestinal Pol!-posis Syndrome) rs an inherited disorder that is characterized by having multiple intestinal polyps and inrraoral melanin pigmentations. These pigmentations usually appear during the first decade of life and at this time are restricted to the oral region. They may be located anyu here on the mucosa, but are most ftequently seen on the buccal mucosa, gingiva and hard palate. The mucosal surface of the lower lip is almost invariably involved.

\ote: Cushing's syndrome

is a hormonal disorder caused by prolonged exposure

ofthe

body's tissues to high levels ofthe hormone cortisol. It is relatively rare and most commonly affects adults aged 20-50. The symptoms vary but most people have upper body obesity, rounded face, increased fat around the neck ("buflblo hunp"), and thinning arms and legs.

,'

year oro patient pnesenrs presents ro A /ru 40 yeer physician wrtn otd pauena to nrs his pnysrcran with comptaints muscle complaints ol of musce werkness and loss of appetite. II has noticed a loss of weight and also his skin hss started to "bronze,' His labs show lowered blood glucose and sodlum and an increased potasslum. One condition likely to be crusing this is:

t[rt

Peutz-Jeghers syndrome

. Cushing's syndrome
. Addison's disease

. Albright's syndrome

120
CoplriShr
@ 201

1,2012 - Dental Decks

Focal melanosis is a common circumstanc in which brownish areas of pigmentation occur in the oral cayity; once properly diagnosed:

. Surgical excision . Radiation

is required

is required

. No treatment is necessary . Antibiotics are required

121

Copliight C 20ll-2012 - Dental Dects

Addison's disease occurs when the adrenal glands do not produce enough cortisol /a g/ricocorticoid). Cortisol's most important function is to help the body respond to stress. The failure to produce adequate levels ofcortisol can occur for different reasons. The problcm may be due to a disorder ofthe adrenal glands themselves (prinary adrcnal irrsulJiciencv) or to inadequate secretion ofACTH by the pituitary gland f.secondary adrcnal insullicienc,").
The symptoms ofAddison's disease usually begin gradually. These include:

. Muscle weakness . Loss ofappctitc . Weight loss . Skin changes with areas of hyperpigmentation covering cxposed and nonexposed parts ofthe body. This darkcning ofthc skin is most visiblc on scars, skin folds, pressure points such as elbows, knccs, knucklcs, and tocs as wcll as thc oral mucous membranes is diffuse pigmentation ofthe gingiva, tongue, hard palate. and buccal mucosa.

-there . Nausea,

vomiting and diarrhea


has

. Low blood pressure *** Clinical features do not begin to appear until at least 9070 of glandular tissuc
been destroyed.

Laboratory tests show: . Lou'blood concentrations ofsodium and glucose

. .

Increased serum potassium Dccreascd urinary output ofcertain steroids

Important: The main concern when performing dental procedurcs on a patient with Addjson's diseasc is that the adrenal cortex has no capacity to put out cxtra cortisol as a response to strcss.

Focal melanosis is a disorder of increased melanin pigmentation that develops without


preceding inflammatory disease. nelanrn (especially in the skin).

lt

is a condition characterized by abnormal deposits

of

f tiologl': Developmental
Location(s): Any mucosal site; gingiva a comrnon location (cqlled an oral melqnotic picture #55 in booklet) and the lips, most frequently on the lower lip nracrile -See rc?lled a labial melanotic macule)

Clinical Features: Brownish coloration to the oral mucous rlembrane Radiographic Features: None \Iicroscopic Features: Melanin pigment within rnelanocytes Complications: None Treatment: None
Prognosis: Excellent Pathogenesis: Increased numbers olmelanocytes locally

Important: Melanosis may be observed in adrenal insufficiency (Addison s disease),


-{CTH-producing tumors, malignant tumors metastatic to the pituitary gland, or metastatic malisnant melanoma.

. Intradermal news
. Compound nevus . Junctional news . Blue news

. Intramucosal nevus

122
Coplright O 20ll-2012, Dental Deck

. Well-defined unilocular or multilocular radiolucency with scalloping around the roots

. Not well-circumscribed radiopaque lesion which may have a "ground-glass"


ance

appear-

Saucer-shaped radiopaque lesion

. Not well-defined multilocular radiolucency with a "pear-shaped" appearance between the maxillary central incisors

123
CopJright O 20ll-2012 - Dental Decks

Most pigmented skin tumors are composed of ner.us cells and are a result of a developmental anomaly ofmelanocytes; they are rare in the oral cavity. The initial, flat, raised lesion can become nodular, with an increase in consistency. Spontaneous involution may occur and malignant transformation is a rare complication. When found intraorally they most frequently occur on the hard palate (See picture #58 in booklet) but can also be seen on the gingiva and lips. lntramucosal nevi are the most common variety found in the oral cavity followed by blue nevi. Compound and junctional nevi are very rare. Subtypes of Acquired Nevi: .Intradermal nevus: most common lesion olskin. Known as the common mole. Nevus cells lie exclusively within the dermis. See picture #57 in booklet . Junctional nevus: nevus cells are located at the inteface between the epithelium and lamina propria. They are flat and not detected by palpation. Some regard as premalignant, may undergo transformation into malignant melanoma.

picture #59 in booklet . Compound nevus: nevus cells are located at the epithelium/lamina propria interface and also deep in the dermis. They are raised and solid. See picture #60 in booklet . Blue nevus: congenital, painless; color based on the deep cutaneous or subcutaneous/ submucosal deposits of melanin. See picture #56 in booklet . Intramucosal nevus: nevus cells are located in the connective tissue or lamina propria ofthe oral mucosa. Under palpation these nevi appear solid and are slightly raised
See

or er the surface

ofthe mucosa.

Important: Ifa pigmented lesion shows ulceration, an increase in size, darkening in color, erc.. a biopsy should be performed--+his may indicate transfomation into a malignant
melanoma.

The traumatic bone cyst is an asymptomatic intmosseous empty cavity ofyoung patients located p:imanll' *,ithin the mandible, lined by a thin loose connective tissue membrane. TrarLmalic bone cysts are non-cysts fre/ened lo as pseudocysts) and have many names, which include the simple bone cyst, hemorrhagic bone cyst, unicameral bone cyst, extravasation

t'one cyst. idiopathic bone cyst and solitary bone cyst.

Clinical features: . Children and adolescents, usually before the age of20 . \lore conmon in males . Usually asymptomatic, may produce enlargement ofthejaw . Conmonl,"- found in posterior mandible *** See picture #61 in booklet . Regional teeth are vital teeth
The treatment of traumatic cysts is relatively easy. lt consists of opening the lsion, curettage and closure. The resultant blood clot soon undergoes organization and the bone defect is soon reoarred.

is a closed compartment that has a connective tissue lining (iro epithelial linilg,r ofvaried thickness. It may contain blood, serosanguineous fluid, debris composed chiefly ofa blood clot, or it may be completely devoid ofsolid rnaterial.

\ote: This cyst

The following are also not true cysts

. Latent bone cyst

-they

are called "pseudocysts":

. Lingual mandibular concavity


. Aneurysmal bone cyst

ORAL PATHOLOGY

Pseudocyst

All ofthe following statements are true concerning the aneurysmal bone cyst.EXCEPI one. Which one is the EXCEPTIOM

. It is an uncommon expansile

osteolytic lesion of bone consisting of a proliferation vascular tissue that forms a lining around blood filled cystic lesions
age, and

of

. Most aneurysmal bone cysts occur in patients under 20 years of mon after the age ol30 . It commonly involves
the

it is uncom-

jaws

. The lesions are usually tender or painiul, particularly upon motion ofthe bone affected . Upon entering the lesion surgically, excessive bleeding is encountered

124
Copvnglu

e 2011-:012 '

Dental Decks

ORAL PATHOLOGY

R-B Les

A patient presnts with an asymptomatic, elongated, erythematous patch of atrophic mucosa of the mid-dorsal surface of the tongue due to a chronic C. albicans infection. The most likely diagnosis is:

. Thyroglossal duct cyst


. Lymphangioma . Hemangioma . Median rhomboid glossitis

.125

Copyiight O 20ll-1012 - Dental Decks

***

This is false; it commonly involves the proximal humerus, femur, tibia and pelvis. is not as common in the jaws, but it does appear here as well.

It

The aneurysmal bone cyst (ABC) is classified as a pseudocyst because it appears radiographically as a cysrlike lesion but microscopically exhibits no epithelial lining. It represents a benign lesion ofbone that may arise in the mandible, maxilla, or other bones.

It is typically seen in patients under 30 years old, with

a slight female predilection. When

the mandible and maxilla are involved, the more posterior regions are affected, chiefly the molar areas. Pain is described in approximately half the cases, and a firm non-pulsating swelling is a frequent clinical sign. On auscultation, a bruit is not head, and on firm pal-

pation, crepitus may be noted.


The radiographic picture ofthe lesion is often distinctive. Usually a unilocular radiolucency is found with slightly irregular margins. The bone is expanded and appears cystic with a "honeycomb" or "soap bubble" appearance. Teeth may be displaced with or without concomitant extemal root resorption. See picture #62 in booklet

Histologically, the lesion consists ofa fibrous connective tissue stroma containing many cavemous or sinusoidal blood-filled spaces. Fibroblasts and macrophages (histiocytes) line the sinusoids. Multinucleated giant cells, similar to those ofa central giant cell granuloma, are dispersed throughout. Surgical curettage or excision is the treatment ofchoice, with little chance ofrecunence.

\ote:

The differential diagnosis should include ameloblastoma, central giant cell granuloma and a central vascular lesion fthis can be ruled out uoon auscultation since a bruit is often heard).

This entity was once thought to be a congenital abnormality related to the persistence of the tuberculum impar, however, it is now believed that this condition is a permanent end result of a chronic Candida albicans infection. Diabetics, immunosuppressed patients and patients on long-term antibiotic therapy are more susceptible to this condition.
as a smooth, denuded, beef,, red lesion devoid offiliform papillae. The most common location is the midline ofthe dorsum ofthe tongue, just anterior to the circumvallate papillae. It is generally asymptomatic. Generally

Median rhomboid glossitis usually presents

no treatment is necessary, however, topical or systemic antif,rngal drugs to manage the pre-

disposing factors may be helpful.

Median rhomboid glossitis

. Children

Teenagers

. Post-menopausal women . Middle-aged males

126
CoDrighr O 201l-2012 - Dental Decks

. Wegener's granulomatosis

. Rendu-Osler-Weber . Sturge-Weber . Juvenile nasopharyngeal angiofibroma

127
Cop)'righr O 201 I -2012 - Dental

Deck

Burning mouth syndrome (BMS) is a relatively common condition that is characterized by a complaint of an abnormal sensation of the lining of the mouth that most patients describe as feeling like their mouth has been scalded. Usually this sensation develops in the ftont part of the mouth, typically affecting the inner surfaces of the lips, the roof of the mouth and the sides and tip of the tongue. In some patients, only the tongue will be affected, however, any combination ofthese sites may be seen. Some patients may have a decreased taste ability or altered taste sensation (bitter or salt!). Other patients may feel that their mouths are dry or sticky. In all cases, howeveq the lining of the mouth clinically appears normal. Note: BMS is a "diagnosis of exclusion."
The etiology is unknown. There are a few common diseases that should be tested lbr, such as anemia. diabetes and oral yeast infections. Buming mouth syndrome is diagnosed by doing blood tests and cultures to make certain that one of the other problems mentioned previously is not present. Ifthose tests are all negative, and ifthe lining ofthe mouth appears normal, then the diagnosis ofburning mouth syndrome is made.

Unfortunately, no one has developed a medically proven treatment for buming mouth slndrome. For about halfofthe affected patients, the condition will resolve after a period of tin.re. but no one can predict how long that will be for a particular individual. For the most part. this problem is a nuisance, and it is a frustrating situation for both patients and doctors. Note: Some individuals find relieffrom axiolytics, antidepressants, or low doses ofcapsaicin, applied 3 or 4 times topically on the area(s) where the pain is localized. appear to be quickly effective in alleviating the pain in BMS.

Rendu-Osler-Weber syndrome, also known as hereditary hemorrhagic telangiectasia, is a congcnrtal hcrcditary form of hemangioma. It is characterized by numerous spiderlike telangiectases on the face, neck. chest. lips, gingiva, buccal mucosa and tonguc. One ofthe earliest signs ol' rhe disease is epistaxis (ho:tehleeds). Arteriovenous fistula, cspecially ofthe lungs and liver. are a \ ariablc conlponent. Blceding from the tclangicctases may be recunent, lifethreatening and in;rease in sevcrity with aging. The onset often is in childhood. Both sexes are affected equally

Rememberi . Encephalotrigeminal angiomatosis fslurge-lleber disease) is an uncommon congcnltal syndronrc ofunknown etiology (somelines classdied as a variant.forn ofhemangioma). It consists of a l'acial lcsion, known as the port-wine stain, rvhich is distributed ovcr the trigeminal nerve accompanicd by a similar vascular disorder ofthe underlying mr;ninges and cerebral cortex lt usuall! occurs unilaterally. . Juyenile nasopharyngeal angiofibroma is a rare, benign neoplasm that ncarly always aflects adolescent males. It characteristically produces a mass in the nasopharynx that leads to obstruction or epistaxis. Treatment is surgery. Recurrences are conmon.
egener's granulomatosis is a disease involving granulomatous inflarnmation, necrosis and vascuLtis that most frcquently targets thc uppcr respiratory tract, lower rcspiratory tract, and kidneys.

\\

The diagnosis ofWegener's granulomatosis is established most securely by biopsy specimcns showing the triad ofvasculitis, granulomas. and large arcas ofnecrosis (lototvn a.s geographic necrosis) admixed with acute and chronic intlammatory cells. Also CANCAs (otlttueulophil c)'toplashic antibodies) are highly spccific for Wegener's granulomatosis. The treatment includes prednisone and

cyclophosphamide.

Note: Midline granuloma is a unifocal destructive process, generally in the midline of the orofacial region, that does not affect other organ systems. lt clinically mimics the lesions of Wegener's granulomatosis. Most cases represent peripheral T-cell lymphomas of the upper
respiratory tract or m ovth (perforation of the lwrd palate na!- be seen). There ls a good prognosis when treated early with radiation.

A clinical term delined as a red patch that cannot be clinically or pathologically diagnosed as any other condition is callsd:

. Leukoedema . Psoriasis . Erythroplakia


. White sponge nevus

124
Cop)righr ()

20ll

20ll

Denral Decks

ORAL PATHOLOGY

R-B Les

The arrow in the picture trelow is pointing to a fast-growing reactive proliferation ofendothelial cells which is commonly found on the gingivo and usually forms in response to chronic irritation. The most likely diagnosis is:

129
Copy.ighr e

20ll 201?

Denral Decks

"erythroplakia'r like the term "lerkoplakia" has no histologic connotation. It is a clinical diagnosis. Almost all true erythroplakias exhibit a microscopic picture of epithelial dysplasia, carcinoma in situ, or invasive squamous cell carcinoma. \ote: Biopsy is mandatory.
The term

Remmber: Carcinoma in situ exhibits all of the histologic characteristics of malignancy (pleomorphism, hyperchromatism, abnonnal niloses, onaplasia, e/c.), but does not
shorr invasiveness or extension into adjacent structures. Enthroplakias may be located anywhere in the mouth, but are most likely to be found in rhe mandibular mucobuccal fold, oropharynx and floor of the mouth. There is no sex predilection and patients over 60 years old are most commonly affected.

This lesion represents an overexuberant connective tissue reaction to a known stimuIus or injury. The term "pyogenic granuloma" is somewhat ofa misnomer in that it is not pus producing, as "pyogenic" implies. It is, however, a "tumor" ofgranulation tissue, as "granuloma" implies. Pl ogenic granulomas may also occur on the lower lip, tongue and the buccal mucosa. Thel rarel.v occur on other areas ofthe oral mucous membrane. It is generally believed lhar rhe pyogenic granuloma arises as a result of some minor trauma to the tissues fceneutdtion of q crotvrt, calculus, etc,),which provides a pathway for the invasion olnonspecitic t-vpes of microorganisms.

soft pedunculated broad-based growths that have a smooth red surface. This red appearance is due to the presence ofhyperplastic granulation tissue, which contains many capillaries. They are often ulcerated, bleed easily and ma1 have a raspberry-like appearance. Treatment consists ofexcision. They are benign, but rnav occasionally recur.
Pr ogenic granulomas present as

\ote:

Pregnant patients are prone to these lesions (sorreliz es called "pregntncy tumor").

Peripheral giant cell granulomas are seen xclusively in:

. Buccal mucosa
. Alveolar mucosa

. Bone . Gingiva

't

30

coplright o 20ll-2012 'DmialDects

ORAL PATHOLOGY

The picture below shows a benign, soft, moderately well-circumscribed prinless mass which is deep red or blueish red in coloration. The most likely diagnosis is:

131 Coplyighi C
201

1,2012 - Dentnl Decks

The peripheral giant cell granuloma has an unknown etiology, with some dispute as to whether this lesion represents a reactive or neoplastic process. However, most authorities believe the peripheral giant cell granuloma is a reactiv lesion. Local irritation due to dental plaque or calculus, periodontal disease, poor dental restorations, ill-fitting dental appliances, or dental extractions has been suggested to contribute to the development of the lesion. They are seen exclusively in the gingiva, usually in the area between the first molars and the incisors. See picture #63 in booklet

Peripheral giant cell granulomas typically present as red to blue broad-based masses. They usually bleed easily and may occur at any age and tend to be seen more frequently in females than in males.
When this process occurs on the edentulous ridge, a superficial, cup-shaped radiolucency may be seen. Histologic sections are diagnostic and are identical to those of a central giant cell granuloma. The lesion is composed of a proliferation of spindled fibroblasts in a stroma containing variable amounts ofcollagen. Multinucleated giant cells are present throughout the connective tissue stroma. Conservative excision is typically curative, although the lesion must be completely removed to prevent recurrence. Generally, this lesion is clinically indistinguishable from a pyogenic granuloma. -{lthough the peripheral giant cell granuloma is more likely to cause bone resorption than rs the pvogenic granuloma, the differences are otherwise slight. Biopsy will provide detinitir e results.

\ote:

Hcmangiomas are benign vascular tumors composed ofcells that nonnally line the blood vessels rentlothelial cells). They arc the most conmon tumor ofchildhood, occurring in up to l0% ofin1ants. Henangiomas may be visible at birth or may not be recognized until the first few weeks or cr en months of lif'e. ln general, however, most hemangiomas becomc evident by 2 to 3 weeks 01' age. -\lthough they comrnonly occur on the head or ncck (60%). they can occur in any region of rhc bodl ). Some lesions are small and hardly visible, whereas others are large and rcadily obscrv:b1c'. \lost hemangiomas appear as single tumors, though l57o present with more than one lesion.
1 . Congenital hemangioma (slrawberry revr.r'./: local proliferation of capillaries; exhibit a rapid growth phase followed several years latcr by an involution phase; persist-

\otes

ent lesions are excised.

2. \'ascular malformation: a pcrsistcnt malfonnation of blood vessels; do not involutc; exhibits a thrill (palpate a pulse) andbrdt thear a pulse). A typc ofvascular nallomation is known as Sturge-Weber disease (encephalotrigeminul angiomatosis) w hich consists of a facial lcsion. known as the port-rvine stain, which is distributcd over thc trigeminal nervc accompanied by a similar vascular disorder ofthe underlying meninges and cerebral co ex. lt usually occurs unilaterally. 3. Remember: Rendu-Osler-weber syndrome, also known as hereditary henlorrhagic telangiectasia, is a congenital hcreditary lbrm ofhemangioma. 11 is charactcrizcd by numerous spider-like telangiectases on thc face, neck, chest. lips, gingiva, buccal mucosa and tongue. One of the earliest signs of the disease is epistaxis (nosebleeds). Arteriovenous listula, especially ofthe lungs and liver, are a variable component. Bleeding from thc telangiectases may bc rcculrent, lil'e{hreatening and increase in sevcrity rvith aging. The onsct oftcn is in childhood. Both sexes arc aflected equally. 4. lmportant: How to distinguish between a hemangionra and a hcmatoma -hemangioma will blanch on diascopy. hematomas do not blanch.

. Uncommon and represerfiz4yo ofhead and neck neoplasms

. Common

and represent 75-80% ofhead and neck neoplasms


25-30olo

. Uncommon and represent

of head and neck neoplasms

. Common

and represent 95-98% ofhead and neck neoplasms

132
Coplright
@ 201 I

-2012 - Denral Decks

. White sponge nevus . Lichen planus . Necrotizing sialometaplasia . Focal hyperkeratosis

133 Coplright O 201l-2012 - Denral Decks

Salivary gland tumors may be broadly categorized into benign neoplasms, tumorJike conditions, and malignant neoplasms. The glands are divided into major and minor salivary gland categories. The major salivary glands are the parotid, the submandibular. and the sublingual glands. The minor glands are dispersed throughout the upper oral-digestive submucosa (i.e., palate, lip, pha4nx, nasopharl,rrx, latynx, paraphatyngeal space).

Benign Salivary Gland Tumors:

. Pleornorphic adenoma lbenign

mLxed tumor./ is the most common tumor of major and

minor salivary glands. It is a mixture of epithelial and mesenchymal elements (pleomorphi.).85yo occnr in the parotid gland. Within the oral cavity, the majority are found in the
palate. Pathologically, it is characterized by slow growth and few symptoms. . Monomorphic adenomas: The term "monornorphic adenoma" refem to a group ofrare salivary tumo$ that includes the basal cell, canalicular, sebaceous, myoepitheliomas and oncocytomas. Ofthese, the basal cell adenoma is the most common.

. Warthin's ttrnor

(papillary' .!-stadenoma lymphomatosnn) is almost exclusively

parotid neoplasm. It is a glandular and cystic tumor lined by

a bllayered (inner columnar

otlcoc)lic qnd outer ba.sal) epithelium with a lymphoid stroma. . Benign lymphoepithelial lesions include a wide range of cystic changes that share the common denominator in atypical lymphoid hyperplasia. Hyperplasia refers to a proliferation ofcells that is non-neoplastic. These changes are found often in patients infected with
H

I\i

-\.'ot

L Clinical features of benign salivary gland tumors: mucosa is normal, painless, nodular, movable, fifm. and slow-growing. 2. The most common site ofintraoral minor salivary gland neoplasms is the palat. 3. The most common site of intraoral major salivary gland neoplasms is the parotid.

\ecrotizing sialometaplasia is a recognized lesion of the minor saliYary glands characterized by necrosis ofthe glandular parenchyma with associated squamous metaplasia and hyperplasia ofthe ductal epithelium.
Important: The initiating event ofnecrotizing sialometaplasia is believed to be related to ischemia, secondary to alteration of local blood supply. lnfarction of the salivary gland tbllou s. presumably due to compromise ofthe vascular supply.
It is usually observed in adult rnales and presents as an asymptomatic, necrotic. ulcerated area inr,olving the palatal mucosa. The hard palate appears to be the most common site.

Histologicall!', there is lobular necrosis of the glandular parenchyma, with squamous metaplasia and hlperplasia of the ductal epithelium. Both clinically and histologicallS the lesion may sin.rulate a malignancy and in the past, the condition has been misdiagnosed as a squamous cell carcinoma or mucoepidermoid carcinoma.

Following biopsy and the establishment ofthe diagnosis, further treatment generally is not recommended since healing usually occurs within 6-10 weeks.

. Measles

. Mumps
. Rubella

. Chicken pox

1U
Copyright O 20ll-2012 " Dent.l Decks

. Parotid gland tissue . Submandibular gland tissue . Sublingual gland


.
tissue

All of the above

l3s
coplright
@

20r I -20 12 - Dental Decks

Mumps is an acute viral illness. The mumps virus is a paramyxovirus in the same group
parainfluenza virus. The virus is acquired by respiratory droplets. It replicates in the nasopharynx and regional lymph nodes.
as the

Important: Mr.rmps is the most frequent diagnosis of acute viral sialadenitis, mostly in
the parotid gland.

Clinical: . 907o ofthe

cases occur before l4 years of age . A major sign is sudden salivary gland swelling without purulent discharge from the duct *** Parotid involvement and bilateral in two-thirds of the cases. -90o%and anorexia . \4iltl ferer. malaise

. Most cases are self-limiting

Complications:

. Orchitis (inflqmmation of the testis) and epididymitis males. *** Important: May cause sterility
. CNS

can occur in post-pubertal

. Dealness. myocarditis, pancreatitis, oophoritis

systeln

rneningitis and encephalitis


and pyelonephritis

The serum amylase may be elevated during the acute phase. Prevention with a live attenuated vaccine is 95% effective for at least five years. However, in non-inoculated indir iduals, it is still a cause ofacute non-suppurative saliyary adenitis.

The stafne bone cavity or cyst is a developmental anomaly represented by a bone conitl usually containing submandibular gland tissue. The posterior mandible region, pafiicularly at the angle and below the mandibular canal is the most common location. The sraric bone cyst is usually discovered incidentally on dental radiographs. It is asymptomatic and is not a true cyst. rather an anatomic depression in the lingual aspect of the bodl of the mandible where normal salivary gland tissue rests. The radiographs show a small, circular, corticated radiolucency below the level of the mandibular canal. Histologicalll', nonr.ral submandibular salivary gland tissue is found and no treatment is required except routine radiographic follow-up. Note: Many other terms have been used to describe this entity, including submandibular salivary gland depression, lingual salivary eland deoression. and Stafne bone cvst.
car

Stafne bone cyst in characteristic location inferior to the mandibular canal

ORAL PATHOLOGY

SG Thm

A 53-year-old woman comes into the dental clinic with bilaterally enlarged parotid glands. It was discovered that she had recently ben to the African continnt and had contracted tuberculosis. What is the name of the autoimmune disease associatd with enlarged salivary glands in association with a secondary disease?

Sjogren's syndrome

. Mikulicz's disease . Gorlin-Goltz syndrome . Pierre-robin syndrome . Apert syndrorne


136
Copyrighr
aq

20ll l0ll - Dcnbl

Decks

ORAL PATHOLOGY

SGTirm

A 33-year-old patient comes into your office for a routine maintenance appt. While doing an intraoral exam, the hygienist discovers a bluish lesion ofthe lower lip. The patient relates a history of biting this area last week whn he had a sinus infection. What is the most likely diagnosis ofthis lesion?

. Ranula

. lnfectious sialadenitis . Maxillary sinus retention cyst


. Mucocele

137 CoDrighr
aq

20ll-2011- D.ntal

Decks

The term "benign lvmphoepithelial lesion" (also called Mikulicz's disease) is manifested essentially as a progressive, asymptomatic enlargement of the parotid and submandibular glands. It is initially unilateral, but over time, it becomes bilateral. The etiology is unknoun, however, there is increasing evidence that both Mikulicz's disease and Sjogren's syndrome are both actually autoirnmune diseases in which the patient's o\\ n salivaD gland tissue becomes anligenic. The benign lymphoepithelial lesion is rare. It occurs most frequently in middle-aged rvomen. Histologically', there is replacement of the gland parenchyma by lymphocytic inliltrate rvithin which there are scattered epimyoepithelial islands. This is tl.re histologic comerstone lor the diagnosis of the benign lymphoepithelial lesion.

differential diagnosis ofchronic bilateral enlargement ofthe salivary glands as a result of benign lymphoepithelial lesion must be separated for similar findings seen in sarcoidosis. lymphoma, gout. leukemia, diabetes mellitus, chronic alcohol abuse, and, rarely
The

hlpenension.

Important: Most of these lesions remain benign. however. malignant transformation


of the epimyoepithelial islands has been demonstrated.

\lucuse\tralasationphenomenon,commonlykno\\nbytheclinical

term, "mucocele", is scparrtcd from

mucus retention c!-st bccause each has a disdncrivc pathogenesis and microscopy. Scveral clinical features rl.,r .il.a:ale thcsc lcsions. . \lucus ertravasation phenomenon: is considered to be related to rnechanical lrauma to thc minor sali\ 3a e\cretory duct. Although the lower lip is thc most frcqucnt sitc. the buccal mucosa, vcntral sur_ -!lalrd ficr- Lrf thc rongue (\'here the glanls of Blondin Nuhn arc locdterll. lloor ol thc rnouth, and reiromolar :r!iLrr rrc ofien atfccted. These lesions are usually painless and smoolh surl'accd, \\'ith an o\eral1 bluish hue !rf rranslucenc) associated \lith a lnore superficial localion. The trcrtment ofchoice firr mucus extravasr:r!r: r!hcnonrenon is surgical cxcision rvith rcmoval ofthe associatcd gllnd. . \luaur retention cyst: is regarded as a cyst becausc it is lincd by an epithelium unlike mucus exi::r\ 3sairon phcnomcnon. r.hich contains mucus pool surrounded by granulation tissue.This cyst resuhs tirm obtnu.lion of sali\ary flo\l', The mucus letention typc mucocclc is less common than the extravasa:ri:] \rrc Thc rcrcntion t-vpe usually appcars aftcr 50 ycars ofagc, and it rarely prcscnts in the lorver lip. I:r.iead it is found in the palate, cheek, floor ofihe mouth. as wcll as in the nraxillary sinus. Clinical prcs.::]iron i\ characicrized by an asympbmatic s$elling, oflen without previous injury. The Icsions vary l:L.nr I to l0 mm and. on palpation. are rnobile, non-tender and lhe overlying mucosa is intact and ofrormal color. Thc trca{ment ofthe mucus rctcntion cyst inchldcs surgical excisron *'ith rcmoval ofthe asso;ra:ed gland.

lmportant: "}Iucoccle" is uscd in the clinical setting

as a generic term (hclbrc nicroscopic didguo.ri.r) to refer to both the mucus retention cyst and the mucus extravasation phenomenon. L The maxillary sinus retntion cyst and pseudoclsts are commonly discovcrcd incidentally, \ote3 ofren being ofgrcatcr curiosity than clinical significance on panoramic radiographs. May repre. sent b lockage o f sinu s sal ivary gland, or focal fluid accumulation of sinus mucosa. These lesions

'

arc asymptomatic and require no treatment. :. Infectious sialadenitis: int'ections ofthe salivaF"' glands tc al.

ftav

be acute or chronic.

viral or bac-

. \'iral:
- Nfumps is the most frequent diagnosis ofacute viral sialadenitis, mosdy in parotid. - Cytomegalovirus infections are chronic; usually seeD in immunosuppressed patjents . Bacterial: the most commonly isolated organisms arc Staphylococus aurcus. Strcptococcus. viridans. II. intluenzae. and E. coli.

What is the most probable diagnosis for a lesion that presents as a translucent, bluish, well-rounded, smooth-surfaced bulge that protrudes from one side ofthe floor ofthe mouth?

. Adenoid carcinoma

Squamous cell carcinoma

. A ranula

. A lymphangioma

138
Cop)aighr
iar

201l-2012 - Dental Decks

ORAL PATHOLOGY

SG Thm

A patient comes to your ollice complaining of pain when eating and even sometimes when thinking about food. Your intraoral exam reveals a small,

hard swelling in the floor ofthe mouth. A mandibular occlusal radiograph shows a per-sized radiopacity with 'onion-skin" thickening lingual to the right mandibular border. Name the likely diagnosis:

. .

Sialometaplasia Sialadenitis

. Sialolith
. Sialosis

139

Coplriglit C 2011-2011 - Denral Dects

Ranula, is

a clinical term that is used to designate a mucocele that occurs specifically in the floor ofthe mouth (see picture below). Pathogenetically and microscopically, it may represent either mucus extravasation phenomenon or mucus retention cyst. The ranula is associated with the duct system ofthe sublingual salivary glands, and, less commonly, the submandibular glands.

The ranula is usually caused by an obstruction produced by either a salivary stone or soft organic substance. It usually presents as a painless, fluctuant, unilateral, soft tissue mass in the floor ofthe mouth. It t)?ically has a bluish appearance that has been compared to a frog's belly, hence the term "ranula."

Of diagnostic significance is a history of increased size just before or during a meal,


and a decrease in size between meals. The treatment is surgical, either through complete excision or by removing the roof ofthe cysl. If it persists, excision ofthe gland may be
needed.

A sialofith is a stone (salivary calculus) within a salivary gland or duct. The formation of a sialolith is called sialolithiasis and occurs as a result of precipitation of calcium and phosphate salts around a nidus ofmucous or bacterial debris. Sialoliths occur as single or multiple stones and can cause swelling and pain. The pain is experienced during salivary stimulation and is intensified at mealtimes.
The best radiographic projection for visualizing sialoliths in the submandibular duct and gland is the standard mandibular occlusal view. Occasionally, sialoliths are seen incidentally on periapical radiographs, in which case they may be misdiagnosed as osteosclerosis.

The rate of occurrence in submandibular gland and duct is much higher than in the parotid or sublingual areas. This is thought to be due to the tenacity ofthe submandibular saliva and the long and irregular shape of the duct. See picture #64 in booklet
The treatnent ofchoice is almost invariably surgical

extirpation ofthe sialolith. Stones

located in the glandular parench;.rna usually require removal ofthe gland as well.

Of the neoplasms rffecting the mNjor or minor


glands;

the

is the most common.

. Basal cell adenoma

Sebaceous adenoma

. Pleomorphic adenoma
. Ductal papilloma

140 Coplright O 201 I -20 l2 - Dnral Decks

. Sarcoidosis
. Mikulicz's disease

Sj<igren's syndrome

. Hypothyroidism

. Diabetes mellitus
. Malnutritior/Starvation . Dehydration

. Cystic fibrosis

141
CopJaight @ 201l-2012 - Dnral Decks

Pleomorphic adenoma (benign mixed tumor, is the most common tumor of major and minor salivary glands. It is a mixture of epithelial and mesench).,rnal elements (pleomorphic).Int\e parorid, 90% occur in the superficial lobe and most conmonly are seen in the tail ofthe gland. Within the oral cavity, the majority are found in the palate. Pathologically, it is characterized by slow growth and few symptoms. See pictures #65 and #66 in trooklet The gross pathologic appearance of a pleomorphic adenoma is a smooth or lobulated.
well-encapsulated tumor that is clearly demarcated from the surrounding normal salivary gland. They are tlpically solid tumors and may have areas ofgelationous myxoid stroma. Cystic degeneration or tumor infarction and nectosis are rarely seen except in large, longstanding lesions. Microscopically, these tumors are composed of varying propoftions of gland-like epithelium and mesenchymal stromas. The treatment ofchoice is surgical excision with a generous margin ofnormal tissue. Inadequate initial removal ofthe mixed tumor in major glands may result in recurrence. Approximately 25% ofbenign mixed tumors rvill undergo malignant transformation illesions are untreated for an extended length

of tirne.
Sebaceous adenomas are rare lesions composed predominantly of sebaceous gland-derir ed cells; they are well differentiated in the benign forms. The parotid gland is the site of chief involvement. Parotidectomy is the treatment ofchoice when lesions arise in this sland.

Ductal papillomas are a rare group ofbenign papillary neoplasms ofthe large excretory duct. There are three types major subtypes of ductal papillomas of salivary gland origin
thar are about equally rare: the intraductal papilloma, the inverted ductal papilloma, and the sialadenoma papilliferum. The treatment for all three is conservative simple local excls1(]n.

There are many conditions associated with parotid gland enlargement, this may be unilateral or bilateral. . Sjtigren's syndrome: is an autoimmune disorder affecting lacrimal and salivary glands \\ hich causes decreased moisture in glands. Dry mouth, tooth decay, mouth sores, enlarged salir ary glands, sialoliths, and recunent salivary gland infections are possible symptoms. The syndrome also et'fects moisture in the eyes, which might cause chronic eye infections, comeal ulcers and vision loss. . Sarcoidosis: unknown cause; believed to be alteration in cellular immune function and in\ ol\'ement of some allergen. Most often involves the lungs; can affect the parotid gland. Granulomatous fmacrophage infiltates) inflammation causes organ nodularity and loss of parenchyma. Note: Sarcoidosis is also characterized by distinctive laboratory abnormalities, including hyperglobulinemia, an elevated serum angiotensin converting enzyme level and hypercalcemia. Glucocorticoids remain the mainstay oftherapy when treatment is required. although other anti-inflammatory agents are being used increasingly often. . \\'arthin's tumor: also called papillary cystadenoma lymphomatosum . Infections: for example, mumps, actinomycosis, tuberculosis . Benign lymphoepithelial lesion: also called Mikuticz's disease

. l\4alnutdtion/Starvation . Dehydration . Cystic fibrosis and diabetes mellitus

\letabolic conditions associated with bilateral parotid gland enlargement include: chronic alcoholism, dictary deficiencies, diabetes mcllitus, hypertcnsion, hyperlipidemia and Sjdgren's syndrome.

The

lcinic cell crrcinoma

is derived from serous acinsr cells

and is fouRd almost exclusively in the:

Submandibular gland

. Parotid gland . Minor glands of the palate

. Sublingual gland

142
Coplrighr O
201 1-201 2

, Dnial Decks

. Adenoid cystic carcinoma . Mucoepidermoid carcinoma . Acinic cell carcinoma . Polymorphous low-grade adenocarcrnoma

1/f,3

Copt'righr O

201 I

'2012 - Dental Decks

the second most common parotid malignancy and the second most conlmc\n salivary gland ma\gnancy (nucoepidermoid carcinoma is lhe nost conmonlbr both). CrL," ^:i:rlnc appearance demonstrates a mass ihat is well-circumscribed but lacks a true caps[le. Microscopic appearance has been categorizcd as solid, microcystic, papillary cystic and follicular. Tumor cells are dark staining and have granular or honeycomb cytoplasm. The surrounding sffoma often demonstrates a lymphoid infiltrate. Treatment of acinic cell carcinoma includcs surgical cxcision. This tumor is gcncrally regarded as a low-grade malignancy.

\cinic cell carcinoma is

not olheheise specirted) ofthe salivary glands are rare but aggressive tuAdenocarcinomas (NOS present in the parotid glands. Minor saiivary glands on the palate, lip mors. About halfofthese trmors and tongue are the next most commonly affbctcd sitcs. Adenocarcinoma is different from other salivary gland neoplasms in that as many as 2570 ofpatients will complain ofpain or facial weakness at presentalion. Gross pathology reveals a firm mass with irregular bordcrs and infiltration into surroundin-e tissue. It is generally a solid tumor wjthorit any cystic spaces- These malignancies can demonstrate a wide range ofgrowth pattems and, for lhis rcason, can bc somewhat difficult to classily. However' all adcnocarcinomas have in common thc formalion ofglandular structures and they are described as grades I. II or lll based upon thc degrcc of ccllular differcntiation. Grade I lesions havc wcll-formcd ductal itruclures \\hile Grade III lesions have a more solid growth pattern with few glandular chamcteristics.
Bccausc thcsc are more aggressive tumors, treatment for adcnocarcinoma is more aggressive. Complete local excision is the mainstay oftherapy.

\lalignant Mixed Tumor


clude the:

rcpresents thre separate entities that are histologically distinct; These inis the most oommon

. Carcinoma ex-mixed tumor

ofthe three salivary ncoplasms. It occurs when ofa preexisting pleomorphic adenoma. . Carcinosaacoma or true malignant mixed tumor: thc mctastatic lesions contain both the stromal
a carcinoma develops from the epithelial component

end epithelial elements. This is difl'erent from the carcinoma ex-mixed tumor in which only the ep'rl_alial clcmcnti are prc5cnt in mclaslasii.

. \Ietrstasizing mixed tumor


r eloos metastatic deoosits

refers to an otherwise benign acting pleomorphic adenoma that de-

oftumor

\tucoepidermoid carcinoma is the most common malignant salivary gland neoplasm in both minor and rNaior glands. It devclops rnost commonly in thc major salivary glands, mosl oftcn the parotid (45-70'/o) Tha sccond most common sitc ofoccurrence is the palate //8Zr). Thcse tumors grow slorvlY and preslr:: as painless masses in most cases. Nlicroscopically, thcsc fumors are charactcrized by the presence
..: t\\ o populations ofcells the mucus cells and the epidermoid cells. the proponion of rvhich helps to i3ine thc grade ofthc txmor. They arc uncncapsulated or poorly encapsulated and intlltrate srirround_

::r!lirsuefreelt.'High-grade"and"lowgradc"formsexisteachwithadifiercntprognosiskoodforloy'

!,ttti.

poor.fot high-g^rrle/. Surgical cxcision is the ffeatment. See picture #67 in booklet

.\denoid clstic carcinoma /'1CC/ accounts for approximately 23% ofall salivary gland carcinomas. App:o\iDrarcl) 50 to 709/0 occur in the minor salivary glands ofthe palate- In the maior salivary glands. the
prrotrd gland is nrost often affected. Microscopic appearance is described as cribriform, rubular or solid Th. cribrifonn pattem is the most common and most easily rccognizable. It is often referred lo as the _s\riss cheese" pattem. Tumor cells arc armnged in ncsts around cylindrical spaces that may contain a nucrnous or hyalinized material. Cells lhat are arranged in layers and form ductal struclures character_ lzi rhc i.rbular pattem. Thc solid pattem contains sheets offumor cc]Js u'ith lo intent'ning spaces.
See

picture #68 in booklet

.\CC is an unusual tumor It is slow-growing but relentless. It tcnds to be locally invasive and infiltrate rhe 'iheaths" or coatings surrounding nerve fibers (per-lneurol spaces). ACC often rccuN years later at ihe site \\here thc fumor firct arose or it mny metastasize. Unlike most carcinomas, it seldom metastasizes to nearby lFnph nodcs but rather to distant sites. The lung is thc most common site ofmetastasis, \\ rth rhc liver second. Bone metastases indicate a poor prognosis. PofJmorphous low-grade adenocarcinom (PLGA) is the second most common malignancy in the minor salivary glands and occurs most frequently in the palatc. lip and buccal mucosa. The microscopic appearance ofthcsc tumors is what gjves them their name. Any ofa variety ofgrowth pattems (solid, tu' hula4 trubecular, glandular tribriforn, cysti., can be seen within the same lcsion or among different lesions. PLCA displays a tendency for perinerual and perivascular invasion, howevcr it typically follows an indolent course. Treatment consists ofconscrvativc yct complete local excision. Distant metastasis is
not common,

. Lupus Erythematosus

'

Sjogren's disease

. Sarcoidosis . Crohn's disease

Coplrighr

201

141 l'2012 - Dental Dcks

. Malignant
. Mixed ftenign and malignant) . Carcinoma in situ . Benign

'145

CopltiSht O 20ll-2012 - Dental DcIs

Sjiigren's syndrome is a chronic lymphocyte-mediated autoimmune disorder aIi'ecting the lacrimal and salias well as other organ systcms. Thc paticnts most commonly afTected arc post-mcnopausal women who prescnt with dry eycs. dry mouth and, in about 50% of the cases, enlargemen! ofthe parotid and submandibular gl^nds bilarerally. Primary Sjdgren's syndrome consists ofxeroslomia and kcratoconjunctivilis
vary glands

stcc (dry eres), and possibly salivary gland enlargement. Scondary Sjcjgren's syndrome has the same xerostoma and keratoconjunctivilis sicca but is also associated with anothcr autoimmune disordr such as R-{, SLE, etc. There is dense inflammatory infiltrate $ ith destruction of glandular tissue- Treatmenr is palliative.
Note: Biopsy ofthe labial or palatal salivary glands may bc helpful in establishing the diagnosis, along with sialograms, salivarj" flow rate tests , the Schimer test ldeterniles \lrcther the e),e procluces enough tears to *eep it moist) work. Important: Only halfofSjdgrcn paticnts have anti-SSA and/or anti-SSB /SJd^ndblood grcn\ Svndronte A dnd B,) antibodies in their blood. Other tests which may be abnormal include the white blood cell count //ot . total gamna globulin level /rr'll. blood C3 and C4 complemen! levels /hx C4l. sedimentation rate frigrl aud rheumatoid factor lporltlr?r.

Important: The decrease


nign l)
It is
m ph

ir1

salivation may cause rampant caries reminiscent ofradiation cariqs.


f'eafures

Important: The histological

ofthe salivary gland lesions in both Sjitgren's syndrome and the "be-

opithfial lesion" lalso calle.l

Milrll.:

./rs?.rr., are idcntical.

ponant to remenlber that malignant lymphomrs and "pseudolymphomas" (also cdlled at picdl benign lvehoid h.r!)erpld.rld) develop in some patients who have been diagnosed *ith Sjcigren's syndrome. This

mandarcl clo$ follo$-up oflhe parienl!. Lupus er]thematosus is a chronic autoimmune disease that occurs in tivo forms: . Discoid lupus erythem^tos\s (DLE)| is I chronic skin condiiion of sorcs with inflamrnation and scarring fa\oring the facc, can, scalp, and/or oral mvcosl (i.e., buccdl nucosa. gingivd, rcr illion). Thcse lciions delelop as a red, inflamed patch with a scaling ilnd crusty appearance. Oral lesions mimic erosive fichen planus. Trcated with cortisone or other drldgs (i.e., Pldqucnil. Aralen, or Quinacrine).
. St stemic lupus erythematosus (SIrr: is a chronic inflammatory connectivc tissuc disorder that involvcs muhiplc organs including the kidneys. heaft,joints, skin, mucous membranes, and blood vessel walls. Abutt.rll)-shapcd rash /ma1dr rdrrl on the t'ace that covers the cheeks and bridge ofthe nose is a c)assic sign\ole: \umercus autoantibodics direcled against nuclcar and cytoplasmic antigcns are fbund in SLE palients. Oral lesions are generally similar to thosc secn in DLE. Scrologic rcsis (ANA [aubantibodie.\J test and LE icll re-vr are Dositive in Datients with SLE.

.\pproximatel,v 8 out of l0 salivary tumors diagnosed are in a parotid gland. One in 10 diagnosed is in l submandibular gland. Thc remaining l0% are diagnosed in othcr salivary glands. In gneral, glands morc likcl! to show tumor growth are also glands least likcly to show malignant nrmor growlh Thus. elihr.ugh tumors ofthe sublingual glands arc rarc, almost all ofthem are malignant. In contrast, only rl.rlrt Iion ofparotid gland tumors are mahgnant.

Clinical leatures of malignant salivary gland tumors: - Jlucosa is ulcerated - Firm - Painful - Nodulas

Fixed

Rapid growth

\lafignant Safivary Gl^ndTumors (b eflt)t

\denoca rcinoma,

OS lnot olherv,ise specified), arc rare but aggressive tumors About half of

rh.se tumors present in the parotid glands. Adenocarcinoma is different fiom othcr salivary gland neofla5ms in that as many as 259lo ofpaticnts rvill complain ofpain or frcial rreakness ai presentatlon. {denoid c!,stic carcinoma: approximateiy 50 to 7oyo occur in the minor salivary glands of the talate. In the major salivary glands, the parotid gland is most often affected. Microscoplc appearance is described as cribriform, tubular or solid. The cribriform pattern is the most common and most carily recogrizable. It is often refened to as the "swiss cheese" pattem. . \lucoepidermoid carcinoma: is the most common malignant salivary gland neoPlasm in both

'

minor and major glands. lt develops most commonly in the major salivary glands. nost otien the patotid G5-70a.,'). Microscopically, these tumors arc characterized by the prescnce of two populations ofcells thc mucns cells and the epidermoid cells, the proportion ofwhich helps to define lhe grade of thc tumor . Acinic cell carcinoma: Acinic cell carcinoma is the second most common parotid malignancy and thc second most common pediatric salivary gland malignancy ih&coep[dermoid carcinona is lhe osl t'onnnon lor hoth).This tumor is gcncrally regarded as a lort-grade malignancy. .llalignant mixed tumors: represents three scparate entities that are histologically distinct; These
includc thc carcinoma ex-mixcd tumor, carcinosarcoma or true malignant mixed tulnor, and thc metaslasizing mixed tumor. . Polymorphous low-grade adenocarciliotn (PLGA) is the second most common malignancy in the minor salivary glands and occurs most frequently in the palate, lip and buccal mucosa. The microscopic appeamnce ofthesc tumors is what gives them their name.

ORAL PATHOLOGY

Oncocytomas are tumors that constitute about of benign epithelial salivary gland neoplasms.

. Common; 50o/o
. Common; 75%

. Rare; 2%
. Rare;
l5olo

'146

Copyright C

20ll-l0l:

- Denkl Dects

ORAL PATHOLOGY

A 65-year-old patient comes to your oflice complaining of a slowly-growing enlargement ofthe jaw. You palpate the angle of his right ramus and find an encapsulated mass that is non-tender and firm. Your oral pathologist defines it as a glandular and cystic tumor lined by a bilxyered (inner columnar oncoc!,tic and outer basal) epithelium with a lymphoid stroma. Name this second most common benign neoplasm of the parotids.

. Pleomorphic adenoma . Warthin's tumor . Fibroadenoma . Monomorphic adenoma

147

CoDrigllr t) 20ll-201? - Dental Decks

The term "monomorphic adenoma" refcrs to a group ofrare salivary tumors that includes thc basal cell adenomas, canalicular adenomas. scbaccous adenomas, myoepithcliomas and oncocytomas.

Oncocytomas arc rare tumors that constitute only 2% of benign epithelial salivary gland neoplasms Thc majorjty ofthese tumors affcct lhc parottd gland 178%). The clinical prcscntation ofoncocyomas is esscntially identical to other bcnign salivary tumors a slowly growing, nontender mass. typically
in the superficial lobe cytes.

ofthe parotid. Microscopically, thcrc arc shccts,

nests or cords

ofuniform onco-

Basal cell adenomas are also rare tumors that constitute only J'lo ofbcnign epithelial salivary gland !umors and gpically occrrrs in the 6th decade oflife. It seems to occur morc frequently arnong Caucasians than African Amcricans. The majority occur in the parotid gland where thcy present as a slowly cnlarging firm mass. They are well-encapsulated, smooth tumors on lross inspection and arc divided into four subtypes based on their microscopic appearance solid, trabecular, tubular and membranous

Canalicular adenomas are also rarc ftrmors (1.596) that most commonly involve thc minor salivary glands of the npper lip (74yu) or brc.al mrcosnl ( 12a;).It peaks in thc 7th decade of life and, like tlle
basal cell adcnoma, is morc common in whites than blacks Clinically it presents as a nonpainful submucosal nodule. On gross pathologic examination, canalicular adenomas may or may not possess a cap_ sulc and it is not unusual for therc to bc multifocal glowth. Microscopically there are cords ofsingle-layer columnar or cuboidal cells forming duct-like structures in a background of fibrous stroma Thc rarc m!oepithelioma accounts for less than one perccnt ofall salivary gland ncoplasms. Most arise \! irhin the parotid gland and, less frequcntly, in the submandibular gland and intraoral minor salivary glands. Three paftems ofmicroscopic appearance have been describcd. The spindle cell pattern is the mosl common ovcrall and is typical for parotid myoepitheliomas. The plasmacytoid pattern is less common but the most frequently encountered pattern in palate nlmors. Tle third pattern demonstrates 3 combination ofthc spindle and plasmacytoid cells and is uncommon.

\ot: Sialoscintigraphy
rres

is a simplc and non-invasive procedurc that can usually scparate benign enti-

like \\'arthin's tumor and oncocytoma ofthe salivary glands from maligrant tumors, and signifiaanrlv affcct the course oftrcatment.

c! ameforWafthin's tumorl ls almost exparotid neoplasm. lt is a glandular and cystic tumor lined by a bilayered (inner .olunindt on.oc)lic and outer basal) epithelium with a lymphoid stroma.
Papillar)- cystadnoma lymphomatosvm (afq
cltLsir elv a

Clinical features: The vast majofity ofpatients are over 50 years ofage, with a 5:l male predc'minance. Approximately l0To to l5Yo of the tumors are bilateral. The tumor most often
arises in the lorver pole ofthe parotid and presents clinically as a non-tender' slowly enlarging. flmr to fluctuant nodule oyer the angle or ramus ofthe mandible.

Histologic features: The tumor is encapsulated and co[tposed ofcystic spaces containing an .r)sinophilic coagulum into which extend papillary projections of the lining epithelium. The epithelium consists ofa double row ofcells with eosinophilic, granular cytoplasm, a luminal
lay er oftall columnar cells and a basal layer ofround, cuboidal or polygonal cells. lnterspersed among the cystic spaces are aggregates of lymphoid tissue, some with germinal centers.

Treatment and prognosis: Surgery is the treatment ofchoice and recurrence is uncommon. \lalignant vadants ofthe tumor have been reported but are mre.

\ote: Pleomorphic adenomas are the most common parotid tumor lt grows slowly and is benign. A pleomorphic adenoma begins as a painless lump at the back ofthe jaw just below rhe earlobe. These are more common in women.

A 40-year-old female presented with a subcutaneous nodule on the right latcral surface of her tong\e (afiort is poinrtng tu lesion). The nodule was non-tender, soft, movable and had been slowly growing for about 2 years. The nodule was excised. Its eut surface was yellow and lobulated. What is the clinical signilicance ofthe nodule being movable and slowly growing?

148

CopriSlrr O

20ll ?01?

Denral Decks

ORAL PATHOLOGY

A developmental dfect characterized by an overgrowth of tissus NORMAL to the organ in which it arises is a:

. Teratoma . Choristoma

. Hamartoma
. None ofthe above

't

49

Copyrighl e 201l-2012 - Denral Decks

Gcncral charactcristics of benign neoplasms:

. . . .

Well-diffcrcntiated Slow growth


EncapsulatedAVell circumscribcd Localized

Gcncral charactcristics of malignant neoplasms:

. Invasion

. Immovable
. Rapid growth . Metastasis . Not well-differentated (or qnaplqstic)

Important: . Paresthesia is suggestiv of metastatic

disease

. Metastasis is the most important characteristic that distinguishcs malignancy from benign Radiographically, a benign neoplasm in bone may be diffcrcntiated ftom a malignant neoplasm in the following rvays:

. .

ln a benign lesion, the cortex tends to remain intact but may be thinncd and thc pat

involved expanded ln a benign lesion, the margins are usually defined and demarcated lrom surrounding
The nodule in the picture on thc front ofcard is a lipoma.

bone

***

Te

rms used in oncology:

. Oncology: the study ofneoplasms . \eoplasm: an uncontrolled new growth oftissue


. Tumor: a Iocalized swelling, may or may not be a true neoplasm . Hlperplasia: an increase in the size of a tissue or organ due to an increase in the nurnber of component cells . H]'pertrophy: an increase in the size of a tissue or organ due to an increase in the
size of component cells . Cancer: a general term for all malignant neoplasms . Carcinoma: a malignant epithelial neoplasm . Sarcoma: a malignant mesenchymal (connective /i.tsae,,) neoplasm . Hamartoma: a developmental defect characterized by an overgrowth oftissues normal to the organ in which it arises . Teratoma: a neoplasm composed of multiple tissues foreign to the organ in which it arises; may be benign or malignant

. Metaplastic

. Hwerplastic
. Araplastic

. Dysplastic

150 Cop)r'ghr O
201

l-2012 - Dntal Dcks

Metaplasia most commonly occurs by rephcement

of

. Cuboidal cells; columnar cells . Columnar cells; stratified

-bv

squamous epithelium

. Columlar cells; pseudostratified colurnnar cells

. Cuboidal cells; stratified squamous epithelium

151

Cop)rigltt O 20ll-2012 - Denral Decks

Lack of differentiation, or anaplasia, is considered a hallmark of malignant transformation. It is marked by a number ofmorphologic and functional changes. Both the cells and the nuclei characteristically display pleomorphisrn variation in size and shape. Characteristically the nuclei contain an abundance of DNA and are extremely dark staining (hyperchromatlc). The nuclei are disproportionately large for the cell, and the nuI :l instead of the normal 1 :4 or 1 :6. The nuclear shape is usually extremely variable, and the chromatin is often coarsely clumped and distributed along the nuclear membrane. Large nucleoli are usually present in these nuclei.

clar-cytoplasmic ratio may approach

Histological grading of malignant neoplasms:

. An

attempt to estimate the aggressiveness of degree of malignancy of a rnalignant neoplasm based upon the degree of differentiation of the component cells and the number of mitoses.

Grade

l. Well-differentiated

Grade 2. Moderately well-differentiated Grade 3. Poorly undifferentiated Grade 4. Undifferentiated

.Applicable mainly to squamous cell carcinomas. Most pathologist use three grades and prefer to designate squamous cell carcinomas as well differentiated, moderately well-differenriared or Doorlv differenliarea.

\Ietaplasia is the process whereby one cell type changes to another cell ty?e in response to stress and generally assists the host to adapt to the stress. The most common type of epithelial rnetaplasia involves rplacement of columnar cells by stratified squamous epithelium.

Dlspfastic cells exhibit considerable pleomorphism (vqriation in size and shape) and often
possess deeply stained (hyperchromatic) nuclei, which are abnonnally large for the size ofthe cell. It is associated with chronic irritation ofa tissue by a chernical agent, such as cigarehe smoke or by chronic inflammatory irritation, such as chronic cervicitis. The tissue appears some\\ hat structureless and disorganized and may consist ofatypical cells without invasion.

Epithelium exhibits acanthosis 6rrr.ll r-t an abnorual thickening ofprickle cell layer). Important: When dysplastic changes are marked and involve the entire thickness ofthe epirhelium. the lesion is considered a pre-invasive neoplasm and is refened to as carcinoma in sttu. *** \{ild to moderate changes that do not involve the entire thickness ofepithelium may be relersible, and with removal of the putative inciting causes, the epithelium may revert to

normal
Histologic features of malignancy:

. Anaplasia . Hyperchromatism ' Pleomorphism '

Abnormal mitosis

The host response to a malignancy is best reflected by lymphocytic infiltration at the edge of a tumor The most characteristic feature ofa malignancy as opposed to an inflammatory Iesion is that a malignancy will grow after removal ofthe causative agent. The most important characteristic ofmalignant neoplasms, which distinguishes them from benign neoplaslns,
is their

abilitv to metastasiz.

. Recurrent apthous minor . Recurrent aphtous major

. Recurrent herpetiform aphthous . Recurrent herpetic stomatitis

'152 CoplriShr O 20ll-2012 ' Dental Declc

. Stomatitis
. Lesions ofthe eye . Genital lesions

. Maculopapular

rash

153
Copyrighr O

20lt 2012, DentalDecks

*** Recunent aphthous ulcers are commonly referred to as "canker sores" by 1ay persons- In the literafure othcr terms include aphthous stomatitis, rccurrent aphthous stomatitis, rccurrent ulcerative stomatitis or ulcerative stomatitis.

Three classifications:
1. Recurrent aphthous

minor:

807o occur on non-keratinized movable mucosa. See

picture #92 in

booklet
2. Rccurrcnt aphthous major: heal often with scarring, more common in HIV patients. occur on the soft palate, tonsillar fauces, labial and buccal mucosa, and tongue. See picture #93 in booklet 3. Recurrent herpetiform: occur in up to 100 at a time, occur on any mucosal surface, heal without scarring. See picture #94 in booklet *** All three classifications present painful recurrent ulcers.

Importanti Vcsiclcs do not precede the formation ofthese painful recurrent ulcers. They appear on wet tnot |ennilion) nonkeratinizcd oml mucosa (i.e., not hanl palate).
Remmber: Herpetic lesions are preceded by vesicles and are more likely to be found on keratinized tissue. See picture #95 in booklet

\ote:
hcet
s

Systemic diseases in which aphthous ulcers are seen include: Crohn's disease /rarnor aphthae),Besyndrome ftnln or aphthae), Celiac sprue (minor aphthae). and AIDS (maior aphlhae).

These ulce6 appear to be associated with stress. The stress factors may include:

. . . . . .

Bacterial infection Trn[fia li.e., self-i ticted, oral surgen, procedures, routine dental procedures) Endocrine conditions (i.e., a lemales menstrual period) AIIergic factors fi.e., ceittain foods or drugs) Immunologic abnomalities lr,Jn- Viramin B or folic acid deficiencres *** The calse is unknown; however, evidence supports they are related to the focal immune dysfunction where T-lymphocytcs play a major role

Stevens-Johnson syndrome fS-/S/ is an immune-complex-mediated h)?ersensitivity complex that is a severe expression of er''thema multiforme. It is now known also as erythema multiforme major ln SJS, the systemic symptoms are sever and the lesions are xtensive, inr olling multiple body areas (especiqlb, the mtrcous membra es).

Tl?ical
. . . .

s\ mptoms are as follows: Cough productive of a thick purulent sputum Headache

\lalaise
Arthralgia

The tvpical " bull's-eye-shaped" target lesions are present. These lesions are considered parhognomonic.The classical triad ofthis SJS consists ofeye lesions, genital lesions and stomatitis. lmportant: The lesions ol SJS are severe and often vesicular or bullous, r ith hemonhage after denudation. Note: Blindness can occur due to secondary infection. Treatment of SJS is primarily supportive and symptomatic: . Vanage oral lesions with mouthwashes . Topical anesthetics are useful in reducing pain and allowing the patient to take in flurds

. Areas ofdenuded skin must be covered with compresses ofsaline . Underlying diseases and secondary infections must be identified and treated . Offending drugs must be stopped Drugs and malignancis are most often implicated as the etiology in adults and the elderly. Pediatric cases are related more often to infections than to malignancy or a reaction
to a drus.

ORAL PATHOLOGY

Ulc Cond

The lesion below is a. smnll (2mm-Smm in diameter), whitish sore with a red border. The patient strtes that it usually begins as a reddish area with a trurning or tingling sensation. The most likely diagnosis is:

154
Copyrighr

lil20ll

1012

'

Denlal Deck!

ORAL PATHOLOGY

Ulc Cond

nrythema multiforme (EM) is an acute self-limited eruption characterized by a distinctive clinical eruption, the hallmark of which is the:

. Chronic desquamative gingivitis . Petechial hemorrhage . Iris or target lesion . Mucocutaneous


rash

155 Coplrrght O

20ll

l0l:

Denral Decks

There are three forms or classifrcations ofrecunent aDhthous stomatitis fuhich is also called recurent aphthous ulcers, canker sores, etc.)

l.Recurrent aphthous minor: is the most common form of the disease and the one referred to by the lay public as the "canker sore." The lesions occur somewhat more frequently in women than in rnen. Minor aphthae have a propensity for movable mucosa that is situated over minor salivary gland tissue. The lesions begin as a single or multiple superficial erosion covered by a gray membrane. The lesion is typically very painful. The lesions vary in size from 2-3 mm to over l0 mm in diameter They generally persist for 7- l0 days and heal gradually with little or no evidence of scarring.

2. Recurrent aphthous major: is characterized by the occurrence of large (5-20 nm) painful ulcers, usually one to ten in number. These ulcers occur
at frequent intervals and rrany patients with this disease are seldom free from the presence ofat least one ulcer Unlike the minor aphthous ulcer, these lesions persist for up to six weeks and leave a scar upon healing.

3. Recurrent herpetiform: the prominent feature of the disease is the numerous, pinhead-sized, gray-white erosions that enlarge, coalesce, and become irregular ulcers. Ulcers occur in clusters of 10 to 100. Remember: Vesicular lesions do not precede the formation of ulcers in all of the abor e. This is a distinctive diagnostic feature.

\ote:

In healing ofan ulcer, the epithelium that eventually will cover the defect is derived liom intact epithelium al the ulcer margin.

Erlthema multiforme frMl is a hypersensitivity reaction that occurs in mild and scvere forrns lt produccs tissue reactions ccntered on the superficial vcsscls of lhe skin and mucous menbranes. Pre.ipilating iactors includc infcctions such as herpes simplex, mycoplasma pneumonia and histoplasmosis. dmgs, radiation therapy. etc. lt occurs primarily in children and young adults. The dia!:no5is oferythema multilorme is primarily based on the classic skin lesion appearance. Clinical features; 3 forms: . E\1 minorl prirnarily involves thc skin with only
2570 in oral mucosa. A low-gradc fcvcr, gcneral malaise and headache usually precede thc appearance ofthe lesions by 4 to 7 days. Assoclared * ith secondary herpes simplex hypersensitivity. Oral lesions appear as .cd macules, papules or \ esicles that may become erodcd and painful. Note: These lesions are covercd by a yellowish-\t hitc membnne aftcr rupturing. Focal or diffuse arcas oferytherna follow and the skin "targct" or "bulls eye" lcsions appear See picture #69 in booklet

. Chronic ENI Minor: mildcst foml

and lesions are small in sizc and shortcr in duration. Oral lesions rary from focal erosions similar to aphthous ulcers to more dilluse areas of erythema or erosions that are painful.

. Elu Nfajor lterers-Johnson Syndrome): acute form that involvcs the skin and mucous Inelnbrane. Large bullae form on the mucous membranes and skin. A positive Nikolsky's sign is common. The bullae collapsc which produces a whitish pscudomembrane on the tnucosa and dark red crusted lesions on the dry skin surfaces (lips and eyes very common).

Note: Behcet's syndrome is a rare disorder that causcs chronic inflammation in blood vessels
rhroughout the body. Manifestations include oral and genital aphthous-type ulcers. conjunctivitis, uveitis, arthritis, headache, and other CNS symptoms. Treatment with corticosteroids and other immunosupprcssive drugs prcvents serious complications, such as blindness.

. Coccidioidomycosis . Histoplasmosis . Tuberculosis

. Actinomycosis

Scarlet fever

156

Copfight O 2011,2012, Dntal Decks

. Syphilis
. Gonorrhea

. Chlamydia
. Tuberculosis

't57
Cop)ri8hr O
201 I

'2012 - Dertal Dcks

Actinomycosis is a subacute{o-chronic bacterialinfection caused byActinom)ces israellii, an anaerobic, graln-posilive filamentous bacterium. The infection is not a contagious discasc and can not be spread fron person lo pcrson. Inlcctions appear after trauma! surgery or previous infection.

Clinical fcatures: . \4ost infections occur in thc lhorax. abdomen and head and neck called cervicofacial actinomycosis /i, fection n'picall.v occurs i puti?ntsIi!hpoorlental hryiene or lilloti gorc| surgen) . oftcn prescnts as a swelling ofthe nandible and usually is painful . Once in the tissue, Actinomyces forms an abscess, prcducing a hard. red-to-reddish-pu4)le Iump, olien on the

jaw

-\\hich *+* Treatnrent for aclinomycosis

"lump) jalr" *** See picture #70 in booklet Exudatc from thc draining sinus tracts often contain small, clinically visiblc ycllow-green calcified structures (sulfur grunules) arc adually colonics ofinficting organisms

is

long-term penicillin.

Histoplasmosis is a chronic lung infection caused by inhalalion ofspores of Histoplasma capsulatum. It is endcmic ro rhcAmerican Mid\r'esr. The lassic oral manifestation ol histoplasmosis infection is a chronic nonhealing ulcer. Note: The lesions are usually covercd bv a non-specifrc gray membranc and are induratcd.

Clinical features:

. Fe\er.

malaisc, cough, and dyspnea

. Ccrvicll lymphsdenopathy is common . In chronic fonns, dissemination to the skin and oral mucous membrancs may be the first sign ofinfec-

"**

Treatmenr for histoplasmosis is an)photericin B, itraconazolc or ketoconazole for 6-12 months.

Coccidioidomlcosis (also ktlo\n as Mller lever) results fron inhaling the spores olCoccidioides species 'C,iLtJi)id!\ i,tmilis or Coccidioicles posada';ir. Most infections in the United States arc acquired within the l|: ,,: reuions ofcndcmiciq ofsouthern Arizona, centfitl or othcr arcas ofClalifornia. southcm Nc$'Mcxico, ::rc \.i: Tc\as. Symptoms are similar to histoplasmosis and it is usually treated $ iih amphotericin B.

\otc:

Scarlet

feier

is a systemic infection causcd by Strcptococcus pyogenes. It

i!

characterized by pharyn-

irirs. l!!cr. malaise, strawberry tongue (hos d :||hile coating with red, inflaned fungiforn pupillae) and ^ ikr:r :ash Important: The p!'rogenic (et_rlhrogeni() e\?toni,'? causes the rash ofscarlet fevcr and systemic
:.,\ ta ihock slndrome.

Srphilis is an inf'cctious. contagious venereal discase that is caused by the spirochetc treponema

pallidum. It is one of the

less common scxually transrnitted infections /S11/. The incubation pe-

nod \aries some$.hat bctween 10 and 90 days. Thc sylnptonls ofsyphilis may pass unrecognizcd, !rr Dra\ be nlisintcrpleted and at times there arc no initial symptoms at all. Whcn prcscnt. thc clasirJai s\mptoms ofsyphilis lnanifest themsclves in threc stages as lbllows:

L Primarr': the first symptom is a non-painlil chancre that generally appears 2-6 weeks alier e\l-r.)sure. lt usually is fbund on thc part ofthe body exposed to thc partner's ulcer. such as thc tL'ni\. thc \ul\a, or the vagina. lt can also develop on thc cervix, tonguc. lips. or othcr parts of lhe brrd) The chancrc disappears within a l'erv wccks whethcr or not a penon is trcated. Ifnot lrcatcd during thc primary stagc. about one third ofpeoplc will progress to chlonic stages.
Secondarl: is a highly infectious stage; it occurs 6 weeks aftcr non-treatment of primary :r philis. \\'idely dissetrinated spirochetes cause mucous lnerlbranes 10 cxhibit a reddish brown nraculopapular cutaneous rash and ulcers that are covered with a nucoid cxudate ku/led mu.ot,r pdtches). Condylomata lat^ (\'hich are elevated bntad-based pluquesl are also seen on ikln and mucosal sr-rrl'aces. lf left untrcatcd, thesc symptoms will rcsolve on their oq,n but thc iniictious microbc remains behind. It is at this point that syphilis passes into its latent phase. This \ilent period ntay last fbr many ycars and permits the infcction to evolve wi{hout any ob\ rous cxtemal symptoms. At this point. the only nrcthod ofdctecting the prcscnce ofsyphilis is \ i r., blood lc.l spccilic lor syphrlis.
.l. Tertiary: occurs in inf'ected persons many years alier non-trcatment ofsecondary syphitis. Thc gumma (*,lrich is a .fotal nodular nlasr) typifies this stage. lt most cornmonly occurs on lhe palate and tonguc. The bacteria damage the hcart, eyes, brain, nervous svstem, bones.joints, or almosl any othcr part ofthe body. Note: Headache, stit'fneck, and f'ever are symptorrs of neur-

i.

os)philis.
Remember: Conorrhca is a sexually transmitted disease caused by Neisseria gononhoeae. An oral manifestation of gonorrhea is oral pharyngitis.

158 CoplriShr
@

201|,2012 - Dntal Deck

. Coccidioiodomycosis
. Mucormycosis

. Aspirgillosis
. Zygomycosis

'|

59

Cop)'righr O 201 I -2012 - Dental Dcks

Congenital syphilis is caused by infcction with thc spirochctc Treponema pallidum during the
fetal period. Expectant mothers who have syphilis can transmit the disease through the placenta to the unbom infant. Nearly halfofall infants infected with syphilis during gestation die shortly beforc or after birth. The severity ofcongenital syphilis depends upon l) The time in which the organisn]s pass the placental barier (protected up to I6th week), 2) The mother's stage of syphilis and, 3) The immunologic rcsponsc ofthe fctus. Iftrcated by the 4th or 5th month, 957n show no manilestationsi ifnot feated, fetal sepsis may result in stillbirth or visceral and mucocutaneous manifestations.

l. Symptoms in the newborn .lrritability


. Failure to thrive . Bloody discharge from the nose . Saddle nose, lrontal bossing and short maxilla

. Later rash: coppcr-colorcd, vcsicles on the palms and soles . Early rash: small blisters or a flat bumpy rash on the lbce. palms
2. Symptoms in older infant and young child

and sole

. Bone pain . Joint swelling . Abnormal teeth fHrbhinson's incisors)

Saber shins r'borre abnormalitT-

. Gray, mucous-like patches on

the anus

qfthe lower leg) andy\tlya (cond),lona lata)

. Visual loss. CN VIII nerve deafness and intentitital keratitis . Scaning of the skin around earlier lesions of the mouth, genitalia, and ants (called rhagctdesl

combination of Hutchinson's teeth, interslitial keratosis, and deafness due to lesions ofthe eighth cranial nerve. Note: "Screwdriver" incisors and "Mulberry molars," dcntal defects seen in congenital syphilis are caused by direct invasion of tooth germs by Treponema organisms.

Remember: Hutchinson's triad

-the

The gencra most commonly rcsponsible for mucormycosis usually are Mucor or Rhizopus. Orbitorhrno-cerebral mucormycosis, the most common typc, genemlly occurs in conjunction rvith sinus rrr nasal involvement. Mucormycosis also may aflect other parts ofthc body, including the lungs, GI tract. or skin.

Diabetic patients arc predisposed to mucormycosis because of thc decreased ability of thcir neurrophrls 10 phagocytize and adhcre to endothelial walls. Furthermore, the acidosis and hyperglycemia provide an excellent environrnent for the fungus to grow.
Othcr patrent. at risk include thc follou ing: . Patients on chronic antibiotics, steroids, or c),totoxic therapy . Palienls with chronic renal failure or liver problems

. Parients with transplants

Palients with cancet HtV, malnutrition or acidosis


as destructive ulcerations

Important: In the head and neck, most lesions appear


sinuses or nasal cavity.

in the paranasal

The use of systemic amphotericin B is important in treating mucomrycosis; its use, along with incrcased awareness ofthe disease, has decrcascd thc mortality

Rhizopus is the principal cause ofzygomycosis, which occurs primarily in patients suffcring from diabetic ketoacidosis (thinocerebral diseasel, malnutrition, scvcrc bums, or who are itnmunocompromrsed.

Aspergillosis is a group ofillnesses caused by mold. In some people, the spores trigger an allergic reaction. Other people develop mild to serious lung infections. The most serious form of as-

pergillosis

invasive aspergillosis occu6 when the infection sprcads to blood vessels and beyond, into the lungs to othcr organs. Note: Aspergillosis is the second most frequently seen f'ungal infection ofthe f'ace and mouth in patients receiving chemotherapy. It is second only to candida in its fiequcncy. Oral lesions appear as necrotic ulcerations with gray pscudornembranes ofthe ginsiva and hard palate.

A 4-year-old

ptint comes with hr mother for a routine appointment. The mother states that her d.ughter just started not feling well and had a mild fever earlier in the day. Th daughter has ben having troubl swallowing. An intraoral exam reveals multiple 1-2 mm vesiculopapular lesions ofthe nasopharynx and soft palate. Your working diagnosis is:

. Herpangina . Hand-foot-and-mouth
disease

. Herpes simplex infection . Pemphigus lrlgaris

'!

60

Coplrigbr

iO

20ll-2012 , Denul Decks

ORAL PATHOLOGY

Which type ofherpes virus is associated with the lesion on the lower lip?

. HSV- l
. HSV-2 . HSV-3

. HSV-4

161

CopFighr C

201

l-201 2 - Denral Decks

Herpangina refers to a stomatitis (inflammotion ofthe mouth) caused by a strain olthe Coxsackie- virus. It is differentiated in clinical practice from Type I Herpes infection (the cold sore virus) by the fact that the ulcerations ofherpangina are localized to the posterior soft palate and nasopharynx. Remember: Herpes Type I lesions are found typically more forward in the mouth on the tongue, gingiva, buccal mucosa and appear as vesicles (smal/, cleqr blisters that ulcerate and crust) around the mouth and on the lips.

Clinical features: . Most commonly affects infants and young children . Typically occurs during the summer . Typically spreads via the fecal-oral route or via the respiratory droplets . Symptoms are mild and short in duration (no more than I v'eek) . Sore throat and difficulty swallowing . Mild fever . Small vesicular or punctuate lesions with white base on posterior soft palate near uvula and anterior fauces ofthe tonsils *** See picture #71 in booklet Important: The disease usually runs its course in less than a week. The treatment is palliative.

coxsackievirus

\ote: Hand-foot-and-mouth disease, is a highly contagious systemic infection caused A of limited duration in which vesicular eruptions occur on the palms of
ofthe anterior part ofthe mouth. It is uncommon in the
are

hands. soles offeet and mucosa

oropharyngeal

a (unlike herpangina).

*** Remember:

The majority

oforal herpes cases

are caused by

HSV-l

and the majority

of

genital herpes cases are caused by HSV-2.


Oral and para-oral presentations of Herpes Simplex Type

include;

. Herpes fabialis (also called fever blisters or cold sores) is an extremely common disease caused by the herpes simplex virus Type I . charactedzed by an eruption of small and usuaLll painful blisters on the skin ofthe lips, mouth, gingiva, or the skin around the mouth. \ote: The reason most patients suffering from recunent herpes labialis rarely give a history of having had actne herpetic gingivostomatitis is that the pdmary infection rvas subclinical.

. -{cute herpetic gingiyostomatitis (also known as primary herpeti. gingivostomqtltis)


generall-v'. affects children under the age ofthree and young adults. There are prodromal slmptoms (,fever, malaise, irritabiliry, headache, clysphagia, vomiting, b,mphadenopdth)')

-2 days prior to local lesions. Then small, yellowish vesicles form, which rupture quickly. resulting in shalloq round, discrete ulce$ with an eDthematous halo. It affects both the free and attached mucosa. A generalized marginal gingivitis may prccede the ulcers. Treatment includes fluid intake, good oral hygiene and gentle debridement ofthe mouth. ln healthy individuals the lesions heal spontaneously in 7- l4 days without a scar.
l

. Recurrent /secord4ry, herpetic stomatitis generally occurs in adult patients and is triggered by trauma, fatigue, immunosuppression, stress, allergy or sunlight, which causes the release (or rcdctivalioz) ofthe latent HSV-l virus in the trigeminal ganglion. This reactivation causes a recurrent infection f.e., told soles) on the lips (that is bound to periosteuml, hard palate, attached gingiva and alveolar ridge. Site-specificity is a characteristic manifestation. Note: Lesion on the finger is called herps whitlow.

. Cytomegalovirus

. Epstein-Barr virus . Herpes simplex virus I


. Herpes simplex virus 2

162
Cop)righr O 201l-2012 - Dental Decks

. Lipschutz bodies . Civatte bodies


. Lisch nodules . Reed-Stemberg cells

163 Coplright O20ll-2012, Denhl Decks

Primary herpes is most common in children and young adults. Patients develop fever. irritability, regional lymphadenopathy and headache. Within days. the gingiva becomes intensely inflamed. Any pan of the oral mucosa and lips may become involved. Vesicles
then form and rupture a short time later to leave shallow ulcers covered with a gray membrane and surrounded by a red halo. These ulcers are very painful. These ulcers will heal on their own within 7 to 14 days.

After recovery from primary HSV infection, the virus is not cleared from the body, but, rather. it lies dormant in a non-replicating state, in the sensory nervous system (specilicall,r, the trigeminal ganglion/. Periodically, Iatency reactivates. allowing the virus to retum to the skin or mucous membranes, where it causes a recurrent inlection. Cold sores are a manifestation ofrecurrent herpes simplex virus infection around the mouth. The lr.rost common site is on the lips. Some factors that are often associated with a recurrent
outbreak are: sunbum, fatigue, emotional upset, trauma, upper respiratory tract infection or menstruation. Often a day belore the formation of vesicles there will be a tingling or itching ofthe skin or mucosa. Vesicles ulcerate and resolve the same as in the primary ini-ection.

Histofogically, the cytopathic effect (CPE) take the form ofballooning degeneration of rhe epithelial cells with loss of cohesion to adjacent cells. The nuclei are olten multiple
margination ofthe chromatin around the intra-nuclear inclusions called Lipschultz bodies. These changes can be seen in scrapings taken from an unroofed vesicle (tlrcse .rcrapings are callerl a Tzanck smear).

*ith

The Tzanck smear is done by smearing cells taken from a fresh blister or ulcer onto a mrcroscope slide. The cells are stained with a special stain, such as Wright's stain, and then examined under a microscope for characteristic changes caused by a herpesvims. Herpes causes giant cells with multiple nuclei. The shape ofeach nucleus appears molded ro t'it together u ith those adjacent. The background ofthe cell looks like ground glass and contains snall dark spots called inclusion bodies (Lipshutz bodies).
Orher tests that can be used to diagnose herpetic lesions include:

Fluorescent staining: cells show positive fluorescence when stained with fluor-

escent labeled HSV immune serum and globulin. This procedure is used to distinguish benr een herpes zoster and heryes simplex.

.Isolation in tissue culture . Antibody titers (anti HSV ontibod!- titers)i is a test for complement fixing or neutralizing antibody in acute and convalescent sera as well as on tissue sections (this begins irt one v:eek arul peaks in three veeks). Biopsied material will show an intraepithelial cleft covered by an exudates offibrin and polyrnorphonuclear leukocytes. The epithe|um willexhibit degenerative cells, which include bizarre giant cells and cells with displaced chromatin with perinuclear halos and inclusions.
The treatment for herpes is

primarily supportive:

. Analgesics . Topical anesthelics prior to eating . Maintain electrolyte balance . Antiviral agents

. Herpes zoster
. Herpangina . Recurrent herpes . Chicken pox

tu
Copynghl O 201l-2012 Dntal Decks

. Manifested

as

ANUG

. Associated with HIV

. Subclinical
. Characterized by severe lymphadenopathy
and acute dermatitis

165

CoprrightO 20l l-2012 - De al Deks

The varicella zoster virtrs (VZV) is a member ofthe herpes virus group. It causes the disease chickenpox (varicella) and shingles (herpes zoster). The virus is very contagious and may be spread by direct contact or droplets.

Chickenpox is primarily a disease of childhood, which peaks at school-age in the winter and spring. lt is characterized by the appearance on skin and mucous membranes ofsuccessive crops of typical pruritic vesicular lesions that are easily broken and become scabbed. lt is generally accompanied by mild constitutional symptorns (/everr malaise).lt is most contagious one day before the onset of the rash and until all the vesicles have crusted. It is relatively benign in children, but adult infection may be complicated by pneumonia and encephalitis. Note: ZIG (Zoster Immune Globulin) reaches morbidity in highrisk children.
Shingles (herpes zosle, is the result ofreactivation ofa latent varicella-zoster virus that is believed to reside in the sacral ganglia from a childhood case ofchickenpox. The virus reaches the sensory ganglia ofthe spinal and cranial newes, producing an inflammatory response. It is characterized by painful vesicles that occur on the skin or mucosal surfaces along the distribution ofa sensory nerve in a distinctive unilateral pattern.

\ote:

The histology for both chickenpox and shingles shows the same cytopathic effect

as seen in herpes simDlex.

Herpes simplex is one ofthe most common viral diseases affecting man. The primary infection. rvhich is known as primary herpetic gingivostomatitis, is most common in r oung children /rin der live yeors old). It usually occurs in a child who has had no contact u ith the Type I herpes simplex virus, and who therefore has no neutralizing antibodies. h mal also affect young adults //J-25). Nearly all primary rnfections (90o/o) are ofthe subclinicaf type (they moy only have flulike symptoms) and one or two mild sores in the mouth lvhich go unnoticed by the parents. In other children, the primary infection may be manifested by acute s)'rnptoms, which is knorvn as acute herpetic gingivostomatitis. These symptoms include feveq initability, cen ical lymphadenopathy, fiery red gingival tissues and small yellowish vesicles that mprure and result in painful ulcers on the free and attached mucosa. The most serious potential problem in a child with this infection is dehydration due to the child not wanting io eat or ddnk because ofthe pain. See picture #72 in booklet The treatment is supportive and aimed toward the reliel of the acute symptoms so that fluid and nutritional intake can be maintained. Primary herpetic gingivostomatitis usually runs a course of l2-20 days, and the ulcers heal without scaning.

\ote: Corticosteriods are contraindicated in patients with

herpes simplex infections.

v-B Dis

After the initial primary attack during the early childhood period, the herpes simplex virus remains inactive most comrnonly in the:

. Geniculate ganglion

. Ciliary ganglion
. Trigeminal ganglion

. Pterygopalatine ganglion

166 Coplright O

20ll 1012

Denral Decks

ORAL PATHOLOGY

V-B Dis

A 49-year-old patient ofAshkenazi heritage presents to your oflice complalnlng of "blisters in her mouth.' Your intraoral exam shows ulcers present on multiple areas of mucosa. She also related to you that while getting out ofthe car earlier, her skin ofher arm rubbed against the car door and tore. Known as Nikosky's sign, this phenomenon is associated with which disease?

. Herpes zoster

. Lupus erythematosus . Lichen Planus . Pemphigus

t67
Copyright,e 201

l-:012

Dental Decks

The inactive herpes simplex virus resides in sensory nerve ganglia (mo.st commotll!', the trigeminal gangliol, but will often reappear later as the familiar "cold sore," usually orr the outside of the lips. This disease is referred to as "recurrent herpes labialis." Emotional stress, trauma and excessive exposure to sunlight have been implicated as factors for the appearance of the recurrent herpetic lesions on the lip. Acyclovir 57o ointment (Zovirax) as well as valacyclovir, and famciclovir have been successful in reducing the du-

ration and severity ofthese sores.

Remcmber:
Herpes Simpfex Type I (prinary; herpelic gingirostomLttiti.t, recuftent herpes labiis transmitted by direct contact. It affects the lips. face, skin and oral mucosa. 2. Herpes Simplex Type ll (herpe.s geuitalis/ is spread by sexual contact. It affects
I.

dlrt

the mucosa

ofthe genital and anal regions.

\ote: Cenital

herpes may have serious consequences in pregnant women because the r irus can be tnnsmitted to the infant during vaginal delivery. The virus can cause damage to the infant's central nerr,'ous system and/or eyes.

Remember: The primary infection of herpes simplex can range from subclinical
IQs\tnptonqtic, v'hich is most common) to severe systemic infections.

Penrphigus (an autoimmune disorrler) is a term used to describe blistering ofthe skin caused b1 binding ofantibodies to the surface ofthe cells ofthe outer layer ofthe skin, the epidermis. In pemphigus yulgaris, the most common form ofpemphigus. antibodies are directed against rhe desmosomal adhesion molecule Dsg3, resulting in severe mucosal erosions and eprdermal blistering in areas such as the mouth. As a result, patients develop severe oral ulcerations. and may also have inflammation or erosions of the lining of the eye and eyelids I conitol(tiya), the nasal mucosa, or the genital mucosa. Half of the patients also develop b)isrers or erosions ofthe skin, often in the head and neck area. It seldom occurs before the age oi j0 rususlly benteeu 30 and 50) and occurs more frequently in Jewish people. There are four types ofPemphigus: L Pemphigus vulga s: most common L Pemphigus vegetans

3. Pemphigus foliaceus 4. Pemphigus erythematosus

first manifestation ofthe disease. Intact bullae are rarely seen in the oral cavit,v. instead, large areas ofulceration and erosions are often seen that are covered by a Oral lesions
are often the

u hite or blood+inged exudates. Sometimes, areas of epithelium will slide ofl simply by rubbing of an apparently unaffected area (this is 1e7-lt?ed Nikolsky's sign/. This disease is often

fatal without therapy, which includes high-dose systemic steroids or chemotherapy drp1e, methotrxate/.

/br

e-r-

Important histological features: The vesicles and bullae are fbrmed entirely intraepithelially, just above the basal layer of cells (calletl suprabasilar veslcleJi. There is intercellular edema and loss ofintercellular bridges with loss ofcohesiveness. This is called acantholysis. Clumps of cells are often found floating free in the vesicle space (tieve cells are called Tzanck cells).

. Candidiasis
. Hairy leukoplakia

. Desquamative gingivitis
. Hemorrhagic mass

168 Copt{ight O 20ll-2012 - Denhl Decks

. Human papillomavrrus (HPV)


. Adenovirus

. Epstein-Barr virus

. Parvovirus

169

Cop'-i8hI O

201 I

'2012 - Dental Decks

Benign mucous membrane pemphigoid (BMMP) is an autoimmune or "self-allergy" disease in which a patient's own circulating antibodies become altered so that they attack the fibrous attachment of the skin and membrane epithelium to the underlying connective tissues. Women are more commonly affected than men and the disease is usually diagnosed between the ages of 40-60 years. The typical lesion is a small or large, clear-fluid blister which breaks fairly rapidly in the mouth to leave a flat white, somewhat tender ulcer with a thin red line around it. The gingivae are especially likely to be involved, resulting in sloughing during eating or tooth brushing ("desquamative gingivitis"). Systemic steroid therapy has provided adequate management of BMMP Note: Conjunctival involvement
may lead to blindness.

Remembr: Nikolsky's sign, which is an indication ofpemphigus vulgaris, nay also be lound in BMMP The sign occurs when apparently normal epithelium may be separated at the basal layer and rubbed offwhen pressed with a sliding motion.

While similar in its clinical presentation to pemphigus vulgaris, it is much less severe and the involved antibodies attack the attachment fibrils (Type VII collagen) ofthe basement membrane antigens fi.e., Laminin 5 and BP 180), rather than the desmosomal attachments (desmoglein 3 [Dsg3]

) between epithelial cells,

as occurs in pemphigus.

Important: Histologically, the major difference between BMMP and pemphigus r,ulgaris is that the vesicles in BMMP are subepidermal and there is no evidence of acantholysis in pemphigus vulgaris there is acantholysis and a suprabasilar vesicle,

This is the same virus that causes genital wans which lcads to dysplasia and cervical cancer. Some esti mates ofthe prevalence of IIPV infection in the population range as high as 79%. Warts may arise on anv skin surface, bnt occur most commonly at acrd (peripheral) sites. Although treatment may remove rhe \\'art. the virus remains latent within the skin cells. However, treatrnent may diminish spread ofHPV in the skin ofthe infected patient and possibly to uninfccted contacts.

. Papillomas: . Benign epithelial prolifefttions (pedu culated or sessile) of \ftle significance. . \trruca vulgaris (also called squamous papillona) has an incubation period from about six \\'eeks to a ycar. Although it is primarily a lcsion ofthe skin, it may occur in the oral cavity, particularly on the lips and palate. Clinically, it is a sessile, soft, cauliflower-like lesion. Ifexcised, they
usually do not recur, but autoinoculation is possible. Note: Intraorally, that is how most cases dcvclop. Much more prevalent in HIV positive patients . Condyfoma acuminatum (ge ilalwa s): Caused by HPV subtypes 6 and l1; oral lesions acquired b) oral-genital contact; broad-based verruciform lesion.

. Focal epithelial hyperpl^si^ (Heck\ dise4se,): Most common in ethnic groups fi.e., Nat[re Artelicans ond Centtul Anlencdrt. Multiple small, dome-shaped warts on oral mucosa. Caused by HPV
subttpes 13 and 32.

Important: Sexually transmitted, high-risk HPVS include types 16, 18, 31, and 33. Tlese are the HPV t] pes associated with cancer.

Verrucr vulgaris on the tongue

V-PLes

A 45-year-old female walks into your olfice complaining ofa "wart" on her gums that has been thre for yars. Your exan reveals an asymptomatic, well-circumscribed, slightly raised, papillomatous lesion on the buccal gingiva of tooth #5. A likely diagnosis of this is:

. Fibrosarcoma . Neurosatcoma

. Lipoma . Verucifonn xanthoma

170
CopyriSlrr O

20ll

?01: - Denral Decks

ORAL PATHOLOGY

V-PLes

A 63-year-old completely edentulous patient comes into your office because her dntures hrve "finally gottn too bothersome to wear.' Eer health history consists of COPD and cigarette 40-year pack history. Your intraoral exam reveals a noxious odor and an ill-litting upper denture. When you remove the upper denture you note multiple red, papillary projections ofthe hard palate. Your patint states she does not remoye her dentures at night or tretween meals. After reviewing denture hygiene instructions, you give her the diagnosis of:

. Epulis fissuratum
. Papillary hyperplasia

. Nicotinic stomatitis . Kaposi's sarcoma


171

CopriSht O 201l'?0ll ' Denlal Decks

i.s 45 t t'ars). Thc usual intraoral locations are the gingiva and alveolar mucosa, but any oral mucosal site may bc involvcd. Thc lesion appears as a rvcll-circurnscribed, slightly elevated mass with a papillomatous or vcrrucous surfacc.Thc level of keratinization of the surface will influence its color. which ranses fiom \1hit(r or red.

This unique Iesion occurs in middle-aged individuals /alera.ge a.ge

Well-circunNcribed, lobulated, soil yellowish mass; vessels visible over suiface; buccal mucosa, floor of
mouth. long! common srtes

Rarely encountercd in soft tissues of the head and neck. Yellowish submucosal mass. Slow-growing malignancy.
Adnlts (average age is ,l5r; gingiva alveolar mucosa are most common sites; well-circumscribed, slightly ftised, papillomatous or verrucous surface

Surgery or radiationi prognosis is lair to good.

&

When in bone,lesion ma), arise fiom periostcum. endosteum or PDL. When in sofl
trssue. fibroblasts.

Rare sofl tissue and bony malignancy

Wide surgical excision;

ofthe head and neck. Young adults are reculTence not common
most commonly affected. Infi ltrative neoplasm that is locally deslructive.
Rare malignancy; Pain or paresthesia may accompany lesion in bone. ln sofl tissues, it appgars as an expansile mass that is usually asymptomatic

Eithcr from a pre-existing lesion of neurofibronlatosis or dl: ,,o'r,. Thought to be from Schwann cell.

Widc surgical excision; Recurrence is conmon, metastases are fiequently sccn. Prognosis is fair io
good

f,nrity
De.nire-induced fi brous hlpcrplasia , I t'll a|""at ory b,peryla si a, d?nhrc h\perplasia, attd

Etiolo$
lll-fitting prosihesis

Location
Common lesion that occuis in the

Clinical Characteristics
I'ainless folds

lrd

Treatment
Prognosis

offi'

Surgical excision prosthesis with possible remaking

overextended denture flange conects Hard palate

'Epulis lissvtatum")
Poor oral hyglene & _Palatrl papillomatosis!') ill fitlinf prosthesis r Papillar,v hyperplasia

flange relining dnturs


Surgical cxcision

papillary projections.

\otes

l. There is no malignant potential to any ofthe librous hyperplasias. fibroma is a reactive hyperplastic mass that occun in the gingiva and may be derived from connective tissue ofthe submucosa or the PDL. It
2. The peripheral

prcsents as well-demarcated focal mass with either a sessile or pedunculated base. It is similar in color to the surrounding connective !issue. [t may be ulcemted. The treatment for a peripheral fibroma is local excision. Recurrence is rare. Note: Other variant forms ofthe peripheral fibroma include the peripheral ossiiying fibroma and the peripheral odontogenic fibroma. 3. Focal fibrous hvperplasia is hyperplasia oforal mucosa. Tt is also called tmumatic fibroma, irritation fibroma, and hyperplastic scar. lt is a reactive lesion caused usually by chronic tmrrma to oral mucous membrane. See picture #23 in booklet. The giant cell fibroma is a focal fibrous hyperplasia in which connective tissue cells, many ofwhich are multinucleated, assume a stellate shape.

The picture below shows a benign epithelial neoplasrn which appears as a pdunculated, whitish cauliflower-like mass on the lower lip. The most likely diagnosis is:

172

coprighl

a(,

201 1-2011

- Dental Decks

ORAL PATHOLOGY

A 54-year-old African American female presents to your clinic for an initial exam. She has a history of hypertension controlled with beta-blockers but no other contributory health lindings, Your intral oral exam reveals a bilateral

lilmy opalescence ofthe buccsl mucosa. When stretching out hr cheeks, this white hue disappears. Your most likely diagnosis is:

. Squamous cell carcinoma . Fordyce granulation . Leukoedema


. Leukoplakia

173
Cop)right C 20ll-2012 - Dental Decks

The paprlloma is a benign exophytic papillary growth of stratified squamous epithelium. Note: The common wart, or verruca vulgaris, is a lrequent tumor of skin analogous to the oral papilloma.

Clinical features: . Adults


. Anywhere on the oral mucosa . Sessile or pedunculated exophytic growth . Papillary ftauliflowerJike) appearance . Long duration . May show considerable keratin and in some instances appear white clinically. has little keratin on the surface it will appear pink.

If it

Treatmenl and Prognosis: Conservative excision: recurrence is rare.


Rememtrer: A fibroma is a benign neoplasm of connective tissue origin.

The appearance varies from a filmy opalescence of the mucosa in the early stages to a more definite grayish-white cast with a coarsely wrinkled surface in the later stages. The lesions usually occur bilaterally and are most noticeable along the occlusal line in the bicuspid and molar region. Diagnostically, one can stretch the tissue and the white essentiallv disappears. Important: Leukoplakia would not disappear when stretched.

Important point: Leukoedema appears to be simply a variant ofnormal mucosa and no treatment is necessary; merely diagnosis. See picture #73 in booklet
The differential diagnosis should include:

. Leukoplakia . Snuffpouch

(SP)

. Frictional keratosis

. \Vhite sponge nevus . Hereditary benign intraepithelial dyskeratosis


Histologically, in leukoedema, the epithelium is parakeratotic and acanthotic, with marked intracellular edema of spinous cells. Note: The white appearance ofleukoedema is caused bv s ater within the spinous cells causing the light to reflect back as whitish.

Remember: Snuff pouch (SP) is a form of hyperkeratosis with various degrees ofclinical manifestation (1.e., white mucosal change). SP develops on those mucosal sites where the tobacco is held. The causal agents ofSP are considered to be the nitrosamines and hydrocarbons contained in tobacco. Prolonged use ofthis habit may conduce to the devel-

opment of a squamous cell carcinoma due to the carcinogenic potential


components. See picture #77 in booklet

of

those

ORAL PATHOLOGY

WLes

A 67-year-old Caucasian male comes into your oflice for a routine check-up, He relates to you that he just got back from Florida where he goes for the fall and winter months. He enjoys taking his boat out with his wife. Your extraoral exam shows chapped lips but his lower lip also presents with grayish-white plaques. There is marked loss ofelasticity of the vermillion border. Which of the following would you make your diagnosis:

. Actinic keratosis
. Actinic cheilitis . Actinic dermatitis

Solar lentigo

174
Copyright O 20ll-2012 ' Denlal Decks

ORAL PATHOLOGY

WLes

An incisional biopsy is indicated for which of the following lesions?

. A.5 cm papillary fibroma ofthe gingiva . A 2 cm exostosis ofthe hard palate . A 2 cm area ofFordyce's disease ofthe cheek . A 3 cm hemangioma ofthe tongue . A 3 cm area of leukoplakia ofthe soft palate

175
Copyrighl

(l20ll ?012

DentalDecks

Actintc (solar) cheilitis, a variant oforal leukoplakia, is considered to be the labial counterpart of solar (actinic) kerctosts (a precursor of SCC of the skin). The lips appear dry mottled, and opalescent with slightly elevated white or gray plaques and that cannot be stripped off Isolated areas ofhyperkeratotic callus may also be evident as well as loss of elasticity and definition ofthe vermilion border. See picture #74 in booklet The short-term effects of exposure to UV light (especioll.v UVB, 2900 to 3200 wn) are transient, but the cumulative long-term effects produce irreversible damage (actinic cheili/lr, usually to the lower lip ofexposed individuals.

Other clinical signs include:

. Eq.thematous or hemorrhagic areas . Parallel marked folds . An unobtrusive "chapped lip" appearance
. The junction of vermilion and skin becomes indistinct.

Malignant change is manifested clinically by areas of more diffuse cheilitis and ulcerations ofrelatively long duration. Although degenerative changes have been observed predominantly in men after the age 40, the condition now is increasingly recognized in
) ounger men.

Important: This condition is considered premalignant and may lead to squanous cell carcinoma. It should be treated accordinslv.

r(r

Leukoplakia is a premalignant lesion. This means that ifleft untreated, some ofthe lesions progress carcinoma. [t is bccause ofthis chance ofmalignant translbrmation that all leukoplakias should

be biopsied, Note: Leukoplakia is a clinical diagnosis.

. Idiopathic leukoplakia: refers

to white/opaque oral mucosa lesions that do not rub off and

are not clinically diagnostic for any other whitc lesions. The causc is unknown, however, tobacco iitation (especialll pipel and alcohol may be contributing facto.s. [t is more common in older men. lt is a slowly developing change in a mucous membrane characterized by thickened, white, 1irmly attached patches that arc slightly raised and sharply circumscribed. Lesions ofthe floor

of the mouth and base ofthe tongue are most aggressive. Most display no dysplasia but can de\ elop into rnalignancy (5'% to I5o/A.ln all cases, leukoplakia must be completely excised
since diagnosis cannot be made clinically. See picture #75 in booklet

. Proliferative verrucous leukoplakia: it

is a high-risk form of leukoplakia The cause is unknorvn, although somc are associated with human papillomavirus 16 and 18. Lesions arc recuffent or persistent and usually multiple. Lesions may start llat but progress to broad-based' v artllke (vermcforn) lesions. There is a high risk ofmalignant transformrtion to verrucous carcinoma or squamous cell carcinoma.

Explanation oflesions on front ofcard:

. Papillary fibroma:

is a benign neoplasm ofconnective tissue ongin Exostosis of the hard palate: lori (p!:tlatal or nandibulat): most cotr..rnon exophytic lesions,

slou-grou rng benign Lnots ofbone . Fordyce's disease (or granules): ectopic sebaceous glands in the buccal mucosa and/or lip They are present in over 757o ofadults. They usually appear as yellow, sometimes yellow white submucosal clusters that are essentially normal. See picture #76 in booklet Note: When mechanical irritation produces
a

white lesion it is called frictional keratosis.

ORAL PATHOLOGY

You are conducting a routine exam on a 54-year-old patient with diabetes mellitus t'?e 2 and a 20-year pack history of smoking. You see a white patch on left alveolar edentulous ridge. The lesion cannot b wiped off and the patient denies a history of trauma or allergies. Which ofthe following would be your diagnosis?

. Squamous cell carcinoma

. Lichen planus
. Erythroplakia . Leukoplakia

175
Copyright

(i 201l':01:

- Dental Decks

ORAL PATHOLOGY

WLes

A 75-year-old patient comes to your olfice wanting a new set of dentures, She hasn't ben wearing her old dntures for rbout 2 years. She has a collapsed \aDO and her physician is concrnd rbout her iron deficiency. The corners ofher mouth are fissured, dry, and erythernatous. Which ofthe following conditions is the likely diagnosis?

Squamous cell carcinoma

. Angular cheilitis
. Vemrca vulgaris

Stomatitis nicotina

177
Copyrighr

e 20ll-:0ll

- Dcntal Decks

The etiology ofleukoplakia is thought to be a varied one. Possible etiologic tactors include tobacco, aicohol, and oral sepsis. It is most often due to tobacco use. Some investigators believe that pipe smoking is most harmful.

clinical white patch or plaque on the oral mucosa which will not rub off and which cannot be characterized as any specific disease. Most reports indicate that leukoplakia is more common in elderly men. Although less common than leukoplakias. erythroplakias, have a much greater potential for becoming malignant. Leukoplakia is
a

Important: Any white or red lesion that does not resolve itself in two weeks should be reevaluated and considered for biopsy to obtain a definitive diagnosis.
The term carcinoma in situ is applied to mucosal lesions which resemble leukoplakia in all respects xcept that dysplasia is very pronounced and involves almost all epithelial layers. It shows no tendency to invade or metastasize to other tissues. The clinical differential diagnosis ofa white patch should include:

. Leukoplakia . Lupus erlthematosus

. Candidiasis . Lichen planus


. Migratory glossitis/stomatitis . Frictional hyperkeratosis

. \\'hite
.

sponge nevus -A. chen.rical / thermal bum

It is also associated with the loss of vertical dimension.This situation is generally obsen ed in elderly patients.The corners ofthe mouth become painful, iffitated, red, cracked, and scaly. The fungus Candida albicans (thrush) may grow in the corners ofthe mouth, keeping them sore. Note: It can also result from a bacterial (i.e., Staph)'lococcal) infec-

tlon.

It occurs in individuals that habitually lick their lips and deposit small amounts ofsaliva in the commissural angles. It is also associated with nutritional deficiencies (i.e.. vitamin B-) [riho.favinJ, vitamin B-3 [niacinJ, vitamin 8-6 [pvridoxineJ. or vitamin B-12

Io

anoc obalaminJ, or a deficiency in iron).

\ote: Nvstatin will invariably eliminate the fungal infection and an antibacterial is used lo treat the bacterial inlection ifpresent.
See

picture #78 in booklet

. Hyperkeratosis
. Leukoplakia . Epidermolysis bullosa

. White

sponge nerus

Coplrighr C

201

178 l'2012 - Dental Decks

Hairy tongue is a condition chrracterlzed by hypertrophy of the:

. Filiform papillae

. Fungiform papillae . Circumvallate papillae


. Foliate papillae

179 Coptrighr O 201l-2012 - Dental Decls

White sponge nevus is an autosomal dominant trait due to mutations ofkeratin 4 and/or 13. tt has no sex preference. lt presents as an asymptomatic. deeply folded, white or gray lesion that may affect several mucosal sites. Lesions tend to be somewhat thickened and have a spongy consistency. The presentation intraorally is almost a)ways bilateral and symmetric and usually appears early in life, typically before puberty. The buccal mucosa is the usual location. See picture #79 in booklet

Important: There is no treatment for $,hite sponge nevus! howevel since the condition is
perfectiy benign, the prognosis is excellent. There are no serious clinical aomplications. Note: lt is often mistaken for leukoplakia.

Nlicroscopically, a featufe that unique to prickle cells lbund in rvhite sponge nevus is perinuclear eosinophilic condensation of cytoplasm. Note: Hyperkeratosis is an abnonnal increase in the thickness of the keratin layer (stratum corneun) of Ihe epithelium. It is one ofthe most common white cheek lesions ofthe oral mucous rnembranes (often in an area of cht onic cheek biting) and presents as being thick and
scailr.

Epidermolysis bullosa is a general term that encompasses one acquired and several genetic t ariet\es (dystrophic, .jtnctional, sizpler/ of disease that are basically chamcterized by the lbnnation ofblisters at sites of minor trauma (especiall ot'er tlrc elbov and kleesl. Several
genetic types range from autosomal dominant to autosomal recessjve. The feature common to a1l subtypes of epidermolysis bullosa is bulla formation from minor provocation, usually or er areas ofstress, such as the elbows and knees. Oral lesions are panicularly common and :er ere in the recessive forms and uncommon in the acquired form. These lesions include bullae. scaning. and hypoplastic teeth. These lesions are most pronounced in the type known as recessi!e dystrophic epidermolysis bullosa.

Hain tongue is a benign condition ofthe tongue. The dorsum ofthe tongue appears furry due to the elongated papillae. The color varies lrom yellowish-white to brown or black. \ote: It is associated with poor oral hygiene, extended use ofantibiotics, cofticosteroids. hr droven peroxide and smoking. See picture #80 in booklet
The four trpes of papillae present on the tongue: 1. Filiform: most numerous, small cones aranged in "V"- shaped rows paralleling the sulcus terminalis. They are characterized by the absenc oftaste buds and increa-

keratinization. l. Fungiform: scattered among the filiform papillae, they are flattened, mushroom shaped and found mainly at the tip and lateral margins. ,'V',- shaped -1. Circumyallate: iargest, have circular shape. Arranged in an inverted rou tolard the back ofthe tongue. Associated with the ducts ofvon Ebner's glands. Thev are the least numerous of all papillae. l. Foliat: found on lateral margins as 3-4 vertical folds.
sed

. Lupus erylhematosus
. Erythema multiforme

. Pemphigus vulgaris . Lichen planus

180 Cop)Tiglt O 2011-2012 Dental Decks

. Leukoplakia . White sponge nelus

. Candidiasis . Lichen planus

t81
CoplriSht O 201 I,20
12 -

Dmral Dcls

These lacelike white striae, the so-called Wickham's striae, are a classic presentation lichen planus. They are often bilateral and symmetrical in distribution.

of

Lichen planus is a fairly common inflanmatory disease that usually affects the skin, the mouth, or sometimes both. It aflects women slightly more than men, and occurs most often in middle-aged adults. TJymphocytes destroy basal keratinocytes, however. the reason for this immunologically mediated phenomenon is unknown. Lichen planus of the mouth most commonly affects the buccal mucous membrane. It may also been seen on the tongue, lips, hard palate and gingiva. The lace-like striae are usually asymptomatic, but sometimes the patient may complain ofa burning sensation. The intraoral lesions respond to topical steroid therapy. In addition to the usual form of lichen planus, there are lrvo other foms, bullous and erosive. [n the bullous forrn, fluid-filled vesicles project from the surface. [n the erosive form, the lesions are intensely red or raw-appearing. When drese lesions of erosive lichen planus involve the gingiva, they resemble desquamative gingivitis. See picture #81 in booklet

The microscopic appearance of lichen planus is characteristic and pathognomonic: . Hyperparakeratosis with thickening ofthe granular cell layer

. Deyelopment ofa "salv tooth" appearance ofthe rete pegs . Degeneration ofthe basal Iayer ofcells . lnilltration of inflammatory cells into the subepithelial layer ofconnectrve

tissue

Oral candidiasis also kaown as thrush, causes white, curd-like patches in the mouth or lhroat. These patches typically appear on the tongue, inside ofthe cheeks, or on the palate. Oral candidiasis typically occurs in people rvith abnormal immune systems. These can include people undergoing chetrotherapy for canceq people taking immunosuppressive dmgs to protect transplanted organs, or people with HIV infection.
Candidiasis is caused by a yeasrlike fungus, C. albicans, rvhich causes an inflammatory, pruritic infection characterized by a thick, white discharge. Acute lesions appear as diftirse. curly or velvety white mucosal plaques on the cheeks. palate and tongue rhat car be lriped off, leaving a red, raw or bleeding surface. Note: Chronic lesions are erythematous.

This veast-like fungi is a normal inhabitant of the oral cavity and vaginal tract, however it is normally held in check by the indigenous bacteria of these areas. The treatment for oral candidiasis is topical through the use of lozenges (also called trouc}esl and mouth rinses. the most widely used is nystatin, Note: Systemic treatment includes the use of lluconazole or ketoconazole.

\otes'

Acute pseudomembranous candidiasis is the most common fom of oral candidiasis and is usually found on the buccal mucosa, tongue and soft palate. ....r. Oral cytology smears are usel'ul for a diagnosis (it will reveal budding otgqnisnts *ith branching pseudohyphae). 2. Angular cheilitis (Perleche) has also been linked to C. albicans. 3.Factors that may stimulate Candida growth include: the extended use ofantibiotics, steroids, diabetes, pregnancy, or a deficiency in iron, folate. Mtamin Brr or zinc.

ORAL PATHOLOGY

WLes

The "white patch" seen below has been present for nine months on a patient who is a heavy pipe smoker. What is the treatment of choice?

142
CopyriShr C 2011

20ll

Denral Decks

ORAL PATHOLOGY

WLes

A 3s-year-old healthy female prsents to your office for routine dental work While completing the restorativ treatment on the LL quadrant you notice that her tongu has multiple irregularly shaped red lesions that have a white trorder. You make a note in your chart. When she returns two weeks later for the restorative work on the LR quadrant, you note that there are still lesions, but in different locations on the tongue with different shapes. What is your diagnosis?

. Fissured tongue . Macroglossia . Geographic tongue


. Hairy tongue
183 Copvriglrt

a):0ll-l0ll

- Dental Decks

Clinically. this is leukoplakia and should ahvays be biopsied due to the possibility ofit being a premalignant lesion. Remember: Pipe smoking is thought to bc one of the nrost impona nt predisposing ctiologic factors for thc dcvclopmcnt ofa lcukoplakia.

Important: The floor ofthe moutir,


cinoma occurring in leukoplakia.

tongue and lower lip ale the regions at greatcst risk for car-

Stomatitis nicotina is related to pipe smoking (as uell as cigor snrokfugl and occurs exclusively on thc palate. It aff'ccts malcs prcdominantly. The palate first appea$ rcd and inflamqd. Soon it develops a diff-usc, grayish-white, thickened, multinodular popular appearance rvith a small red "spot" in thc ccntcr ofeach tiny nodule. This "spot" corrcsponds to the orifices ofpalatal salivary gland ducts. Thc treatmcnt for this condition is the cessation ofslnoking. It is usually not
considcrcd to bc a prcmalignant lesion. See picture #84 in booklet

\ote: In India,

thc usc ofsmokclcss tobacco in various forms is vcry popular. This habit, which usually involves the chewing ofa betel q\id (combined areca nut, betel le.i, tohacco dnd slack lindl. has lcd to thc dcvelopment, in a large proportion ofuscrs. ofa unique generalized fibrosis of thc oral soft tissues, called oral submucous fibrosis. It typically affects the buccal mucosa. Lips. rctromolar arcas and thc soft palatc. Early lcsions prcscnt as a blanching of the mucosa, rmpaning a mottlcd, marble-like appcarance. Latcr lesions dcmonstrate palpablc fibrous bands running vertically in the buccal mucosa and in a circular fashion around the mouth opening or lips. As the disease progresses the mucosa becomes stiff, causing difficulty in cating and considcrably rcstricting thc paticnt's ability to open the mouth (risnl ir,r/. Ifthc tongue is involvcd. it becomes sritTand has a diminishcd sizc. Thqre is a fibroclastic transformation of thc.iuxta-epithclial connecti\ e tissues and an increased risk of oral carcinoma liom the tobacco ofthe quid.

ImDortant: Betel quid and smokcless tobacco also increascs the risk ofvcrrucous carcinoma.

***

lr rs also called eD4hema migrans. benign migratory glossitis, rvandcring rash ofthe tongue, erythema lreara migrans. and stomatitis arcata migrans.

Grographic tongue is a hannless and very common condition in u'hich there is dcsquamation ofthe filiform papillae. h is charactcrizcd by having one or more iregularly shapcd patches on thc longuc. Thc a-r.r arca is redder lhan the rcst of the tongue and the edges of the patch are whitish in color Thcsc r:rchcs eppcar and remain for a short time. heal. then reappear aI anothcr site. The patchcs usually do rrrr r.sprrnd to reatment but disappcnr spontaneously. Note: The patient may complain ofa slight burnr:1g ofrhe tonguc. Se picture #82 in booklt Fis!ured tongue /a/.f.r tttlleclsctotultongue) is charactcrized by a deep median fissure wilh laterally ra.iratrns srrrovcs. Thc lateral groovcs vary in number but are usually symmctrical in arrangcmcnt. It is rarc r:r .illdrcn and increascs in incidence with age. The fissuring occurs across the dorsum ofthe tonere :r.J rs u'ua11r' asyrnptomatic. but may bccomc painful if infccted with Candida Albicans.
See Dictur #83

in booklet
Syndrome (dlong
h

\ote:

Fissurcd tongue is lbund in NlelkerssorFRosenthal

ilh gtdnulonatous cheili-

t:t and liaial ner\e parallsis).


Remember: Gcographic tongue often occurs in association \\'ith fissured tonguc

\lacroglossia is tonguc enlargement that leads to ftrnctional and cosmctic problenls. Although this is a relarivcl-v uncommon disorder. it may causc significant morbidity. Thcrc is no clear definition of macroglossia and it may bc dcfined in relative, funclional, or structural terms. Causes includci . An inflammatory cause is chronic glossitis . Traumatic causes includc postoperative edema

. \Ietabolic

causes are myxedcma, amyloidosis, lipoid proteinosis. chronic steroid therapy, and acromegal) . The congenital causes are p n1ary idiopathic rnacroglossia. cretinism, hemangioma, lymphangioma, Beckwith-Weidmann Syndrome, Dorvn Syndrome, generalized gangliosidosis syndrome, and anv of the MuooDolvsaccharidoses.

You might also like